В урне 4 белых и 2 черных шара из урны наудачу извлекают 2 шара: В урне 4 белых и 2 чёрных шара. Из этой урны наудачу извлечены 2 шара. Какова вероятность того, что эти шары разного цвета?

Теорема умножения вероятностей зависимых событий



Вероятность совместного появления двух зависимых событий равна произведению вероятности одного из них на условную вероятность другого, вычисленную в предположении, что первое событие уже произошло:

В нашем примере:  – вероятность того, что из полной колоды будут извлечены 2 червы подряд.

Аналогично:  – вероятность того, что сначала будет извлечена карта другой масти и затем черва.

Вероятность события   получилась заметно больше вероятности события , что, в общем-то, было очевидно и безо всяких вычислений.

И, само собой, не нужно питать особых надежд, что из конверта с десятью лотерейными билетами (Задача 48) вы вытяните 3 выигрышных билета подряд:
 (впрочем, это щедро).

Да, совершенно верно – теорема умножения вероятностей зависимых событий естественным образом распространяется и на бОльшее их количество.
Закрепим материал несколькими типовыми задачами:

Задача 49
В урне 4 белых и 7 черных шаров. Из урны наудачу один за другим извлекают два шара, не возвращая их обратно. Найти вероятность того, что:

а) оба шара будут белыми;
б) оба шара будут чёрными;
в) сначала будет извлечён белый шар, а затем – чёрный.

Обратите внимание на уточнение «не возвращая их обратно». Этот факт говорит нам о том, что события зависимы. Действительно, а вдруг извлечённые шары возвращают обратно? В случае возвратной выборки вероятности извлечения чёрного и белого шара меняться не будут, и в такой задаче уже следует руководствоваться теоремой умножения вероятностей НЕзависимых событий. 

Решение: всего в урне: 4 + 7 = 11 шаров, и мы приступаем к их извлечению:

а) Рассмотрим события  – первый шар будет белым,  – второй шар будет белым и найдём вероятность события , состоящего в том, что 1-й шар будет белым и 2-й белым.

По классическому определению вероятности: . Предположим, что белый шар извлечён, тогда в урне останется 10 шаров, среди которых 3 белых, поэтому:
 – вероятность извлечения белого шара во 2-м испытании при условии, что до этого был извлечён белый шар.

По теореме умножения вероятностей зависимых событий:
 – вероятность того, что оба шара будут белыми.

б) Найдём вероятность события , состоящего в том, что 1-й шар будет чёрным и 2-й чёрным.

По классическому определению:  – вероятность того, что в 1-м испытании будет извлечён чёрный шар. Пусть чёрный шар извлечён, тогда в урне останется 10 шаров, среди которых 6 чёрных, следовательно:
 – вероятность того, что во 2-м испытании будет извлечён чёрный шар при условии, что до этого был извлечен чёрный шар.

По теореме умножения вероятностей зависимых событий:
 – вероятность того, что оба шара будут чёрными.

в) Найдём вероятность события  (сначала будет извлечён белый шар и затем чёрный)
После извлечения белого шара (с вероятностью ) в урне останется 10 шаров, среди которых 3 белых и 7 чёрных, таким образом:
 – вероятность того, что во 2-м испытании будет извлечён чёрный шар при условии, что до этого был извлечен белый шар.

По теореме умножения вероятностей зависимых событий:
 – искомая вероятность.

Ответ:

Данную задачу нетрудно проверить через теорему сложения вероятностей образующих полную группу. Для этого найдём вероятность 4-го недостающего события:
 – того, что сначала будет извлечён чёрный шар и затем белый.

События  образуют полную группу, поэтому сумма их вероятностей  должна равняться единице:
, что и требовалось проверить. …Какая, однако, полезная и рабочая теорема!

И я сразу предлагаю проверить, насколько хорошо вы усвоили изложенный материал:

Задача 50
Какова вероятность того, что из колоды в 36 карт будут извлечены два туза подряд?

Задача 51
В урне 6 черных, 5 красных и 4 белых шара. Последовательно извлекают три шара. Найти вероятность того, что

а) третий шар окажется белым, если до этого был извлечён черный и красный шар;
б) первый шар окажется черным, второй – красным и третий – белым.

Решения и ответы в конце книги.

Надо сказать, что многие из рассматриваемых задач разрешимы и другим способом, но чтобы не возникло путаницы, пожалуй, вообще о нём умолчу.

Наверное, все заметили, что зависимые события возникают в тех случаях, когда осуществляется некоторая цепочка действий. Однако сама по себе последовательность действий ещё не гарантируют зависимость событий. Так, при последовательном подбрасывании монеты результат любого броска никак не зависит от предыдущих исходов. Это, кстати, важный момент, развевающий одно распространённое заблуждение, к которому мы вернёмся позже.

Ну а сейчас мы возвращаемы к нашим урнам. Если в задачах на теорему умножения вероятностей независимых событий хозяйничают стрелки, то здесь происходит самое настоящее нашествие урн с шарами =)
Задача 52
Из урны, в которой находится 6 белых и 4 черных шара, извлекаются наудачу один за другим три шара. Найти вероятность того, что:

а) все три шара будут черными;
б) будет не меньше двух шаров черного цвета.

Решение: всего: 6 + 4 = 10 шаров в урне.

Событий в данной задаче будет многовато, и в этой связи целесообразнее использовать смешанный стиль оформления, обозначая прописными латинскими буквами только основные события. Надеюсь, вы уже поняли, по какому принципу подсчитываются условные вероятности:

а) Рассмотрим событие:  – все три шара будут черными.

По теореме умножения вероятностей зависимых событий:
 

б) Этот пункт интереснее, рассмотрим событие:  – будет не меньше двух шаров черного цвета. Данное событие состоит в двух несовместных исходах: либо все шары будут чёрными (событие ) либо 2 шара будут чёрным и 1 белым – обозначим последнее событие буквой .

Событие  включается в себя три несовместных исхода:

в 1-м испытании извлечён белый шар и во 2-м и в 3-м испытаниях – чёрные шары
или
в 1-м испытании извлечён ЧШ и во 2-м – БШ и в 3-м – ЧШ
или
в 1-м испытании извлечён ЧШ и во 2-м – ЧШ и в 3-м – БШ.

По теоремам сложения вероятностей несовместных и умножения вероятностей зависимых событий:
 – вероятность того, что среди трёх последовательно извлеченных шаров будет 2 чёрных и 1 белый шар.

Примечание: на всякий случай озвучу примерный ход рассуждений при конструировании, например, последнего произведения :
«в 1-м испытании с вероятностью  извлекается ЧШ, после чего в урне останется 9 шаров, среди которых 6 белых и 3 чёрных. И во 2-м испытании с вероятностью  извлекается  БШ, после чего в урне останется 8 шаров, среди которых 5 белых и 3 чёрных. И в 3-м испытании с вероятностью  будет снова извлечён ЧШ»

По той же теореме сложения вероятностей несовместных событий:
 – вероятность того, что среди трёх последовательно извлеченных шаров будет не менее двух черных.

Ответ:

Вы просто не сможете от этого отказаться 🙂

Задача 53
Из 20 экзаменационных билетов 3 содержат простые вопросы. Пять студентов по очереди берут билеты. Найти вероятность того, что хотя бы одному из них достанется билет с простыми вопросами

А почему бы и нет? Ситуация более чем реалистичная: представьте, начался экзамен, в аудиторию пригласили 5 человек. Проведите самостоятельное исследование – какова вероятность того, что хоть кому-то из этих пяти добровольцев повезёт с билетом?

К  задаче о сдаче экзамена мы вернёмся в конце параграфа, а пока рассмотрим ещё одну стандартную задачу о перекладывании шаров из урны в урну:

Задача 54
В первой урне содержится 12 шаров, из них 7 белых, во второй – 6 шаров, из них 3 белых. Из первой урны во вторую наудачу перекладывают один шар, а затем из второй урны наудачу извлекают один шар. Найти вероятность того, что он окажется белым.

Решение: по условию, из первой урны во вторую наудачу перекладывают один шар, и, очевидно, он может быть как белым, так и не белым. В этой связи необходимо рассмотреть 2 несовместные гипотезы:

 – из 1-й урны во 2-ю будет переложен белый шар;
 – из 1-й урны во 2-ю будет переложен не белый шар.

Обозначим через  зависимое событие – после перекладывания шара из 2-й урны будет извлечён белый шар.

Несовместные исходы удобно расписать по пунктам:
1) По классическому определению:  – вероятность того, что из 1-й урны во вторую будет переложен белый шар. Пусть гипотеза  осуществилась, тогда во второй урне стало 7 шаров, среди которых теперь 4 белых шара. Таким образом:
 – вероятность того, что из второй урны будет извлечен белый шар при условии, что туда был переложен белый шар.

По теореме умножения вероятностей зависимых событий:
 – вероятность того, что во 2-ю урну будет переложен белый шар и после этого из 2-й урны будет извлечён белый шар.

2) По классическому определению:  – вероятность того, что из 1-й урны во вторую будет переложен не белый шар. Пусть гипотеза  осуществилась, тогда во второй урне стало 7 шаров, среди которых по-прежнему 3 белых. Таким образом:  – вероятность того, что из второй урны будет извлечен белый шар при условии, что туда был переложен не белый шар.

По теореме умножения вероятностей зависимых событий:
 – вероятность того, что из 1-й урны во 2-ю будет переложен не белый шар и после этого из 2-й урны будет извлечён белый шар.

Подводим итог. По теореме сложения вероятностей несовместных событий:
 – вероятность того, что из 2-й урны будет извлечён белый шар.

Ответ:

Более интересная вариация по теме для самостоятельного разбора:

Задача 55
В первой урне находится 3 белых и 2 черных шара, во второй – 4 белых и 4 черных. Из первой урны во вторую наудачу перекладывают 2 шара. Найти вероятность того, что из второй урны будет извлечён белый шар.

Для решения задания нужно рассмотреть 3 несовместные гипотезы, привлечь на помощь комбинаторику и воспользоваться типовой задачей на классическое определение вероятности.

Иногда встречаются задачи повышенной комбинационной сложности – с двумя последовательными перемещениями шаров из 1-й во 2-ю урну, из 2-й в 3-ю и финальным извлечением шара из последней урны.

И в заключение этого параграфа разберём прелюбопытнейшую задачу, которой я вас заманивал в самом начале книги =) Даже не разберём, а проведём небольшое практическое исследование. Выкладки в общем виде будут громоздкие, поэтому рассмотрим конкретный пример:

Петя сдаёт экзамен по теории вероятностей, при этом 20 билетов он знает хорошо, а 10 плохо. Предположим, в первый день экзамен сдаёт часть группы, например, 16 человек, включая нашего героя. В общем, ситуация до боли знакома: студенты один за другим заходят в аудиторию и тянут билеты.

Очевидно, что последовательное извлечение билетов представляет собой цепь зависимых событий, и возникает насущный вопрос: в каком случае Пете с бОльшей вероятностью достанется «хороший» билет – если он пойдёт «в первых рядах», или зайдёт «посерединке», или если будет тянуть билет в числе последних? Когда лучше заходить?

Сначала рассмотрим «экспериментально чистую» ситуацию, в которой Петя сохраняет свои шансы постоянными – он не получает информацию о том, какие вопросы уже достались однокурсникам, ничего не учит в коридоре, ожидая своей очереди, и т.д.

Рассмотрим событие:  – Петя зайдёт в аудиторию самым первым и вытянет «хороший» билет. По классическому определению вероятности: .

Как изменится вероятность извлечения удачного билета, если пропустить вперёд отличницу Настю? В этом случае возможны две несовместные гипотезы:

 – Настя вытянет «хороший» (для Пети) билет;
 – Настя вытянет «плохой» билет (т. е. увеличит шансы Пети).

Событие же  (Петя зайдёт вторым и вытянет «хороший» билет) становится зависимым.

1) Предположим, что Настя с вероятностью  «увела» у Пети один удачный билет. Тогда на столе останутся 29 билетов, среди которых 19 «хороших». По классическому определению вероятности: 
 2) Теперь предположим, что Настя с вероятностью  «избавила» Петю от 1-го «плохого» билета. Тогда на столе останутся 29 билетов, среди которых по-прежнему 20 «хороших». По классическому определению: 

Используя теоремы сложения вероятностей несовместных и умножения вероятностей зависимых событий, вычислим вероятность того, что Петя вытянет «хороший» билет, будучи вторым в очереди:

 – вероятность… осталось той же!
Хорошо, рассмотрим событие:  – Петя пойдёт третьим, пропустив вперёд Настю и Лену, и вытащит «хороший» билет.

Здесь гипотез будет больше: дамы могут «обокрасть» джентльмена на 2 удачных билета, либо наоборот – избавить его от 2 неудачных, либо извлечь 1 «хороший» и 1 «плохой» билет. Если провести аналогичные рассуждения, воспользоваться теми же теоремами, то… получится такое же значение вероятности !
И так далее.

Таким образом, чисто с математической точки зрения, без разницы, когда идти – первоначальные вероятности останутся неименными. НО. Это только усреднённая теоретическая оценка, так, например, если Петя пойдёт последним, то это вовсе не значит, что ему останутся на выбор 10 «хороших» и 5 «плохих» билетов в соответствии с его изначальными шансами. Данное соотношение может варьироваться в лучшую или худшую сторону, однако всё же маловероятно, что среди билетов останется «одна халява», или наоборот – «сплошной ужас». Хотя «уникальные» случаи не исключены – всё-таки тут не 3 миллиона лотерейных билетов с практически нулевой вероятностью крупного выигрыша. Поэтому «невероятное везение» или «злой рок» – это слишком уж преувеличенные высказывания. Даже если Петя знает всего лишь три билета, то его шансы составляют 10%, что заметно выше нуля.

Математика и «чистый эксперимент» – это хорошо, но какой стратегии и тактики выгоднее придерживаться в реальных условиях? Безусловно, следует принять во внимание субъективные факторы, например, «скидку» преподавателя для «храбрецов» или его усталость к концу экзамена. Зачастую эти факторы могут быть и решающими, но в заключительных рассуждениях мы всё же остановимся на вероятностных аспектах:

Если Вы готовы к экзамену хорошо, то лучше идти «в первых рядах». Пока билетов полный комплект, постулат «маловозможные события не происходят» работает на Вас гораздо в бОльшей степени. Согласитесь, что намного приятнее иметь соотношение «30 билетов, среди которых 2 плохих», чем «15 билетов, среди которых 2 плохих». А то, что два неудачных билета на отдельно взятом экзамене (а не по средней теоретической оценке!) так и останутся на столе – вполне и вполне возможно.

Теперь рассмотрим «ситуацию Пети» – когда студент готов достаточно хорошо, но с другой стороны, и «плавает» тоже неплохо. В этом случае целесообразно пропустить вперёд 5-6 человек, и ожидать подходящего момента вне аудитории. Дальше по ситуации. Довольно скоро начнёт поступать информация, какие билеты вытянули однокурсники (снова зависимые события!), и на «заигранные» вопросы можно больше не тратить силы – учите и повторяйте другие билеты, повышая тем самым первоначальную вероятность своего успеха.
Если «первая партия» экзаменующихся «избавила» вас сразу от 3-4 трудных (лично для Вас) билетов, то выгоднее как можно быстрее попасть на экзамен – именно сейчас шансы значительно возросли. Постарайтесь не упускать момент – всего несколько пропущенных вперёд человек, и преимущество, скорее всего, растает. Если же наоборот, «плохих» билетов вытянули мало – ждите. Через несколько человек эта «аномалия» опять же с большой вероятностью, если не исчезнет, то сгладится в лучшую сторону.
В «обычном» и самом распространённом случае выгода тоже есть, ибо расклад
«24 билета / 8 плохих» лучше соотношения «30 билетов / 10 плохих». Почему? Трудных билетов теперь не десять, а восемь! С удвоенной энергией штудируем материал!

Если Вы готовы неважно или плохо, то само собой, лучше идти в «последних рядах» (хотя возможны и оригинальные решения, особенно, если нечего терять). Существует небольшая, но всё же ненулевая вероятность, что на столе останутся относительно простые вопросы + дополнительная зубрёжка + шпоры, которые отдадут отстрелявшиеся сокурсники =) И, да – в совсем критической ситуации есть ещё следующий день, когда экзамен сдаёт вторая часть группы 😉

Какой можно сделать вывод? Субъективный оценочный принцип «кто идёт раньше, тот готов лучше» находит внятное вероятностное обоснование!

Ни пуха Вам, ни пера, ни холлофайбера!

1. 7. Формула полной вероятности

1.6.5. Условная вероятность – что это такое?

| Оглавление |



Полную и свежую версию этой книги в pdf-формате,
а также курсы по другим темам можно найти здесь.

Также вы можете изучить эту тему подробнее – просто, доступно, весело и бесплатно!

С наилучшими пожеланиями, Александр Емелин


Более трудные задачи на сложение и умножение вероятностей

В уроке «Действия над вероятностями» мы познакомились со сложением и умножением вероятностей и простейшими примерами этих действий. В контрольных работах и на экзамене попадаются и задачи поинтересней (посложнее), в которых необходимо применять сразу и сложение и умножение вероятностей. На этой странице — решения таких задач. Как это часто бывает с задачами на нахождение вероятностей, рассматривается урна, в которой находится сколько-то шаров и из урны вынимается сколько-то шаров, а требуется найти вероятность того, что выбранный шар — такого-то или иного цвета.

Пример 1. В урне 9 белых и 7 чёрных шаров. Из урны вынимают (одновременно или последовательно) два шара. Найти вероятность того, что оба шара будут белыми.

Решение. Обозначим через a количество белых шаров, а через b — количество чёрных шаров. По теореме умножения вероятностей

Подставляем в полученную формулу значения количества белых и чёрных шаров и получаем:

Ответ: вероятность того, что оба шара будут белыми, равна 0,3.

Пример 2. В урне 9 белых и 7 чёрных шаров. Из урны вынимаются сразу два шара. Найти вероятность того, что эти шары будут разных цветов.

Решение. Событие может появиться в двух несовместных вариантах: бч и чб. По теремам сложения и умножения вероятностей

Подставляем в полученную формулу значения количества белых и чёрных шаров и получаем:

Ответ: вероятность того, что шары будут разных цветов, равна 0,525.

Пример 3. В урне 9 белых, 7 чёрных и 6 красных шаров. Три из них вынимаются наугад. Найти вероятность того, что по крайней мере два из них будут одноцветными.

Решение. Чтобы найти вероятность события A — по крайней мере два шара будут одноцветными, — перейдём к противоположному — все шары разных цветов:

Отсюда

Подставляем в полученную формулу значения количества шаров и получаем требуемую вероятность:

Нет времени вникать в решение? Можно заказать работу!

Пройти тест по теме Теория вероятностей и математическая статистика


Попадаются и задачи на умножение вероятностей для нескольких событий. Поэтому следует привести формулы для вычисления вероятностей нескольких событий. Для зависимых событий она имеет вид

,

Для независимых событий:

.


Пример 4. Имеется коробка с девятью новыми теннисными мячами. Для игры берут три мяча. После игры их кладут обратно. При выборе мячей игранные от неигранных не отличают. Найти вероятность того, что после трёх игр в коробке не останется неигранных мячей.

Решение. Событие A может произойти единственным способом: первый раз, второй и третий из коробки будут вынуты неигранные мячи. Первый раз это обеспечено. Поэтому

.

Пример 5. Из полной колоды карт (52 карты) вынимают одновременно четыре карты. Рассматриваются события:

A — среди вынутых карт будет хотя бы одна бубновая;

B — среди вынутых карт будет хотя бы одна червонная.

Найти вероятность события C = A + B.

Решение. Переходим к противоположному событию — нет ни бубновой, ни червонной карты:

,

откуда получаем требуемую вероятность суммы событий:

.

НазадЛистатьВперёд>>>

Нет времени вникать в решение? Можно заказать работу!

Пройти тест по теме Теория вероятностей и математическая статистика

К началу страницы

Основные понятия теории вероятностей, непосредственное вычисление вероятностей

Действия над вероятностями

Формула полной вероятности

Формула Байеса

Независимые испытания и формула Бернулли

Распределение вероятностей дискретной случайной величины

Распределение вероятностей непрерывной случайной величины

Математическое ожидание и дисперсия случайной величины

Биномиальное распределение дискретной случайной величины

Распределение Пуассона дискретной случайной величины

Равномерное распределение непрерывной случайной величины

Нормальное распределение непрерывной случайной величины

Вероятность

— В урне 2 белых, 3 красных и 5 черных шаров.

Проблема

$\begingroup$

В урне 2 белых, 3 красных и 5 черных шаров. Случайным образом вынимаются 3 шара, по одному и без замены.

Рассчитайте вероятность извлечения последовательности цветов (белый, черный, красный), зная, что вы извлекли черный шар.

решение

Пусть E будет событием, относящимся к указанной последовательности цветов. Требуемая вероятность есть $P(E|X= 1)$, откуда следует $P(E|X= 1) = \frac{P(E⋂(X=1))}{P(X=1)} = \ frac {P (E)} {P (X = 1)} \ overset {(question1)} {=} \ frac {\ frac {2} {10} \ frac {5} {9}} \ гидроразрыва {3} {8}} {\ гидроразрыва {5} {12}} = \ гидроразрыва {1} {24}} {\ гидроразрыва {5} {12}} = \ гидроразрыва {1} { 10}$

вопрос 1 Если последовательность (белое, черное, красное) в числителе, у нас не должно быть: $\frac{3}{10}\frac{5}{9}\frac{2}{8}$ в конце результата то же самое, но, конечно, рассуждения для его получения разные. Какие рассуждения могут быть сделаны в решении?

А в знаменателе почему $\frac{5}{12}$. $P(X=1)$ указывает вероятность извлечения черного шара. Это не должно быть $\frac{5}{10}$

  • вероятность

$\endgroup$

10

$\begingroup$

Начните с вычисления вероятности извлечения черного шара. Вероятность того, что ни один из черных шаров не будет извлечен, равна $\frac{1}{2} \frac{4}{9} \frac{3}{8} = \frac{1}{12}$. Следовательно, вероятность того, что будет извлечен черный шар, равна $1 — \frac{1}{12} = \frac{11}{12}$.

Теперь вычислите вероятность появления последовательности (белое, черное, красное). Это просто: $\frac{1}{5} \frac{5}{9} \frac{3}{8} = \frac{1}{24}$.

Таким образом, искомая условная вероятность равна $\frac{\frac{1}{24}}{\frac{11}{12}} = \boxed{\frac{1}{22}}$.

Редактировать: предположим, что вместо «черный шар» нас интересует условие, что «вытягивается ровно один черный шар». Для этого учтите, что количество перестановок трех выбранных шаров (независимо от цвета и при условии, что они различимы) равно $(10) (9) (8) = 720$. Чтобы извлечь ровно один черный шар, мы должны выбрать его из $5$ вариантов и выбрать одну позицию из $3$ возможных. Затем для первой невыбранной позиции мы выбираем один из $5$ нечерных шаров, а для последней позиции мы выбираем один из оставшихся $4$ нечерных шаров. Количество способов $(5) (3) (5) (4) = 300$. Вероятность извлечения ровно один черный шар равен $\frac{300}{720} = \frac{5}{12}$. И желаемая условная вероятность будет $\frac{\frac{1}{24}}{\frac{5}{24}} = \boxed{\frac{1}{10}}$.

$\endgroup$

1

$\begingroup$

Суммарная Вселенная равна 10$ \х9\х8 =720$

Часть Вселенной исключена: мы знаем, что по крайней мере один шар черный. Таким образом, все комбинации, основанные только на красном+белом, исключаются: комбинации $5\times 4\times 3 =60$ исключаются.

Таким образом, Вселенная уменьшается до $720-60=660$

Количество комбинаций, совпадающих с последовательностью (белое, черное, красное), равно $2 \times 5 \times 3= 30$

Таким образом, вероятность равна $\frac { 30}{660}=\frac{1}{22}$

Условная вероятность означает, что вы разделите одну дробь на другую. Это слишком сложно. Легче разделить целое число (количество успехов) на другое целое число (размер ограниченной вселенной).

$\endgroup$

В урне A 7 белых и 3 черных шара, в урне B 4 белых и 6 черных шаров, в урне C 2 белых и 8 черных шаров. Одна из этих урн выбирается случайным образом с вероятностью 0,2, 0,6 и 0,2 соответственно. Из выбранной урны наугад извлекают два шара без замены. Оба шара оказались белыми. Найти вероятность того, что вынутые шары из урны C.

Вопрос

Обновлено: 26/04/2023

RS ТЕОРЕМА АГГАРВАЛА-БАЙЕСА И ЕЕ ПРИЛОЖЕНИЯ-Упражнение 30

18 видео

РЕКЛАМА

Текстовое решение

Ответ

Правильный ответ 140

Решение

Р(А)=0,2, Р(В)=0,6 и Р(С)=0,2.
Пусть E — событие, когда извлекаются 2 белых шара. Тогда
P(E/A)=.7C2.10C2,P(E/B)=.4C2.10C2,P(E/C)=.2C2.10C2
∴ требуемая вероятность
=P(C/E) =P(E/C).P(C)P(E/A).P(A)+P(E/B).P(B)+P(E/C).P(C).

Ответить

Пошаговое решение от экспертов, которое поможет вам в разрешении сомнений и получении отличных оценок на экзаменах.

Ab Padhai каро бина объявления ке

Khareedo DN Pro и дехо сари видео бина киси объявление ки rukaavat ке!


Похожие видео

Урна A содержит 2 белых, 1 черный и 3 красных шара, урна B содержит 3 белых, 2 черных и 4 красных шара, а в урне C 4 белых, 3 черных и 2 красных мячи. Наугад выбирают одну урну и из нее наугад вынимают 2 шара. Если выбранные шары оказались красными и черными, какова вероятность того, что оба шара из урны B?

Есть три урны, содержащие 2 белых и 3 черных шара, 3 белых и 2 черных шара и 4 белых и 1 черный шар соответственно. Существует равная вероятность того, что каждая урна будет выбрана. Из выбранной урны случайным образом извлекают шар, и он оказывается белым. Найти вероятность того, что вынутый шар был из второй урны.

32530615

Три урны содержат 2 белых и 3 черных шара, 3 белых и 2 черных шара, 4 белых и 1 черный шар соответственно. Из случайно выбранной урны извлекли один шар, и он оказался белым. Найти вероятность того, что он был извлечен из первой урны.

51237386

Три одинаковые урны содержат белые и черные шары. В первой урне 2 белых и 3 черных шара, во второй урне 3 белых и 2 черных шара, а в третьей урне 1 черный и 4 белых шара. Наугад выбирают урну и извлекают из нее шар. Если вынутый шар белый, какова вероятность того, что будет выбрана первая урна?

234812974

В урне A 2 белых и 4 черных шара, в урне B 5 белых и 7 черных шаров. Если случайным образом заменить по одному шару из A и B и извлечь шар из B, то вероятность того, что шар окажется белым, равна 9.0003

308714505

В одной урне 8 белых и 5 черных шаров, а в другой урне 5 белых и 6 черных шаров. Наугад выбирается одна урна и из нее вынимаются два шара. Найдите вероятность того, что шары белые.

320218559

Текст Решение

В урне 8 белых и 6 черных шаров. Два шара извлекаются из урны один за другим без возврата. Какова вероятность того, что оба вынутых шара белые?

425868197

Предположим, что есть 3 урны, содержащие 2 белых, 3 черных шара, 4 белых 1 черных и 3 белых, 2 черных шара соответственно. Существует равный шанс для выбора урны. Из случайно выбранной урны вынимают один шар, и он оказывается белым. Проблема. То, что он был взят из первой урны, равно

630437056

. В урне A 2 белых, 1 черный и 3 красных шара, в урне B 3 белых, 2 черных и 4 красных шара, а в урне C 4 белых, 3 черных и 2 красных мячи. Наугад выбирают одну урну и из нее наугад вынимают 2 шара. Если выбранные шары оказались красными и черными, какова вероятность того, что оба шара из урны B?

642566704

В урне 2 белых и 2 черных шара. Наугад извлекается шар. Если он белый, он не возвращается в урну. В противном случае он заменяется другим шаром того же цвета. Процесс повторяется. Найти вероятность того, что третий извлеченный шар окажется черным.

643823834

В урне 2 белых и 2 черных шара. Наугад извлекается шар. Если он белый, то в урну его не возвращают. В противном случае он заменяется другим шаром того же цвета. Процесс повторяется. Найти вероятность того, что третий извлеченный шар окажется черным.

644961989

В урне 2 белых и 2 черных шара. Наугад извлекается шар. Если он белый, он не помещается в урну, в противном случае он заменяется вместе с другим шаром того же цвета. Процесс повторяется, найти вероятность того, что третий вытащенный шар будет черным.

646282299

Калаш I содержит 2 белых и 3 черных шара, Калаш II содержит 4 белых и 1 черный и Калаш III содержит 3 белых и 4 черных шара. Случайным образом выбирается урна и извлекается шар. Если вытащенный шар белый, то какова вероятность того, что его вытащат из урны I?

646611223

Три урны содержат 2 белых и 3 черных шара, 3 белых и 2 черных шара и 4 белых и 1 черный шар соответственно.

Матпрофи комплексные числа: Конспект по комплексным числам. Формулы и термины / Библиотека МатПрофи.ком

Комплексные числа — презентация онлайн

После изучения темы «Комплексные числа
учащиеся должны:
Знать:
алгебраическую, геометрическую и тригонометрическую формы
комплексного числа.
Уметь:
•производить над комплексными числами операции сложения,
умножения, вычитания, деления, возведения в степень, извлечение
корня из комплексного числа;
•переводить комплексные числа из алгебраической формы в
геометрическую и тригонометрическую;
•пользоваться геометрической интерпретацией комплексных чисел;
•в простейших случаях находить комплексные корни уравнений с
действительными коэффициентами.

3. Какие числовые множества Вам знакомы?

I. Подготовка к изучению нового материала
Какие числовые множества Вам знакомы?
N
Z
Q
N Z Q R
R
Числовая система
Натуральные
числа, N
Целые числа, Z
Рациональные числа, Q
Действительные числа,
R
Комплексные
числа, C
Допустимые
алгебраические
операции
Сложение,
умножение
Сложение, вычитание,
умножение
Сложение, вычитание,
умножение, деление
Сложение, вычитание,
умножение, деление,
извлечение корней из
неотрицательных чисел
Все операции
Частично
допустимые
алгебраические
операции
Вычитание, деление,
извлечение корней
Деление,
извлечение корней
Извлечение корней из
неотрицательных
чисел
Извлечение корней
из произвольных
чисел
Минимальные условия, которым должны удовлетворять
комплексные числа:
С1) Существует квадратный корень из , т. е. существует
комплексное число, квадрат которого равен .
С2) Множество комплексных чисел содержит все действительные
числа.
С3) Операции сложения, вычитания, умножения и деления
комплексных чисел удовлетворяют обычным законам
арифметических действий (сочетательному, переместительному,
распределительному).
Выполнение этих минимальных условий позволяет определить
все множество С комплексных чисел.

6. Мнимые числа

i = -1, i – мнимая единица
i, 2i, -0,3i — чисто мнимые числа
Арифметические операции над чисто мнимыми числами
выполняются в соответствии с условием С3.
3i 13i 3 13 i 16i
3i 13i 3 13 i i 39i 2 39
i 7 i 2 i i
3
В общем виде правила арифметических операций с чисто мнимыми
числами таковы:
a b i;
a bi ab i;
ai bi
ai bi a b i;
ai bi abi a
где a и b — действительные числа.
2

7. Комплексные числа

Определение 1. Комплексным числом называют сумму
действительного числа и чисто мнимого числа.
z a bi C a R, b R,
i мнимая единица.
a Re z , b Im z
Определение 2. Два комплексных числа называют
равными, если равны их действительные части и равны
их мнимые части:
a bi c di a c, b d .

8. Классификация комплексных чисел

Комплексные числа
a + bi
Действительные числа
b=o
Рациональные
числа
Иррациональные
числа
Мнимые числа
b≠o
Мнимые числа с
ненулевой
действительной
частью
a ≠ 0, b ≠ 0.
Чисто
мнимые
числа
a = 0, b ≠ 0.

9. Арифметические операции над комплексными числами

(а + bi) + (c + di) = (а + с) + (b + d)i
(а + bi) — (c + di) = (а — с) + (b — d)i
(а + bi)·(с + di) = (ac — bd) + (ad + bc)i
a bi (a bi)( c di) ac bd bc ad
2
2
i
2
2
c di (c di)( c di) c d
c d

10. Сопряженные комплексные числа

Определение: Если у комплексного числа сохранить
действительную часть и поменять знак у мнимой части, то
получится комплексное число, сопряженное данному.
Если данное комплексное число обозначается буквой z, то
сопряженное число обозначается z :
z x yi z x yi
Из всех комплексных чисел действительные числа (и только они)
равны своим сопряженным числам.
Числа a + bi и a — bi называются взаимно сопряженными
комплексными числами.

11. Свойства сопряженных чисел

1. Сумма и произведение двух сопряженных чисел есть число
действительное.
z z ( a bi ) ( a bi ) 2a
z z (a bi )( a bi ) a 2 (bi ) 2 a 2 b 2
2. Число, сопряженное сумме двух комплексных чисел, равно
сумме сопряженных данным числам.
z1 z2 z1 z2
3. Число, сопряженное разности двух комплексных чисел, равно
разности сопряженных данным числам.
z1 z2 z1 z2
4. Число, сопряженное произведению двух комплексных чисел, равно
произведению сопряженных данным числам.
z1z2 z1 z2

12. Свойства сопряженных чисел

5. Число, сопряженное п-ой степени комплексного числа z,
равно п-ой степени числа, сопряженного к числу z, т. е.
z n ( z)n , n N
6. Число, сопряженное частному двух комплексных чисел, из
которых делитель отличен от нуля, равно частному
сопряженных чисел, т.е.
a bi a bi
c di c di

13. Степени мнимой единицы

По определению первой степенью числа i является
1
само
число i, а второй степенью – число -1:
i1 = i, i2 = -1
.
Более высокие степени числа i находятся следующим
1
образом:
i4 = i3 ∙ i = -∙i2= 1;
i5 = i4 ∙ i = i;
i6 = i5 ∙ i = i2= — 1 и т.д.
Очевидно, что при любом натуральном n
i4n = 1;
i4n +2 = — 1
i4n+1 = i;
i4n+3 = — i.

14. Извлечение квадратных корней из комплексных чисел в алгебраической форме.

• Определение. Число w называют квадратным корнем из
2
комплексного числа z, если его квадрат равен z: w z
• Теорема. Пусть z=a+bi – отличное от нуля комплексное число.
Тогда существуют два взаимно противоположных комплексных
числа, квадраты которых равны z. Если b≠0, то эти два числа
выражаются формулой:
w
a2 b2 a
i signb
2
a 2 b 2 a
, где
2
1, если b 0
signb 1, если b 0
0, если b 0
При b 0, a 0 имеем : w a , при b 0, a 0 имеем : w i a .

15. Геометрическое изображение комплексных чисел.

Комплексному числу z на координатной плоскости
соответствует точка М(a, b).
Часто вместо точек на плоскости берут их
радиусы-векторы
OM
Определение: Модулем комплексного числа z = a + bi
называют неотрицательное числоa 2 b2
,
равное расстоянию от точки М до начала
z a 2 b2
координат
cos
y
М (a, b)
b
φ
O
a
x
a
и sin
b
a2 b2
a2 b2
аргумент комплексно го числа
;

16. Тригонометрическая форма комплексного числа

z r cos i sin
где φ – аргумент комплексного числа,
r=
a 2 b2 — модуль комплексного числа,
cos
a
a2 b2
и sin
b
a2 b2

17. Умножение и деление комплексных чисел, заданных в тригонометрической форме

Теорема
Если
1.
z1 0, z2 0
и
z1 r1 cos 1 i sin 1 , z2 r2 cos 2 i sin 2 , то:
а)
z1 z2 r1r2 cos 1 2 i sin 1 2
б)
z1 r1
cos 1 2 i sin 1 2
z2 r2
Теорема 2 (формула Муавра).
Пусть z — любое отличное от нуля
комплексное число, п — любое целое число.
Тогда
z r cos i sin r n cosn i sin n .
n
n

18. Извлечение корня из комплексного числа.

• Теорема. Для любого натурального числа n и
отличного от нуля комплексного числа z существуют
n различных значений корня n-степени.
Если
z r cos i sin ,
то эти значения выражаются формулой
2 k
2 k
wk r cos
i sin
,
n
n
где k 0,1,…, (n 1)
n

Комплексные числа и операции с ними

Содержание

Введение

Комплексная плоскость и мнимая единица

Модуль и фаза комплексного числа

Показательная форма комплексного числа. Формула Эйлера

Операции над комплексными числами. Комплексно-сопряженные числа

Выводы

Список литературы

DSPL-2.0 — свободная библиотека алгоритмов цифровой обработки сигналов

Распространяется под лицензией LGPL v3

Страница проекта на SourceForge

Обнаружили ошибку? Выделите ее мышью и нажмите

Введение

Известно, что область определения некоторых функций на множестве вещественных чисел ограничена. Например функция определена для , аналогично можно вспомнить, что функция определена для , а функция определена для .

Однако, ограниченная область определения функций на множестве вещественных чисел не означает, что , или не имеют смысла. Ограниченная область определения функций на множестве вещественных чисел говорит лишь о том, что не может быть представлено вещественным числом. Действительно, среди вещественных чисел не найти такого числа , квадрат которого был бы равен .

При решении квадратных уравнений часто возникает ситуация, когда дискриминант отрицательный. В этом случае это означает что парабола не пересекает прямую абсцисс ни в одной точке. Другими словами, корни квадратного уравнения не существуют среди вещественных значений и их также надо искать за пределами вещественных чисел.

Все бесконечное множество вещественных чисел можно представить в виде одной числовой прямой (смотри рисунок 1), на которой мы можем откладывать рациональные и иррациональные вещественные числа. Но на этой прямой нет числа , значит его надо искать вне числовой прямой. Таким образом мы должны расширить множество вещественных чисел до множества в котором значения , или уже не бессмысленны, а являются такими же обычными числами в этом расширенном множестве, как на множестве вещественных чисел.

Комплексная плоскость и мнимая единица

Естественным расширением числовой прямой является плоскость, которую называют комплексной плоскостью. Числовая прямая вещественных чисел и ее расширение до комплексной плоскости показано на рисунке 1. Любая точка на комплексной плоскости определяет одно комплексное число. Например на рисунке 1 показано число .

Рисунок 1. Расширение множества вещественных чисел до множества комплексных числел

Значение вещественного числа однозначно определяет его позицию на числовой прямой, однако для определения позиции на плоскости одного числа недостаточно.

Для «навигации» по комплексной плоскости вводятся две прямые и , которые пересекаются в начале координат. Прямая это числовая прямая, называемая реальной осью, на которой лежат все вещественные числа. Прямая называется мнимой осью и она перпендикулярна реальной оси . Оси и делят комплексную плоскость на четверти, как это показано на рисунке 1.

Любая точка комплексной плоскости задается двумя координатами и по осям и соответственно. При этом само комплексное число можно записать как , где называется реальной частью и задает координату точки комплексной плоскости на вещественной прямой , а называется мнимой частью и задает координату точки комплексной плоскости на мнимой оси .

Для того чтобы отделить одну координату от другой (реальную и мнимую части) вводят число , называемое мнимой единицей. Это так раз то число, которого не существует на множестве действительных чисел. Оно обладает особым свойством: . Тогда комплексное число может не только перемещаться по вещественной прямой вправо и влево, но и двигаться по комплексной плоскости потому что мы добавили ему слагаемое с мнимой единицей .

Мнимую единицу в математической литературе принято обозначать как , но в технике буква уже закреплена за обозначением электрического тока, поэтому чтобы избежать путаницы мы будем обозначать мнимую единицу буквой .

Если и , тогда число является действительным и располагается на реальной оси .

Если и , тогда число является чисто мнимым и располагается на мнимой оси .

Если и , тогда число располагается в одной из четвертей комплексной плоскости.

Модуль и фаза комплексного числа

Представление комплексного числа как называют алгебраической формой записи. Если из начала координат комплексной плоскости к точке восстановить вектор (смотри рисунок 1), то можно вычислить длину этого вектора как

(1)

 — неотрицательное вещественное число характеризующее длину вектора и называется модулем комплексного числа. При этом сам вектор комплексного числа повернут относительно реальной оси на некоторый угол , называемый фазой. Фаза комплексного числа может быть положительной или отрицательной, в зависимости от того в каком направлении относительно оси отсчитывать угол. Если угол поворота вектора на комплексной плоскости отсчитывать против часовой стрелки (как это показано на рисунке 1), то фаза будет принимать положительные значения, а если по часовой — то отрицательные.

Связь реальной и мнимой частей комплексного числа с его амплитудой и фазой представлено следующим выражением:

(2)

Тогда комплексное число можно представить в тригонометрической форме:

(3)

Связь угла поворота вектора комплексного числа с реальной и мнимой частью комплексного числа, представленного в алгебраической форме:

(4)

тогда

(5)

где учитывает четверть комплексной плоскости в которой расположено число :

(6)

Необходимость поправки возникает из-за того, что функция периодическая функция с периодом рад. В результате возвращает корректные значения только в интервале . Таким образом функция арктангенса не отличает четверть I от четверти III (в обоих случаях отношение положительное), а также не отличает четверть II от четверти IV (отношение отрицательное).

На рисунке 2 показаны значения параметра , в зависимости от того в какой четверти комплексной плоскости расположено число.

Рисунок 2. Значение поправки фазы комплексного числа в зависимости от расположения на комплексной плоскости.

На рисунке 2а исходное комплексное число расположено в первой четверти комплексной плоскости и .

Тогда и значение фазы комплексного числа равно:

(7)

Рассмотрим случай, когда комплексное число расположено во второй четверти комплексной плоскости (рисунок 2б), т.е. и . В этом случае и угол также будет отрицательным (красная пунктирная линия). Тогда для того, чтобы получить корректное значение фазы необходимо ввести поправку рад:

(8)

Пусть комплексное число расположено в третьей четверти комплексной плоскости (рисунок 2в), т. е. и . В этом случае и угол будет положительным (красная пунктирная линия). Тогда для того, чтобы получить корректное значение фазы необходимо ввести поправку рад:

(9)

Если расположено в четвертой четверти комплексной плоскости (рисунок 2г), т.е. и , то в этом случае и угол будет отрицательным и равным фазе комплексного числа без поправок ( рад):

(10)

Функция которая позволяет получить фазу комплексного числа c учетом четверти комплексной плоскости в которой расположено комплексное число называется функция арктангенс-2 и обозначается .

Функция арктангенс-2 присутствует во всех математических приложениях и может быть использована для расчета верного угла поворота вектора комплексного числа.

Показательная форма комплексного числа. Формула Эйлера

Мы уже рассмотрели алгебраическую и тригонометрическую формы записи комплексного числа. Помимо алгебраической и тригонометрической формы существует также показательная форма комплексного числа:

(11)

связанная с тригонометрической формой формулой Эйлера:

(12)

Cоотношение (12) легко доказать, если произвести разложение экспоненты в ряд Тейлора:

(13)

Представим ряд (13) в виде суммы четных и нечетных членов последовательности:

(14)

Рассмотрим более подробно мнимую единицу в четной и нечетной степенях.

Из определения мнимой единицы можно сделать вывод, что , тогда , в свою очередь .

Таким образом, можно сделать вывод что .

Построим аналогичным образом соотношение для нечетных степеней: , тогда , в свою очередь и окончательно можно записать: . Тогда (14) можно представить как:

(15)

В выражении (15) первая сумма по четным степеням дает разложение в ряд Тейлора функции , а вторая сумма по нечетным степеням дает разложение в ряд Тейлора функции . Таким образом, получено доказательство справедливости формулы Эйлера (12).

Необходимо отметить, что формула Эйлера является одной из важнейших в теории функций комплексного переменного. Так например при помощи формулы Эйлера можно связать математические константы и с использованием мнимой единицы :

(16)

Операции над комплексными числами. Комплексно-сопряженные числа

В данном параграфе мы кратко рассмотрим операции над комплексными числами. Сумма двух комплексных чисел и представляет собой комплексное число :

(17)

При сложении реальные и мнимые части комплексного числа также складываются. На комплексной плоскости операцию сложения можно реализовать как сложение векторов комплексных чисел по правилу параллелограмма (рисунок 3а).

Рисунок 3. Операции над комплексными числами

Разность двух комплексных чисел и представляет собой комплексное число

(18)

При вычитании реальные и мнимые части комплексного числа также вычитаются. На комплексной плоскости операцию вычитания можно реализовать как вычитание векторов по правилу параллелограмма (рисунок 3б). На первом шаге из вектора формируется вектор (обозначенный пунктирной линией на рисунке 3б), после чего вектор складывается с вектором по правилу параллелограмма.

Для того чтобы получить формулу для умножения комплексных числен необходимо перемножить два комплексных числа по правилу умножения многочленов:

(19)

Умножение комплексных проще выполнять если числа представлены в показательной форме:

(20)

При перемножении в показательной форме модули комплексных чисел перемножаются а фазы складываются. Операция произведения комплексных чисел показано на рисунке 3в.

Введем понятие комплексно-сопряженного числа. Число является комплексно-сопряженным числу .

Комплексно-сопряженные числа отличаются знаком перед мнимой частью.

Графически комплексно-сопряженные числа показаны на рисунке 3г.

При этом можно заметить, что модули комплексно-сопряженных чисел равны , а фазы имеют противоположные знаки.

Произведение комплексно-сопряженных чисел

(21)

представляет собой действительное число равное квадрату модуля этих чисел.

Из элементарных операций нам осталось рассмотреть лишь деление комплексных чисел. Рассмотрим результат деления комплексных чисел в показательной форме:

(22)

Таким образом, при делении комплексных чисел модуль частного равен частному модулей исходных чисел, а фаза равна разности фаз исходных чисел.

При этом необходимо потребовать, чтобы был не равен нулю, иначе у нас появится деление на ноль при расчете модуля частного.

Рассмотрим теперь деление комплексных чисел в алгебраической форме:

(23)

Домножим и числитель и знаменатель на число, комплексно-сопряженное знаменателю:

(24)

Выводы

В данной статье введено понятие комплексного числа и рассмотрены основные его свойства. Введено понятие мнимой единицы.

Подробно рассмотрена комплексная плоскость и представление комплексных чисел в алгебраической, тригонометрической и показательной формах. Введены понятия модуля и фазы комплексного числа.

Рассмотрены основные арифметические операции над комплексными числами.

Показано как выполнять операции сложения, вычитания в алгебраической форме, введено понятие комплексно-сопряженных чисел, а также операции умножения и деления в показательной и алгебраической формах.

Информация была полезна? Поделитесь с друзьями!

Список литературы

[1] Пантелеев А.В., Якимова А.С. Теория функций комплексного переменного и операционное исчисление в примерах и задачах. М: Высшая школа, 2011.

[2] Дубровин В.Т. Теория функций комплексного переменного. Теория и практика Казань: Казанский государственный университет, 2010. [PDF]

Последнее изменение страницы: 12.05.2022 (19:41:15)

Страница создана Latex to HTML translator ver. 5.20.11.14

Комплексные числа


Комплексный номер

Комплексное число представляет собой комбинацию
Действительного числа и Воображаемого числа

 

Реальные числа — это такие числа, как:

1 12,38 −0,8625 3/4 √2 1998

Почти любое число, которое вы можете придумать, является действительным числом!

Мнимые числа, когда в квадрате дают отрицательный результат .

Обычно этого не происходит, потому что:

  • когда мы возводим в квадрат положительное число, мы получаем положительный результат, а
  • , когда мы возводим в квадрат отрицательное число, мы также получаем положительный результат (поскольку отрицательное число, умноженное на отрицательное, дает положительный результат), например, −2 × −2 = +4

Но представьте, что такие числа существуют, потому что они нам нужны.

Поговорим еще о мнимых числах…

«Единичное» мнимое число (например, 1 для действительных чисел) равно i, которое является квадратным корнем из −1

Потому что, возведя i в квадрат, мы получим −1

i 2 = −1

Примеры мнимых чисел Номера:

3i 1.04i −2,8i 3i/4 (√2)я 1998i

И мы держим здесь маленькую букву «i», чтобы напомнить себе, что нам нужно умножить на √−1

Комплексные числа

Когда мы объединяем действительное число и мнимое число, мы получаем комплексное число :

.

Примеры:

1 + я 39 + 3i 0,8 − 2,2i −2 + πi √2 + i/2

 

Может ли число быть комбинацией двух чисел?

Можем ли мы составить число из двух других чисел? Мы можем точно!

Мы постоянно делаем это с дробями. Дробь 3 / 8 — это число, состоящее из 3 и 8. Мы знаем, что это означает «3 из 8 равных частей».

Комплексное число — это всего лишь два числа, сложенные вместе (действительное и мнимое число).

Любая часть может быть равна нулю

Итак, у комплексного числа есть действительная и мнимая части.

Но любая часть может быть 0 , поэтому все действительные числа и мнимые числа также являются комплексными числами.

Комплексный номер Реальная часть Воображаемая часть  
3 + 2i 3 2  
5 5 0 Чисто настоящий
−6i 0 −6 Чисто воображаемый

Сложно?

Комплекс , а не означает сложный.

Это означает, что два типа чисел, действительные и мнимые, вместе образуют комплекс , точно так же, как комплекс зданий (здания, соединенные вместе).

Визуальное объяснение

Вы знаете, как идет числовая линия влево-вправо ?

Хорошо, пусть мнимые числа идут вверх-вниз :

И получаем Сложный Самолет

Комплексное число теперь может отображаться в виде точки:


Комплекс № 3+4 i

Добавление

Чтобы сложить два комплексных числа, складываем каждую часть отдельно:

(а+б я ) + (с+г я ) = (а+с) + (б+г) я

Пример: добавьте комплексные числа

3 + 2 i и 1 + 7 i
  • добавьте действительные числа и
  • добавить мнимые числа:

(3 + 2i) + (1 + 7i)
= 3 + 1 + (2 + 7) i
= 4 + 9i

Попробуем еще:

Пример: добавьте комплексные числа

3 + 5 i и 4 − 3 i

(3 + 5 i ) + (4 − 3 i )
= 3 + 4 + (5 − 3) i
90 + 3 2 0

На комплексной плоскости это:

Умножение

Чтобы умножить комплексные числа:

Каждая часть первого комплексного числа умножается на
каждая часть второго комплексного числа

Просто используйте «FOIL», что означает » F первоначальных, O маточных, I внутренних, L астровых» (подробнее см. Биномиальное умножение):

  • Первые: a × c
  • Внешний: a × d i
  • Внутренние: b i × c
  • Колодки: b i × d i

(a+b i )(c+d i ) = ac + ad i + bc i

3 id

0 + bc 005 2

Вот так:

Пример: (3 + 2i)(1 + 7i)

(3 + 2i)(1 + 7i) = 3×1 + 3×7i + 2i×1+ 2i×7i

 = 3 + 21i + 2i + 14i 2

= 3 + 21i + 2i − 14 (поскольку i 2 = −1)

 = −11 + 23i

А это:

Пример: (1 + i)

2

(1 + i)(1 + i)= 1×1 + 1×i + 1×i + i 2

 = 1 + 2i − 1  (потому что i 2 = −1)

 = 0 + 2i

Но есть более быстрый способ!

Используйте это правило:

(a+b i )(c+d i ) = (ac−bd) + (ad+bc) i

Пример: (3 + 2i)(1 + 7i) = (3×1 − 2×7) + (3×7 + 2×1)i = −11 + 23i

Почему это правило работает?

Это просто метод «ФОЛЬГА» после небольшой работы:

(a+b i )(c+d i ) =ac + ad i + bc i + bd i 2  Фольговый метод

6 i 0 = 0ac + 1 ad 02 + до н. э. i − bd   (потому что i 2 = −1)

 = (ac − bd) + (ad + bc) i   (собирая подобные термины)

И здесь у нас есть (ac − bd) + (ad + bc) i  шаблон.

Это правило, безусловно, быстрее, но если вы его забудете, просто запомните метод FOIL.

Попробуем i

2

Ради интереса воспользуемся методом вычисления i 2

Пример: i

2

Мы можем записать i с действительной и мнимой частями как 0 + i

i 2 = (0 + i) 2

 = (0 + i)(0 + i )

 = (0×0 − 1×1) + (0×1 + 1×0) i

 = −1 + 0 i

 = −1

И это хорошо согласуется с определением, что я 2 = −1

Так все замечательно работает!

Дополнительные сведения см. в разделе Умножение комплексных чисел.

Конъюгаты

Через минуту нам нужно будет узнать о конъюгатах!

В сопряжении мы меняем знак в середине следующим образом:

Спряжение часто пишется с чертой над ним:

Пример:

5 − 3 i   =   5 + 3 i

Разделение

Конъюгат используется для облегчения сложного деления.

Хитрость заключается в том, чтобы умножить верхнее и нижнее на сопряженное нижнее .

Пример: выполните следующее деление:

2 + 3 i 4 − 5 i

Умножьте верхнее и нижнее число на сопряженное число 4 − 5

9 6

0 45 2 + 3 я 4 − 5 i × 4 + 5 i 4 + 5 i   =   8 + 10 i + 12 i + 15 i 2 16 + 20 i я − 25 я 2

Теперь вспомним, что i 2 = −1, поэтому:

= 8 + 10 i + 12 i − 15 16 + 20 90 090 111 900 10 + 25

Добавьте условия «Нравится» (и обратите внимание, как внизу 20 i − 20 i отменяется!):

=   −7 + 22 i 41

. 9 41 + 22 41 i

ГОТОВО!

Да, нужно немного посчитать. Но это можно сделать.

Умножение на сопряженное

Однако есть более быстрый способ.

В предыдущем примере интересно было то, что произошло внизу:

(4 — 5 i )(4 + 5 i ) = 16 + 20 i — 20 i — 25 i 2

Средние члены (20 i − 20 i ) сокращаются:

(4 — 5 i )(4 + 5 i ) = 16 — 25 i 2

Также i 2 = −1 :

(4 — 5 i )(4 + 5 i ) = 16 + 25

А 16 и 25 — это (магически) квадраты 4 и 5:

(4 — 5 i )(4 + 5 i ) = 4 2 + 5 2

Довольно простой результат. Общее правило:

(a + b i ) (a − b i ) = a 2 + b 2

Это может сэкономить нам время при делении, например:

Пример: попробуем еще раз

2 + 3 i 4 − 5 i

Умножить верх и низ на сопряженное число 4 − 5 i :

2 + 3 i 4 − 5 i × 4 + 5 i 90 90 90 0 + 5 552   =   8 + 10 i + 12 i + 15 i 2 16 + 25

=   −7 + 22 i + 41 90 затем обратно в b i форма:

=   − 7 41 + 22 41 я

ГОТОВО!

 

Обозначение

Мы часто используем z для комплексного числа. И Re() для действительной части и Im() для мнимой части, например:

Что выглядит на комплексной плоскости так:

 

Набор Мандельброта

Прекрасное множество Мандельброта (на фото) основано на комплексных числах.

Это график того, что происходит, когда мы берем простое уравнение z 2 + c (оба комплексных числа) и возвращаем результат обратно в z снова и снова.

Цвет показывает, как быстро растет z 2 + c , а черный означает, что он остается в определенном диапазоне.

Вот изображение, полученное путем увеличения множества Мандельброта

А вот центр предыдущего увеличен еще больше:

 

440, 1070, 273, 1071, 1072, 443, 3991, 271, 3992, 3993

Комплексное число — определение, формула, свойства, примеры

Комплексные числа помогают найти квадратный корень из отрицательных чисел. Концепция комплексных чисел была впервые упомянута в I веке греческим математиком Героем Александрийским, когда он пытался найти квадратный корень из отрицательного числа. Но он просто изменил отрицательное значение на положительное и просто взял числовой корень. Кроме того, реальная идентичность комплексного числа была определена в 16 веке итальянским математиком Джероламо Кардано в процессе нахождения отрицательных корней кубических и квадратичных полиномиальных выражений.

Комплексные числа находят применение во многих научных исследованиях, обработке сигналов, электромагнетизме, гидродинамике, квантовой механике и анализе вибрации. Здесь мы можем понять определение, терминологию, визуализацию комплексных чисел, свойства и операции с комплексными числами.

1. Что такое комплексные числа?
2. График комплексных чисел
3. Свойства комплексных чисел
4. Операции над комплексными числами
5. Алгебраические тождества комплексных чисел
6. Решенные примеры
7. Практические вопросы
8. Часто задаваемые вопросы о комплексных числах

Что такое комплексные числа?

Комплексное число – это сумма действительного числа и мнимого числа. Комплексное число имеет вид a + ib и обычно обозначается буквой z. Здесь и a, и b – действительные числа. Величина «а» называется действительной частью, которая обозначается Re(z), а «b» называется мнимой частью Im(z). Также ib называют мнимым числом.

 

 

 

 

Примерами комплексных чисел являются \(2+3i, -2-5i, \,\,\dfrac 1 2 + i\dfrac 3 2\) и т. д.

Степень i

Алфавит i называется йотой и полезен для представления мнимой части комплексного числа. Кроме того, йота (i) очень полезна для нахождения квадратного корня из отрицательных чисел. У нас есть значение i 2  = -1, и оно используется для нахождения значения √-4 = √i 2 4 = + 2i Значение i 2  = -1 является основным аспектом комплексного числа. Давайте попробуем понять больше о возрастающих силах i.

  • я = √-1
  • i 2  = -1
  • i  = i.i 2  = i(-1) = -i
  • i 4  = (i 2 ) 2  = (-1) 2  = 1
  • i 4n  = 1
  • я 4n + 1  = я
  • i 4n + 2  = -1
  • i 4n + 3  = -i

График комплексных чисел

Комплексное число состоит из действительной и мнимой частей, которые можно рассматривать как упорядоченную пару (Re(z), Im(z)) и представлять в виде точек координат на евклидовой плоскости. Евклидова плоскость применительно к комплексным числам называется комплексной плоскостью или Плоскостью Аргана, названной в честь Жана-Роберта Аргана. Комплексное число z = a + ib представлено действительной частью — a относительно оси x и мнимой частью -ib относительно оси y. Давайте попробуем понять два важных термина, относящихся к представлению комплексных чисел на аргановой плоскости. Модуль и аргумент комплексного числа. 92}\)|. Кроме того, это можно понимать как полученное из теоремы Пифагора, где модуль представляет собой гипотенузу, действительную часть — это основание, а мнимую часть — высоту прямоугольного треугольника.

Аргумент комплексного числа

Угол, образованный линией, соединяющей геометрическое представление комплексного числа и начало координат с положительной осью X в направлении против часовой стрелки, называется аргументом комплексного числа. Аргумент комплексного числа является обратным отношением тангенса мнимой части к действительной части комплексного числа. {-1}\frac{b}{a}\). 9{-1}\frac{b}{a}\)).

Свойства комплексного номера

Следующие свойства комплексных чисел помогают лучше понять комплексные числа, а также выполнять различные арифметические операции над комплексными числами.

Сопряжение комплексного числа

Сопряжение комплексного числа образуется путем взятия той же действительной части комплексного числа и замены мнимой части комплексного числа на ее аддитивную обратную. Если сумма и произведение двух комплексных чисел являются действительными числами, то они называются сопряженными комплексными числами. Для комплексного числа z = a + ib его сопряженным является \(\bar z\) = a — ib.

Сумма комплексного числа и его сопряженного равна \(z + \bar z\)  = (a + ib) + (a — ib) = 2a, а произведение этих комплексных чисел \(z.\bar z \) = (a + ib) × (a — ib) = a 2  + b 2 .

Обратная величина комплексного числа

Обратная величина комплексных чисел полезна в процессе деления одного комплексного числа на другое комплексное число. {-1}\).

Равенство комплексных чисел

Равенство комплексных чисел аналогично равенству действительных чисел. Два комплексных числа \(z_1 = a_1 + ib_1\) и \(z_2 = a_2 + ib_2 \) называются равными, если относительная часть обоих комплексных чисел равна \(a_1 = a_2\),  и мнимая части обоих комплексных чисел равны \(b_1 = b_2 \). Кроме того, два комплексных числа в полярной форме равны тогда и только тогда, когда они имеют одинаковую величину, а их аргумент (угол) отличается на целое кратное 2π.

Упорядочивание комплексных чисел

Упорядочивание комплексных чисел невозможно. Действительные числа и другие связанные системы счисления можно упорядочить, но нельзя упорядочить комплексные числа. Комплексные числа не имеют структуры упорядоченного поля, и нет упорядоченности комплексных чисел, совместимой со сложением и умножением. Также нетривиальная сумма квадратов в упорядоченном поле есть число \(\neq 0\), а в комплексном числе нетривиальная сумма квадратов равна i 2  + 1 2  = 0. Комплексные числа можно измерить и представить на двумерной арграндовой плоскости по их величине, которая является расстоянием от начала координат.

Формула Эйлера: В соответствии с формулой Эйлера для любого действительного значения θ мы имеем e  = Cosθ + iSinθ, и оно представляет комплексное число в координатной плоскости, где Cosθ – действительная часть, представленная относительно ось x, Sinθ – мнимая часть, представленная относительно оси y, θ – угол, образованный по отношению к оси x и воображаемой линии, соединяющей начало координат и комплексное число. Согласно формуле Эйлера и функциональному представлению x и y имеем e x + iy  = e x (уютно + isiny) = e x уютно + т.е. x сине. Это разлагает экспоненциальную функцию на ее действительную и мнимую части.

Операции над комплексными числами

Различные операции сложения, вычитания, умножения, деления натуральных чисел можно выполнять и для комплексных чисел. Детали различных арифметических операций с комплексными числами заключаются в следующем.

Сложение комплексных чисел

Сложение комплексных чисел аналогично сложению натуральных чисел. Здесь в комплексных числах действительная часть добавляется к действительной части, а мнимая часть добавляется к мнимой части. Для двух комплексных чисел вида \(z_1 = a + id\) и \(z_2 = c + id\) сумма комплексных чисел \(z_1 + z_2 = (a + c) + i(b + d) \). Комплексные числа следуют всем следующим свойствам сложения.

  • Закон замыкания: Сумма двух комплексных чисел также является комплексным числом. Для двух комплексных чисел \(z_1\) и \(z_2\) сумма \(z_1 + z_2\) также является комплексным числом.
  • Коммутативный закон: Для двух комплексных чисел \(z_1\), \(z_2\) имеем \(z_1 + z_2 = z_2 + z_1\).
  • Ассоциативный закон: Для данных трех комплексных чисел \(z_1, z_2, z_3\) имеем \(z_1 + (z_2 + z_3) = (z_1 + z_2)+z_3 \). 2 = -1\). Для двух комплексных чисел \(z_1\) = a + ib, \(z_2\) = c + id произведение равно \(z_1.z_2\) = (ca — bd) + i(ad + bc).

    Умножение комплексных чисел в полярной форме немного отличается от упомянутой выше формы умножения. Здесь абсолютные значения двух комплексных чисел перемножаются, а их аргументы складываются для получения произведения комплексных чисел. Для комплексных чисел \(z_1 = r_1(Cos\theta_1 + iSin\theta_1)\) и  z 2  = \(z_2 = r_1(Cos\theta_2 + iSin\theta_2)\) произведение комплексные числа \(z_1.z_2 = r_1.r_2(Cos(\theta_1 + \theta_2) + iSin(\theta_1 + \theta_2))\). 92 + 2z_1z_2 +2z_2z_3 +2z_3z_1\)

Связанные темы:

  • Комплексное сопряжение
  • Калькулятор комплексных чисел
  • Тригонометрия
  • Координатная плоскость
  • Координатная геометрия

Комплексные числа Советы и подсказки:

  • Все действительные числа являются комплексными числами, но не все комплексные числа должны быть действительными числами.
  • Все мнимые числа являются комплексными числами, но все комплексные числа не обязательно должны быть мнимыми числами. 9{2}-4(1)(1)}}{2(1)} \\[0,2 см]
    &=\frac{-1 \pm \sqrt{-3}}{2}\\[0,2 см]
    \text{Здесь } &\sqrt{-3} = \sqrt{-1} \times \sqrt{3} = i \sqrt{3}\\[0,2 см]
    x&= \frac{-1 \pm i\sqrt{3}}{2}\\[0,2 см]
    \end{align} \]

    Таким образом, корнями данного квадратного уравнения являются: \(\frac{-1}{2}+ i\frac{\sqrt{3}}{2};\,\,\ , \ frac{-1}{2}- i\frac{\sqrt{3}}{2}\)

  • Пример 2: Выразите сумму, разность, произведение и частное следующих комплексных чисел в виде комплексного числа.

    \[\begin{align} z_1&=-2+i\\[0.2cm]z_2&= 1-2i \end{align} \]

    Решение:

    Сумма:

    \[ \begin{ выровнять} z_1+z_2&= (-2+i)+(1-2i)\\[0,2 см] &=(-2+1)+ (i-2i)\\[0,2 см] &= -1-i \end{align}\]

    Разница:

    \[ \begin{align} z_1-z_2&= (-2+i)-(1-2i)\\[0,2 см] &=(-2-1) + (i+2i)\\[0,2 см] &= -3+3i \end{align}\]

    Продукт:

    \[ \begin{align} z_1\cdot z_2&= (-2+i)( 1-2i)\\[0,2см] &=-2+4i+i-2i^2\\[0,2см] &=-2+4i+i+2 \,\,\, [\потому что i^2 =-1]\\[0,2 см] &=5i \end{выравнивание}\] 92=-1]\\[0,2 см] &= \dfrac{-4-3i}{5}\\[0,2 см] &=- \dfrac{4}{5}- i \dfrac{3}{5 }\end{align}\]

    Следовательно, имеем:

    Сумма = -1 — i
    Разница = -3 + 3i
    Продукт = 5i
    Деление = -4/5 — 3i/5

  • перейти к слайдуперейти к слайду

    Разбивайте сложные концепции с помощью простых визуальных средств.

    Математика больше не будет сложным предметом, особенно когда вы понимаете концепции с помощью визуализаций.

    Записаться на бесплатный пробный урок

    перейти к слайдуперейти к слайду

     

    Часто задаваемые вопросы о комплексных числах

    Что такое комплексные числа в математике?

    Комплексное число представляет собой комбинацию действительных и мнимых значений. Обозначается z = a + ib, где a, b — действительные числа, а i — мнимое число. i = \(\sqrt{-1}\) и никакое действительное значение не удовлетворяет уравнению i 2  = -1, поэтому I называется мнимым числом.

    Для чего используются комплексные числа?

    Комплексное число используется для простого нахождения квадратного корня из отрицательного числа. Здесь мы используем значение i 2  = -1 для представления отрицательного знака числа, что помогает легко найти квадратный корень. Здесь мы имеем √-4 = √i 2 4 = + 2i. {-1}\frac{b}{a} \)).

    Что такое действительные и комплексные числа?

    Комплексные числа являются частью действительных чисел. Некоторые действительные числа с отрицательным знаком трудно вычислить, и мы представляем отрицательный знак с помощью йоты «i», и такое представление чисел вместе с «i» называется комплексным числом. Дополнительные комплексные числа полезны для нахождения квадратного корня из отрицательного числа, а также для нахождения отрицательных корней квадратного или полиномиального выражения.

    Как делить комплексные числа? 92)}\).

    Как строить графики комплексных чисел?

    Комплексное число вида z = a + ib может быть представлено в плоскости арганда. Комплексное число z = a + ib может быть представлено в виде координат точки как (Re(z), Im(z)) = (a, ib). Здесь действительная часть представлена ​​относительно оси x, а мнимая часть представлена ​​относительно оси y.

    Как преобразовать комплексные числа в полярную форму?

    Комплексный номер можно легко преобразовать в полярную форму.

Таблица умножения на 5 и 6: Карточка «Таблица умножения и деления на 5 и 6».

Тест Таблица умножения на 6 по математике

Последний раз тест пройден более 24 часов назад.

Для учителя

Материал подготовлен совместно с учителем высшей категории Харитоненко Натальей Владимировной.

Опыт работы учителем математики — более 33 лет.

  1. Вопрос 1 из 10

    6 х 8

    • 12

    • 14

    • 48

    Подсказка

    Правильный ответ

    Неправильный ответ

    В вопросе ошибка?

  2. Вопрос 2 из 10

    6 х 4

    • 24

    • 10

    • 2

    Подсказка

    Правильный ответ

    Неправильный ответ

    В вопросе ошибка?

  3. Вопрос 3 из 10

    6 х 9

    • 15

    • 54

    • 48

    Подсказка

    Правильный ответ

    Неправильный ответ

    В вопросе ошибка?

  4. Вопрос 4 из 10

    6 х 0

    • 6

    • 0

    • 2

    Подсказка

    Правильный ответ

    Неправильный ответ

    В вопросе ошибка?

  5. Вопрос 5 из 10

    6 х 6

    • 12

    • 10

    • 36

    Подсказка

    Правильный ответ

    Неправильный ответ

    В вопросе ошибка?

  6. Вопрос 6 из 10

    6 х 1

    • 6

    • 7

    • 5

    Подсказка

    Правильный ответ

    Неправильный ответ

    В вопросе ошибка?

  7. Вопрос 7 из 10

    6 х 3

    • 9

    • 3

    • 18

    Подсказка

    Правильный ответ

    Неправильный ответ

    В вопросе ошибка?

  8. Вопрос 8 из 10

    6 х 5

    • 30

    • 11

    • 1

    Подсказка

    Правильный ответ

    Неправильный ответ

    В вопросе ошибка?

  9. Вопрос 9 из 10

    6 х 7

    • 13

    • 42

    • 1

    Подсказка

    Правильный ответ

    Неправильный ответ

    В вопросе ошибка?

  10. Вопрос 10 из 10

    6 х 2

    • 12

    • 4

    • 8

    Подсказка

    Правильный ответ

    Неправильный ответ

    В вопросе ошибка?

Доска почёта

Чтобы попасть сюда — пройдите тест.

Тест «Таблица умножения на 6» по математике рассчитан на учеников начальных классов. С его помощью можно легко систематизировать и закрепить материал по теме. Это удобный способ разнообразить изучение простейших примеров на деление. Если тест не удалось пройти с первого раза, нужно доучить материал и пройти его еще раз. Задания доступны в онлайн режиме, просматривать их можно с любого доступного устройства.

Рейтинг теста

4.8

Средняя оценка: 4.8

Всего получено оценок: 869.


А какую оценку получите вы? Чтобы узнать — пройдите тест.

Карточки для проверки таблицы умножения | Тренажёр по математике (3 класс):

Опубликовано 27.02.2022 — 12:52 — Александрова Саяна Сергеевна

Данные карточки помогут учителю проверить таблицу умножения и деления.

Скачать:


Предварительный просмотр:

Имя

3*8=

4*7=

3*5=

6*2=

5*6=

8*8=

2*3=

5*8=

7:1=

8*4=

18:2=

4*9=

9:3=

3*9=

24:4=

7*9=

28:4=

8*9=

40:5=

9*8=

36:4=

9*9=

27:9=

7*2=

30:5=

9*2=

16:8=

2*5=

36:9=

6*3=

54:9=

6*8=

70:10=

5*8=

42:7=

6*4=

64:8=

4*3=

36:4=

5*6=

Имя

_____________

2*7=

30:10=

9*3=

30:5=

9*5=

24:4=

4*5=

36:9=

5*3=

21:3=

2*6=

45:9=

2*6=

24:3=

7*5=

21:7=

3*4=

18:2=

6*6=

16:2=

8*5=

14:2=

7*7=

45:9=

8*5=

63:9=

9*7=

54:9=

3*9=

18:2=

1*8=

28:7=

6*5=

70:7=

4*8=

36:9=

8*9=

48:8=

9*7=

Имя

_____________

3*7=

50:10=

7*3=

35:5=

8*5=

24:6=

5*5=

36:6=

4*3=

24:3=

3*6=

36:9=

4*6=

27:3=

8*5=

28:7=

4*4=

20:2=

7*6=

18:2=

8*5=

16:2=

4*7=

54:9=

7*5=

72:9=

8*7=

81:9=

3*9=

14:2=

3*8=

56:7=

7*5=

80:8=

6*8=

63:9=

3*9=

56:8=

8*7=

Имя

_____________

4*7=

60:10=

8*3=

40:5=

7*5=

28:4=

4*5=

45:9=

5*3=

30:3=

4*6=

81:9=

2*6=

24:3=

7*5=

21:7=

5*4=

12:2=

6*6=

16:2=

7*5=

18:2=

6*7=

45:9=

8*5=

81:9=

4*7=

18:9=

4*9=

10:2=

1*9=

49:7=

9*5=

70:7=

3*8=

27:9=

3*9=

40:8=

4*7=

Имя

_____________

5*7=

40:10=

9*3=

45:5=

9*5=

36:4=

2*5=

36:9=

9*3=

27:3=

2*6=

54:9=

8*6=

15:3=

5*5=

35:7=

3*4=

8:2=

4*6=

14:2=

6*5=

14:2=

9*7=

36:9=

8*7=

63:9=

2*7=

72:9=

6*9=

12:2=

1*4=

35:7=

6*5=

80:8=

4*8=

9:9=

8*9=

32:8=

2*7=


По теме: методические разработки, презентации и конспекты

Карточки для проверки таблицы умножения и деления на 2 3 4

индивидуальные карточки для проверки табличного умножения и деления. ..

Карточки для проверки таблицы умножения и деления на 2,3,4,5,6,7,8,9.

Можно использовать как на уроках, так и при отработке таблицы умножения дома. Предложенные карточки помогут учителю проверить у учащихся знания таблицы умножения.Цель: проверка знания учащимися т…

Индивидуальные карточки для проверки таблицы умножения

Карточки для индивидуальной работы по математике…

Карточки для проверки таблицы умножения.

Эти карточки использую на уроках математики  для проверки знания таблицы умножения….

Карточки для проверки таблицы умножения и деления

Таблица умножения и деления…

Карточки для проверки таблицы умножения

Данная табличка не имеет отношения к таблице Пифагора. В ней расположены числа в свободном порядке. Ученик решает данные ему примеры, находит значение произведения в табличке и закрашивает в соответст…

Карточки для проверки таблицы умножения. 2 класс

Карточки для проверки таблицы умножения. 2 класс…


Поделиться:

 

Math Tables of 6 — Изучите таблицу умножения для детей

  • Почему вашему ребенку нужно выучить таблицу 6?
  • Что такое таблица умножения на 6 в математике?
  • Таблица умножения 6
  • Таблица умножения на 6 для детей
  • Советы по изучению и запоминанию таблицы умножения 6 для детей
  • Простые вопросы, которые помогут детям повторить таблицу 6
  • Словесные задачи по таблице умножения на 6 для детей
  • Часто задаваемые вопросы

Все мы знаем, что таблица умножения является основой математики. Для маленьких детей изучение таблицы 6 по математике часто оказывается не очень удобной задачей. Большинство детей очень легко усваивают таблицы до пяти. Но борьба начинается, как только урок по таблице умножения 6 становится под вопросом. Большинству детей будет трудно решать задачи с большими числами в будущем. Если ученик не может вычислять большие числа в быстром темпе, то его изучение математики не продвинется далеко. Как родители или учителя, мы бы не хотели этого для наших детей. Заучивание таблицы умножения наизусть обеспечивает прочную арифметическую основу. Продолжайте читать статью, чтобы узнать о советах и ​​хитростях, стоящих за ней.

Зачем вашему ребенку нужно учить таблицу 6?

  1. Таблицы умножения составляют основу математических навыков.
  2. Таблица умножения на 6 — это тот момент, когда таблица умножения становится сложнее для ученика. Поэтому важно выучить его наизусть.
  3. Таблицы с 1 по 10 являются основами математики для юных учеников. Хорошее обучение по ним может помочь ученику расти в математике.
  4. Таблицы помогают нам решать сложные задачи на умножение и деление. Понимание сложных таблиц, начиная с 6, поможет ученикам в дальнейшем.
  5. Если ваш ребенок мечтает принять участие в конкурсных экзаменах, ему просто необходимо уметь быстро считать. Несколько столов, начиная с 6, помогут им в таких соревнованиях на время.

Что такое таблица умножения на 6 в математике?

Как и другие таблицы умножения, шестую таблицу можно правильно выучить, если начать ее рано. Вы можете объяснить юному ученику таблицу умножения на 6 с помощью схемы, приведенной ниже.

6 х 1 = 6 6
6 х 2 = 12 6+6 = 12
6 х 3 = 18 6+6+6 = 18
6 х 4 = 24 6+6+6+6 = 24
6 х 5 = 30 6+6+6+6+6 = 30
6 х 6 = 36 6+6+6+6+6+6 = 36
6 х 7 = 42 6+6+6+6+6+6+6 = 42
6 х 8 = 48 6+6+6+6+6+6+6+6 = 48
6 х 9 = 54 6+6+6+6+6+6+6+6+6 = 54
6 х 10 = 60 6+6+6+6+6+6+6+6+6+6 = 60
6 х 11 = 66 6+6+6+6+6+6+6+6+6+6+6 = 66
6 х 12 = 72 6+6+6+6+6+6+6+6+6+6+6+6 = 72

Таблица умножения 6

По мере того, как ребенок продвигается в учебе, ему необходимо выучить таблицу умножения от 6 до 20. Это поможет ему быстро выполнять вычисления.

6 х 1 = 6 6 х 11 = 66
6 х 2 = 12 6 х 12 = 72
6 х 3 = 18 6 х 13 = 78
6 х 4 = 24 6 х 14 = 84
6 х 5 = 30 6 х 15 = 90
6 х 6 = 36 6 х 16 = 96
6 х 7 = 42 6 х 17 = 102
6 х 8 = 48 6 х 18 = 108
6 х 9 = 54 6 х 19 = 114
6 х 10 = 60 6 х 20 = 120

Таблица умножения на 6 для детей

Сделайте распечатку таблицы или скопируйте ее на чистый лист бумаги и повесьте таблицу в месте, где ученик может ее часто видеть. Это поможет создать долговременный образ таблицы в их памяти, заставляя их быстрее выучить таблицу 6 от 6 х 1 до 6 х 12.

6 Табличная таблица до 10

Приведенная выше таблица предназначена для умножения 6 от 6 x 1 до 6 x 10, которую вы можете использовать для занятий с математикой вашего ребенка. Просто найдите способ, чтобы ваш ребенок неоднократно просматривал таблицу и был уверен в развитии своих математических навыков.

6 Таблица таблиц до 20

После того, как ваш ребенок выучил наизусть таблицу 6 от 6 x 1 до 6 x 10, пришло время подтолкнуть его дальше. Предложите им выучить таблицу умножения 6 от 6 х 11 до 6 х 20. Таблица, приведенная ниже, поможет им в этом.

Советы по изучению и запоминанию таблицы умножения на 6 для детей

Изучение таблицы умножения на 6 или любой другой таблицы умножения иногда может быть трудным. Всегда есть простой способ запомнить таблицы умножения на 6 или другие таблицы. Итак, вот трюк за столом и другие советы, которые помогут вашему ребенку учиться быстрее и лучше.

  •  Найти ответы с помощью сложения: Поскольку умножение — это более короткий способ сложения числа несколько раз, мы можем учить наших детей сложению за таблицей 6.

Например,

1. Дважды прибавив 6, мы получим значение 6, умноженное на 2, как показано ниже.

6 x 2 = 6 + 6 = 12.

2. Прибавляя 6 семь раз, мы получаем значение 6, умноженное на 7, как показано ниже.

6 x 7 = 6 + 6 + 6 + 6 + 6 + 6 + 6 = 42.

  • Использование таблицы умножения на 3: Большинству детей очень нравится таблица умножения 3. Число 6 в 2 раза больше числа 3. Вы можете использовать 3 таблицы, чтобы научить вашего ребенка таблице 6.

Например,

  1. Если вопрос 6 х 5, то узнайте, сколько будет 3 х 5 и умножьте ответ на 2.

Так как 3 х 5 = 15.

Итак, 6 х 5 = 2 х 15 = 30.

2. Если вопрос 6 х 4, то узнайте, сколько будет 3 х 4 и умножьте ответ на 2.

Поскольку 3 x 4 = 12.

Итак, 6 x 4 = 2 x 12 = 24.

  • Использование таблицы умножения на 5: Таблица 5 очень проста почти для каждого ребенка. Вы можете использовать таблицу умножения на 5, чтобы научить таблицу умножения на 6 с помощью трюка. Любая цифра, умноженная на 5, а затем та же самая цифра, добавленная к результату, всегда дает таблицу умножения на 6.

Например,

1. Когда вопрос 6 х 2 =?

Так как 5 х 2 = 10, 10 + 2 = 12.

Итак, 6 х 2 = 12.

2. Когда вопрос 6 х 5 =?

Так как, 5 x 5 = 25, 25 + 5 = 30.

Итак, 6 x 5 = 30.

  • Идентификация основного шаблона: В таблице 6 есть шаблон. Произведение заканчивается на 6, 2, 8, 4, 0 подряд.

Например,

6     x     1      =      6 912   =      30
6     x     6      =        36
6     x     7        =        42
6     x     8        =        48
6    х     9      =      54
6 x     10    =      60

  • Добавление 6 к предыдущему ответу: Вы всегда можете добавить число к предыдущему ответу, чтобы получить полную таблицу.

Например,

6 х 1 = 6

6 х 2 = 6 + 6 = 12

6 х 3 = 12 + 6 = 18

6 х 4 = 18 + 6 = 24 и так далее.

Простые вопросы, которые помогут детям повторить таблицу 6

Задайте эти простые вопросы своим детям, чтобы они выучили таблицу 6.

  • 6 x _ = 18
  • _ х 6 = 0
  • _ х _ = 30
  • Отметьте правильный ответ, 6 x 7 = _ (12, 24, 42, 54)
  • Обведите числа, кратные 6: 63, 24, 36, 81, 42, 18

Словесные задачи по таблице умножения на 6 для детей

Словесные задачи — отличный способ укрепить математические способности вашего ребенка. Это идеальный способ научить вашего ребенка выучить таблицу на 6. Вот некоторые задачи с использованием таблицы 6:

1. Каково произведение 6 и 8?

Решение: Произведение 6 и 8 означает

6 X 8 = 48

Таким образом, ответ равен 48.

2. Умножьте произведение 6 и 8 на 5.

Решение: Первый шаг — найти произведение 6 и 8

т.е. 6 x 8 = 48

На втором шаге умножить произведение на 6 и 8, то есть 48 с 5

48 X 5 = 240

Итак, ответ 240.

3. Чему равно произведение 6 и 11? 0 1.э. 6 x 11 = 66

Таким образом, ответ равен 66.

Часто задаваемые вопросы

1. Как была получена таблица 6?

Таблица 6 получается путем добавления 6 каждый раз. Вот как мы можем получить таблицу:

6
6 + 6 = 12
6 + 6 + 6 = 18
6 + 6 + 6 + 6 = 24
6 + 6 + 6 + 6 + 6 = 30, и скоро.

2. Как написать таблицу умножения на 6 словами?

Вы можете научить своего ребенка 6-й таблице умножения и в другом формате. Вы можете написать таблицу для шести словами, как указано ниже.

6 раз 1 равно 6
6 раз 2 равно 12
6 раз 3 равно 18
6 умножить на 4 равно 24
6 умножить на 5 равно 30 и так далее.

Важно, чтобы у детей были таблицы с однозначными цифрами в подсказках, чтобы иметь прочную математическую подготовку, и это поможет детям в вычислениях и решении сложных задач.

Читайте также:

Изучите математическую таблицу 7 для детей
8 Таблица умножения с диаграммой для детей
Таблица умножения 9 для детей, чтобы выучить

Как создать таблицу умножения для любого числа в Python

Goodness Chidinma Abarugo

Grokking the Behavioral Interview

Многим кандидатам отказывают или понижают их уровень на технических собеседованиях из-за плохой успеваемости на собеседованиях поведенческого или культурного соответствия. Пройдите собеседование с помощью этого бесплатного курса, где вы будете практиковаться, уверенно отвечая на поведенческие вопросы интервью.

Обзор

В Python мы можем создать таблицу умножения для любого числа, комбинируя input() и range() функции с оператором цикла.

Функция

input()

Функция input() используется для приема ввода от пользователя. Если требуемый тип данных явно не определен, любое значение, введенное пользователем в приглашении, сохраняется в памяти в виде строки.

Синтаксис

 ввод (подсказка)
 

Параметр

  • приглашение : Строка, заключенная в одинарные или двойные кавычки. Его наличие делает наш код более интерактивным. Это необязательный параметр.

Функция

range()

Функция range() позволяет нам автоматически создавать серию чисел. Мы перебираем серию, выполняя некоторые строки кода, пока не придем к последнему числу в серии.

Синтаксис

 диапазон(старт, стоп, шаг)
 

Параметры

  • начало : первое число в серии. Это необязательный параметр.

  • стоп : Последний номер в серии. Это обязательный параметр.

  • шаг : размер шага, на который мы хотим увеличить или уменьшить нашу серию. Это необязательный параметр.

Примечание. Если указан только один параметр, этот параметр классифицируется как параметр стоп , а параметры пуск и шаг принимаются как 0 и 1 соответственно.

Циклы

Циклы полезны, когда мы хотим выполнить строку или блок кода более одного раза, при условии выполнения условия или до тех пор, пока не будет достигнуто последнее значение в серии.

Мы будем использовать для и , пока зацикливаются в этой задаче. Существует небольшая разница между синтаксисом цикла для и в то время как . Цикл for работает в пределах диапазона, а цикл while работает только при выполнении одного или нескольких условий.

Синтаксис

 # для цикла
для х последовательно:
   Сделай что-нибудь
# пока цикл
пока выполняется условие:
    Сделай что-нибудь
 

Создайте таблицу умножения

Мы создадим нашу таблицу умножения на основе приведенной ниже блок-схемы:

Создание таблицы умножения

Наша вышеприведенная блок-схема преобразуется в следующий алгоритм:

  1. Запустите программу.
  2. Получить целочисленное значение от пользователя.
  3. Далее мы определим диапазон или условие. Диапазон будет использоваться в цикле для , а условие будет использоваться в цикле while .
  4. Наконец, код проверит наш предмет или состояние. Код в цикле будет продолжать выполняться до тех пор, пока элементы не окажутся за пределами диапазона или не будут выполнены условия.

Код с использованием цикла

на

Мы будем использовать следующий код для создания таблицы умножения с использованием цикла на .

 

ourNum = int(input("Введите число, для которого вы хотите создать таблицу умножения, затем нажмите клавишу `enter`: "))

ourRange = range(1,6)

для x в ourRange:

result = ourNum * x

print(ourNum," * ",x," = ",result)

Таблица умножения с использованием цикла for

  • В строке 1 запрашиваем номер у пользователя. Для этого числа будет создана таблица умножения. Мы преобразуем число в целочисленный тип данных, заключая наши input() в int() .

  • В строке 2 мы определяем ourRange , который включает числа от 1 до 5.

  • Далее, в строке 3, мы запускаем наш цикл для и определяем x как переменную для хранения элементов, хранящихся в переменной ourRange .

  • В строке 4 мы используем переменную результата для хранения значения произведения заданного пользователем числа и текущего элемента в диапазоне.

  • В строке 5 мы отображаем заданное пользователем число, знак умножения, текущий элемент в ряду, знак равенства и значение, хранящееся в переменной результата на каждой итерации.

  • Вывод

     Введите число, для которого вы хотите создать таблицу умножения, затем нажмите клавишу ввода:7
    7 * 1 = 7
    7 * 2 = 14
    7 * 3 = 21
    7 * 4 = 28
    7 * 5 = 35
     

    Примечание. Во время выполнения кода от пользователя требуется целое число. Поэтому мы введем 7 в качестве числа, для которого мы хотим создать таблицу умножения. В нашем выводе цикл завершается после умножения на 5 вместо 6 . Это связано с тем, что последний элемент в диапазоне не используется во время выполнения кода.

    Код с использованием цикла

    while

    Мы будем использовать приведенный ниже код для создания таблицы умножения с использованием цикла while .

     

    ourNum = int(input("Введите число, для которого вы хотите создать таблицу умножения, затем нажмите клавишу `enter`: "))

    p = 1

    пока p < 6:

    result = ourNum * p

    print(ourNum, " * ", p," = ",result)

    p = p + 1

    Таблица умножения используя цикл while

    Объяснение

    В отличие от цикла for , нашему циклу while для выполнения итерации требуется счетчик.

    • В строке 1 мы запрашиваем у пользователя целое число.

    • В строке 2 мы устанавливаем p в качестве переменной, которая содержит наш счетчик, и устанавливаем его начальное значение равным 9.0421 1 .

    • В строке 3 мы указываем, что блок кода в нашем цикле while будет выполняться только до тех пор, пока p меньше 6 .

    • В строке 4 мы получаем произведение введенного числа на текущее число в ряду.

    Найти вероятность онлайн калькулятор: Калькуляторы по теории вероятностей. Найти вероятность онлайн легко!

    Калькуляторы по теории вероятностей. Найти вероятность онлайн легко!

    Кто сейчас не пользуется калькуляторами? Да только не обычными, а онлайн-версиями. Дошел прогресс и до теории вероятностей, и к вашим услугам несколько бесплатных калькуляторов, позволяющих решить стандартные задачи теории вероятностей. Выбирайте нужный из списка и переходите к решению.

    Практические все калькуляторы снабжены подробной теорией, помогающей решить данный тип задач и примерами, которые помогут разобраться в теме. Помните, что калькулятор — прежде всего помощь в числовых расчетах, а понимание сути задачи и решения — важнее (и это вы тоже найдете у нас).

    Найти вероятность онлайн? Без проблем!

    Комбинаторика

    • Вычисление числа перестановок онлайн
    • Вычисление числа размещений онлайн
    • Вычисление числа сочетаний онлайн

    • Вычисление числа перестановок с повторениями онлайн
    • Вычисление числа размещений с повторениями онлайн
    • Вычисление числа сочетаний с повторениями онлайн

    Подробнее о комбинаторных калькуляторах

    Классическая вероятность

    • Гипергеометрическая вероятность. Общая формула, вывод (доказательство), и разбор с подробными примерами и калькуляторами следующих типов задач:
      • Задача про шары (в урне находится $k$ белых и $n$ черных шаров, вынимают $m$ шаров…)
      • Задача про детали (в ящике находится $k$ стандартных и $n$ бракованных деталей, вынимают $m$ деталей…)
      • Задача про лотерейные билеты (в лотерее участвуют $k$ выигрышных и $n$ безвыигрышных билета, куплено $m$ билетов…)

    Схема независимых испытаний Бернулли

    • Формула Бернулли. Общая формула, разбор с подробными примерами и калькуляторами следующих типов задач:
      • Задача про партии в шахматы
      • Задача про выстрелы
      • Задача про мальчиков и девочек
      • Задача про лотерейные билеты
      • Задача о наивероятнейшем значении
      • Формула Пуассона

    Сложение и умножение вероятностей

    • Задача про стрелков и выстрелы по мишени. Теория, примеры решенных и калькулятор типовой задачи для 2 и 3 стрелков.

    Дискретная случайная величина

    • Как найти математическое ожидание случайной величины?
    • Как найти дисперсию случайной величины?
    • Вычисление математического ожидания, дисперсии и среднего квадратического отклонения дискретной случайной величины, заданной рядом распределения. Основные формулы и автоматический расчет для ваших данных.

    Закажите решение задач по теории вероятности

       Узнать цену сегодня

    Калькуляторы не подошли? Используй решебник!

    Более 11000 задач с полными и подробными решениями по теории вероятностей и математической статистике.

    Изучаем теорию вероятностей

    • Тервер для чайников — онлайн учебник
    • Скачать учебники по теории вероятности
    • Формулы и таблицы по теории вероятности
    • Решенные контрольные по теории вероятности
    • Как найти вероятность в задачах про подбрасывания монеты?
    • Как найти вероятность в задачах про подбрасывание игральных костей?

    Расчет Вероятности — как найти вероятность

    расчет вероятности помогает рассчитать вероятность для одного события, нескольких событий, двух событий, для серии событий, а также событий с условной вероятностью. Если вы хотите рассчитать вероятность a и b и для любого количества событий, то приведенный выше калькулятор вероятностей подойдет вам лучше всего!

    Что ж, переходим к делу; просто прочтите этот пост, чтобы узнать, как рассчитать вероятность, различные уравнения вероятности, все формулы вероятности, статистический калькулятор вероятности и многое другое, что вам нужно знать о вероятности.

    Итак, давайте начнем с наилучшего определения вероятности!

    Что такое вероятность в статистике?

    Под вероятностью понимается вероятность наступления события или нескольких событий. Вероятность – это то, что указывает на возможность достижения определенного результата и может быть рассчитано с помощью простой формулы вероятности.

    Происхождение теории вероятностей начинается с изучения таких игр, как игра в кости, подбрасывание монет, карт и т. Д. Но в настоящее время вероятность имеет большое значение при принятии решений. Классическая теория показывает, что вероятность – это отношение благоприятного случая к общему количеству равновероятных случаев. Субъективный подход показывает, что вероятность события определяется человеком на основе имеющихся у него / нее свидетельств.

    Исследование о вероятности:

    Идея вероятности как полезной науки принадлежит известным французским математикам Блезу Паскалю и Пьеру де Ферма.

    Согласно «Исчислению, том II» Тома М. Апостола, и Блез Паскаль, и Пьер де Ферма решали проблему с азартными играми в 1954 году. Они лучше всего работают при определении количества ходов, необходимых для получения 6 при броске двух кубиков. Да, дискуссии Паскаля и де Ферма заложили основу концепции теории вероятностей.

    Какова формула вероятности?

    Формула вероятности события следующая:

    P (A) = количество благоприятных исходов / общее количество благоприятных исходов

    Или формула вероятности:

    P (A) = n (E) / n (S)

    Где,

    • P (A) называется вероятностью события «A»
    • n (E) называется числом благоприятных исходов.
    • n (S) называется числом событий в выборке

    Примечание. Здесь благоприятный исход указывается как интересующий результат.

    Теперь давайте посмотрим на основные формулы вероятности!

    Каковы основные формулы вероятности?

    Проведите вниз!

    Диапазон вероятности:

    0 ≤ P (A) ≤ 1

    Правило сложения:

    P (A∪B) = P (A) + P (B) – P (A∩B)

    Правило дополнительных событий:

    P (A ’) + P (A) = 1

    Непересекающиеся события:

    P (A∩B) = 0

    Независимые мероприятия:

    P (A∩B) = P (A) ⋅ P (B)

    Условная возможность:

    P (A | B) = P (A∩B) / P (B)

    Формула Байеса:

    Р (А | В) = Р (В | А) ⋅ Р (А) / Р (В)

    Что ж, ближе к делу: вычисление обозначений вероятности становится простым с помощью статистических событий или калькулятора условной вероятности.

    О калькуляторе вероятностей:

    расчет вероятности – это продвинутый инструмент, который позволяет узнать вероятность одного события, нескольких событий, двух событий и для серии событий. Кроме того, этот калькулятор работает как калькулятор условной вероятности, так как помогает вычислить условную вероятность заданного входа. Короче говоря, определение вероятности становится простым с помощью этого калькулятора вероятностных событий. Помимо уравнения вероятности, вы можете легко найти вероятность с помощью этого калькулятора вероятностей.

    как решать задачи на вероятность с помощью калькулятора:

    Что ж, вы можете легко рассчитать условные или вероятностные события с помощью этого калькулятора вероятностных событий, поскольку он загружен с удобным интерфейсом, он на 100% бесплатен для вычисления вероятностей. Читать дальше!

    Рассчитайте вероятность для одного события:

    Вход:

    • Прежде всего, вам нужно выбрать опцию «Single Probability» из выпадающего меню калькулятора.
    • Затем вы должны ввести количество возможных результатов в специальное поле.
    • Теперь вам нужно ввести количество произошедших событий (n) A в назначенное поле.

    Вывод:

    После этого нажмите кнопку «Рассчитать», расчет вероятности одного события сгенерирует:

    • Вероятность наступления события P (A) как в десятичном, так и в процентах
    • Вероятность события, которое не произойдет, P (A ‘) как в десятичном, так и в процентном выражении
    Рассчитайте вероятность нескольких событий:

    Вход:

    • Прежде всего, вы должны выбрать опцию «Вероятность нескольких событий» из раскрывающегося меню этого калькулятора вероятности для нескольких событий.
    • Сразу после этого вы должны ввести количество событий (n) A в заданные поля
    • Затем вы должны ввести количество событий (n) B в специальное поле этого калькулятора.

    Вывод:

    После того, как вы ввели все вышеперечисленные параметры, нажмите кнопку «Рассчитать», и этот расчет вероятности нескольких событий сгенерирует:

    • Вероятность наступления события P (A) как в десятичном, так и в процентах
    • Вероятность события, которое не произойдет, P (A ‘) как в десятичном, так и в процентном выражении
    • Вероятность наступления события B P (B) как в десятичном, так и в процентном выражении
    • Вероятность того, что событие B не произойдет, P (B ‘) как в десятичном, так и в процентном выражении
    • Вероятность наступления обоих событий P (A ∩ B) как в десятичной, так и в процентной форме.
    • Вероятность наступления любого из событий P (A ∪ B) как в десятичной, так и в процентной форме.
    • Условная вероятность P (A | B) как в десятичной, так и в процентной форме
    Рассчитайте вероятность двух событий:

    Вход:

    • Во-первых, вы должны выбрать опцию «Вероятность двух событий» в раскрывающемся меню этого калькулятора вероятности двух событий.
    • Затем вам нужно выбрать формат ввода, хотите ли вы добавить значения в десятичном формате или в процентах.
    • Сразу после этого вы должны добавить значение вероятности P (A) в обозначенное поле.
    • Затем вы должны добавить значение вероятности P (B) в обозначенное поле.

    Вывод:

    После того, как вы добавите все значения в указанные поля, нажмите кнопку вычислить, калькулятор вероятности двух событий сгенерирует:

    • Вероятность того, что событие не произойдет P (A ‘)
    • Вероятность того, что событие B не произойдет P (B ‘)
    • Вероятность наступления обоих событий P (A ∩ B)
    • Вероятность наступления любого из событий P (A ∪ B)
    • Вероятность появления A или B, но не обоих P (AΔB)
    • Вероятность того, что ни A, ни B не встретятся P ((A∪B) ‘)
    • Вероятность появления B, но не A

    Калькулятор покажет все указанные выше значения как в десятичном, так и в процентном формате.

    Рассчитайте вероятность серии событий:

    Вход:

    • Прежде всего, вы должны выбрать опцию «Вероятность серии событий» в соответствующем поле этого калькулятора вероятности серии событий.
    • Затем вы должны ввести значение вероятности и количество повторов для «События А» в предназначенное для этого поле.
    • Сразу после этого вы должны добавить значение вероятности и количество повторов для «События B» в данное поле.

    Вывод:

    После того, как вы ввели все значения в обозначенные поля, просто нажмите кнопку «Рассчитать», и эта вероятность мгновенно выдаст следующие результаты:

    • Вероятность появления А 2 раза
    • Вероятность того, что А не произойдет
    • Вероятность возникновения А
    • Вероятность появления B 4 раза
    • Вероятность того, что B не произойдет
    • Вероятность появления B
    • Вероятность того, что A встречается 2 раза, а B – 4 раза
    • Вероятность того, что не произойдет ни A, ни B
    • Вероятность появления как A, так и B
    • Вероятность появления A 2 раза, но не B
    • Вероятность появления B 4 раза, но не A
    • Вероятность появления A, но не B
    • Вероятность появления A, но не B
    Вычислить условную вероятность P (A | B):

    Вход:

    • Прежде всего, вы должны выбрать опцию «Условная вероятность P (A | B)» в специальном поле этого калькулятора условной вероятности.
    • Затем вы должны ввести значение вероятности a и b в обозначенное поле.
    • Затем вы должны ввести значение вероятности P (B) в предназначенное для этого поле.

    Вывод:

    После этого просто нажмите кнопку вычислить, калькулятор условной вероятности сгенерирует:

    • Условная вероятность P (A | B) как в десятичной, так и в процентной форме

    К счастью, найти вероятность a и b становится легко с помощью этого калькулятора условной вероятности.

    Каковы различные типы вероятностных событий:

    Прочтите, чтобы узнать о различных типах вероятностных событий:

    Простое событие:

    Если событие E содержит только одну точку выборки из пространства выборки, оно называется простым событием или элементарным событием. Помните, что это событие, которое содержит только один результат.

    Пример вероятности единичного события:

    Предположим, вы бросаете кубик, вероятность выпадения 2 на кубике считается простым событием и задается как E = {2}.

    Сложное событие:

    Если в пространстве для выборки имеется более одной точки выборки, то это считается сложным событием. Это событие предполагает объединение двух или более событий вместе и определение вероятности такой комбинации событий.

    Пример сложного события по вероятности:

    Когда вы бросаете кубик, существует вероятность появления четного числа, которая называется составным событием, поскольку существует более одной возможности, есть три возможности, которые равны E = {2,4,6}.

    Определенное событие:

    Определенное событие называется событием, которое обязательно произойдет в любом данном эксперименте. Вероятность такого события равна 1.

    Невозможное событие:

    Когда событие не может произойти, это означает, что событие не может произойти, тогда это считается невозможным событием. Вероятность невозможного события обозначается как 0.

    Пример невозможного события по вероятности:

    Карта, которую вы вытащили из колоды, красного и черного цвета, считается невозможным.

    Равно вероятные события:

    Если результаты эксперимента равновероятны, то они считаются равновероятными событиями.

    Пример равновероятных событий по вероятности:

    Когда вы подбрасываете монету, вероятность выпадения орла и решки одинакова.

    Бесплатные мероприятия:

    Для события E ненаступление события называется дополнительным событием. Обычно говорят, что дополнительные события – это события, которые не могут произойти одновременно.

    Пример вероятности дополнительных событий:

    Когда бросается кубик, получение нечетного и четного лиц считается дополнительными событиями.

    Взаимоисключающие события:

    Два события называются взаимоисключающими вероятностными событиями, когда оба не могут произойти одновременно. Помните, что взаимоисключающие вероятностные события всегда имеют разный исход. Два простых события всегда считаются взаимоисключающими, тогда как два составных события могут быть, а могут и не быть!

    Если A и B – два события, тогда;

    (A ∩ B) = Ø

    и,

    Вероятность пересечения

    P (A ∩ B) = 0

    Вероятность союза

    Р (А ∪ В) = Р (А) + Р (В)

    Зависимые вероятностные события и независимые вероятностные события (примеры задач):

    Опишем оба термина простыми словами:

    • Зависимые вероятностные события связаны друг с другом
    • Независимые вероятностные события не связаны между собой, значит, вероятность того, что одно произойдет, не влияет на другое.

    Вероятность двух событий, происходящих вместе – зависимая вероятность:

    Здесь уравнение вероятности, которое вы используете, немного отличается.

    P (A и B) = P (A) • P (B | A)

    Где;

    • P (B | A) просто обозначено как «вероятность B, если A произошло»)

    Пример проблемы:

    Если 85% сотрудников имеют медицинскую страховку, из 85% только 45% имели отчисления выше 1000 долларов. Итак, какой процент людей имел франшизу выше 1000 долларов?

    Шаг 1:

    • Вам нужно преобразовать проценты двух событий в десятичные числа, давайте посмотрим на пример.

    85% = 0,85.

    45% = 0,45.

    Шаг 2:

    • Теперь вам нужно умножить десятичные дроби из шага 1 вместе.

    0,85 x 0,45 = 0,3825 или 38,35 процента.

    Таким образом, вероятность того, что у физических лиц будет франшиза более 1000 долларов, составляет 38,35%.

    Вот как рассчитать вероятность того, что два события произойдут вместе!

    Вероятность двух событий, происходящих вместе – Независимая вероятность:

    Все, что вам нужно, это использовать определенную формулу правила умножения. Вам следует умножить вероятность первого события на второе. Например, если вероятность события A 2/9 и события B равна 3/9, то вероятность того, что оба события происходят одновременно, равна (2/9) * (3/9) = 6/81 = 2/27.

    Пример проблемы:

    Шансы получить работу, на которую вы подали заявку, составляют 45%, а шансы получить квартиру, на которую вы подавали заявку, составляют 75%, тогда как насчет вероятности того, что вы получите и новую работу, и новую квартиру?

    Шаг 1:

    • Вам следует преобразовать ваши проценты двух событий в десятичные числа, давайте взглянем на приведенный выше пример.

    45% = 0,45.

    75% = 0,75.

    Шаг 2:

    • Теперь вам нужно умножить десятичные дроби из шага 2 вместе:

    0,45 x 0,65 = 0,3375 или 33,75 процента.

    Итак, вероятность получить работу и квартиру составляет 33,75%.

    Вероятность A и B:

    Вероятность A и B означает, что вы хотите знать вероятность двух событий, которые происходят одновременно. Существуют разные формулы, которые полностью зависят от того, есть ли у вас зависимые события или независимые события.

    Формула для вероятности A и B (независимых событий): p (A и B) = p (A) * p (B)

    Помните, что если вероятность одного события не влияет на другое, значит, у вас независимое событие. Итак, как уже упоминалось ранее, вам нужно умножить вероятность одного на вероятность другого.

    Формула для вероятности A и B (зависимых событий): p (A и B) = p (A) * p (B | A)

    Помимо этих уравнений вероятностей, вы можете просто добавить параметры в указанный выше калькулятор вероятностей, чтобы определить вероятность событий.

    Как рассчитать вероятность (вручную, шаг за шагом)?

    Помимо уравнений вероятности, вы можете просто добавить параметры в приведенный выше калькулятор вероятностей, чтобы определить вероятность событий. Но, если вы хотите рассчитать вероятность вручную, то прочтите!

    Все, что вам нужно, чтобы рассчитать вероятность:

    • Прежде всего, вы должны определить одно событие с одним исходом.
    • Затем вы должны определить общее количество возможных результатов.
    • Затем вам нужно разделить количество событий на количество возможных результатов.

    Давайте копать глубже!

    Шаг № 1: Определите одно событие с одним результатом:

    Первым шагом к вычислению вероятности является определение вероятности, которую вы хотите вычислить. Это может быть указано как событие, предположим, что вероятность дождливой погоды, или выпадение определенного числа на кубике. Событие должно иметь хотя бы один возможный исход. Например, если вы хотите найти вероятность выпадения тройки с кубиком при первом броске, вы должны выяснить, что есть возможный результат: означает, что вы либо бросаете тройку, либо не бросаете тройку.

    Шаг № 2: Определите общее количество результатов:

    Затем вы должны определить количество результатов, которые могут возникнуть в результате события, которое вы определили на первом шаге. Если мы говорим о примере броска кубика, то всего может произойти 6 исходов, поскольку на кубике 6 чисел. Итак, ясно, что для одного события – выпадения трех – может произойти 6 различных результатов.

    Шаг № 3: Разделите количество событий на количество возможных результатов:

    После того как вы определили вероятностное событие вместе с соответствующими результатами, вам нужно разделить общее количество событий на общее количество возможных исходов. Например, бросок кубика один раз и выпадение тройки можно считать вероятностью одного события. Таким образом, вы можете продолжать бросать кубик – следовательно, каждый бросок будет считаться одним событием.

    Итак, из приведенного выше примера результат в дроби: 1/6.

    Как рассчитать вероятность с несколькими случайными событиями?

    Хотите мгновенно рассчитать вероятность нескольких событий, а затем просто расчет вероятности для нескольких событий. Несомненно, вычисление вероятности с несколькими случайными событиями очень похоже на вычисление вероятности с одним событием, однако есть лишь несколько дополнительных шагов, которые нужно придерживаться, чтобы достичь окончательного решения. Следующие ниже шаги показывают, как рассчитать вероятность нескольких событий:

    • Прежде всего, вы должны определить каждое событие, которое вы будете рассчитывать.
    • Затем вам нужно рассчитать вероятность каждого события.
    • Наконец, вам нужно умножить все вероятности вместе

    Часто задаваемые вопросы (о вероятности):

    Как найти вероятности с процентами?

    Если вы хотите рассчитать вероятность в процентах, вам следует решить задачу, как обычно, то есть вам нужно преобразовать свой ответ в процент.

    Например;

    Если количество желаемых результатов разделить на количество возможных событий, равное 0,25, тогда вам следует умножить ответ на 100, чтобы получить 25%. Если есть вероятность определенного исхода в процентной форме, тогда вам просто нужно разделить процент на 100, а теперь умножить его на количество событий, чтобы вычислить вероятность.

    Как рассчитать вероятность на калькуляторе?

    Все, что вам нужно для ввода значений в указанные выше поля, калькулятор вероятностей сделает все за вас в течение нескольких секунд.

    Каковы 3 типа вероятности?

    Три типа вероятности следующие:

    • Классический
    • Определение относительной частоты
    • Субъективная вероятность

    Каковы 5 правил вероятности?

    Основные правила вероятности:

    • Правило вероятности первое – (Для любого события A, 0 ≤ P (A) ≤ 1)
    • Правило вероятности два – (Сумма вероятностей всех возможных исходов равна 1)
    • Правило вероятности третье – (Правило дополнения)

    Вероятности, связанные с несколькими событиями:

    • Правило вероятности четвертое (правило сложения для непересекающихся событий)

    Нахождение P (A и B) с помощью логики:

    • Правило вероятности пятое – (Общее правило сложения)

    Как я могу определить вероятность при выборе случайных чисел?

    Запомните все это на основе диапазона генератора случайных чисел. Например, если диапазон от 1 до 9, то вероятность получения определенного числа считается равной 1/9.

    Если я брошу кубик 6 раз, какова вероятность?

    Вероятность того, что он хотя бы раз выпадет на 6, составляет 66,5%.

    Если я брошу обычный шестигранный кубик, какова вероятность получить 5?

    Тогда ваш ответ будет 1/6, или примерно 17%.

    Если один раз бросить шестигранный кубик, какова вероятность выпадения 1 или 2?

    2/6, после подбрасывания кубика вероятность получить 1 равняется 1/6, а вероятность получения 2 также равна 1/6. Таким образом, 1/6 + 1/6 = 2/6 или 1/3 или 0,333.

    Как рассчитать вероятность футбольных матчей?

    На самом деле, ты не можешь. Единственное, от чего можно уйти, так это их умения. Помните, что игроки тоже люди, и у них может быть плохой день, а это значит, что они играют не так хорошо, как обычно!

    Где мы используем вероятность в реальной жизни?

    Вот примеры вероятности из реальной жизни:

    • Прогноз погоды
    • Среднее значение по крикету
    • Политика
    • Подбрасывание монеты или кубика
    • Страхование
    • Вы скорее всего погибнете в результате несчастного случая
    • Лотерейные билеты
    • Играя в карты

    Вывод:

    Помните, что вероятность – это то, что дает вам информацию о вероятности того, что что-то произойдет. Итак, просто воспользуйтесь приведенным выше калькулятором вероятностей, чтобы вычислить вероятность событий или в соответствии с условиями!

    Other languages: Probability Calculator, olasılık hesaplama, kalkulator prawdopodobieństwa, kalkulator probabilitas, wahrscheinlichkeitsrechner, 確率 計算, 확률 계산기, pravděpodobnost kalkulačka, calculo de probabilidade, calcul de probabilité, calculo de probabilidad, calcolo probabilità, todennäköisyys laskuri, sandsynlighedsregning, sannsynlighetskalkulator.

    Калькулятор вероятности, пошаговый расчет

    Выберите вероятность для расчета и выберите соответствующие данные для пошаговых инструкций и решений

    Возможности калькулятора вероятности:

    • Калькулятор зависимой вероятности
    • Независимый калькулятор вероятности
    • Условная вероятность калькулятор
    • Калькулятор теоремы Байеса

    Как пользоваться калькулятором вероятностей?

    Калькулятор вероятности выберет соответствующие формулы для расчета вашего ответа на основе типа вероятности и доступных данных.
    Какую вкладку выбрать?

    Вы смотрите только на одно событие?

    Да — используйте калькулятор отдельных событий . Введите количество наблюдаемых благоприятных событий, а также общее количество наблюдений

    Нет — продолжить

    Влияет ли исход одного события на исход другого?

    Например, количество облаков на небе влияет на вероятность дождя.

    Да — продолжить

    Нет — используйте калькулятор независимых событий .

    У вас есть одна условная вероятность, которую вы хотите преобразовать в другую условную вероятность (теорема Байеса)?

    Да — использовать Калькулятор теоремы Байеса . Введите P(A), P(B) и P(B|A), чтобы получить P(A|B)

    Нет — используйте калькулятор зависимых событий .

    Общие

    Вероятностная терминология
    • Событие – конкретный исход или набор исходов. Если бы мы бросили кубик, то и кубик с шестью, и кубик с четным числом (включая события 2, 4 и 6) были бы событиями.
    • Вероятность — число от 0 до 1, которое используется для описания вероятности возникновения определенного события.
      Для любого события E вероятность или правдоподобие этого события записывается как P(E).
      Независимо от того, как мы выбираем E, P(E) всегда находится в диапазоне от 0 до 1: 0 ≤ P(E) ≤ 1
      Если P(E) = 0, то событие никогда не произойдет.
      Если P(E) = 1, то событие гарантированно произойдет.
      Обычно P(E) находится между этими двумя вариантами, поэтому событие может произойти маловероятно, иметь равные шансы произойти или может произойти.
    • Пространство выборки — набор всех возможных результатов эксперимента.
      Например, при броске игральной кости у нас есть 6 возможных результатов: числа 1, 2, 3, 4, 5 и 6 на верхней грани кости.
      Когда мы говорим о подбрасывании монеты, мы, очевидно, должны включать орел и решку в наше выборочное пространство.
      Однако мы также должны включить монету, приземлившуюся на бок, поскольку это отдельная возможность
      , и все варианты должны быть учтены в пространстве выборки. Поскольку мы перечисляем все результаты в выборке,
      мы знаем, что результатом должен быть ровно один результат. Отсюда мы можем вывести следующее правило:
      сумма вероятностей каждого исхода в пространстве выборок равна 1.
      Например, при бросании обычной кости возможными исходами являются числа от 1 до 6, поэтому P(1 )+P(2)+P(3)+P(4)+P(5)+P(6) = 1,
    • P(A∩B) (пересечение A и B)- Вероятность того, что оба произойдет событие А и событие В.
    • P(A∪B) (объединение A и B) — Вероятность того, что хотя бы одно из событий A и B произойдет.
    • n(E) — количество исходов в событии E. Например, если E — событие, представляющее собой четный бросок игральной кости, то n(E)=3 (2, 4 и 6)
    • Взаимоисключающие — два события A и B являются взаимоисключающими, если они никогда не могут произойти одновременно, т. е. P(A ∩ B) = 0,
    Вероятностное правило

    Как рассчитать вероятность события ? Один из способов сделать это — найти количество благоприятных исходов и разделить его на общее количество исходов следующим образом: P(E) = n(E) / n(S)
    Для нашего события E, где S — выборочное пространство
    Например, предположим, что мы бросили 2 игральные кости и хотели получить в сумме 4.
    Все события исходы в выборочном пространстве:
    [1,1],[1,2],…[1,6],…,[2,1],[2,2],..[2, 6],…[6,1],[6,2],…,[6,6].
    36 различных исходов (6 для первого кубика * 6 для второго кубика)
    , поэтому n(S) = 36
    желательные исходы: [1,3],[2,2],[3,1].
    3 различных исхода, поэтому n(сумма 4) = 3
    Итак, P(сумма 4) = n(сумма 4)/n(S)
    = 3 / 36
    = 1 / 12
    Однако этот подход иногда наивен, поскольку мы предполагаем, что все исходы имеют одинаковую вероятность.
    При проведении эксперимента мы не всегда можем предполагать, что результаты равновероятны. Например,
    мы могли бы бросить смещенную монету с вероятностью 80% выпадения орла и вероятностью 20% выпадения решки. В этом случае мы не можем рассматривать орел и решку как равновероятные исходы. Инстинктивно мы могли бы просто добавить P(A) и P(B). Однако, рисуя это, мы получили бы

    AB + AB = AB

    Это близко к ожидаемому результату, за исключением того, что здесь мы считаем P(A∩B) дважды, один раз как часть A и один раз как часть B.
    Следовательно, чтобы получить P(A ∪B) нам нужно вычесть пересечение A и B. Это приводит нас к формуле сложения.
    P(A∪B) = P(A) + P(B) — P(A∩B)

    Зависимая или независимая вероятность

    Являются ли эти события зависимыми или независимыми?
    Мы можем проверить, независимы ли два события, с помощью следующих уравнений:
    P(A∩B) = P(B) * P(A)
    P(A|B) = P(A)
    Если выполняется любое из этих уравнений, то мы знаем, что A и B независимы.
    Если мы не можем показать, что одна из этих формул верна, то мы должны предположить, что события зависимы при решении задачи.

    Зависимые события

    Правило умножения

    Что такое правило умножения?
    Это правило гласит, что P(A ∩ B) = P(B) * P(A|B)
    Его можно приблизительно прочитать как вероятность того, что произошло и A, и B, равна вероятности того, что произошло B и в этой вселенной тоже произошло А.
    Поскольку это именно то условие, при котором истинно A ∩ B, это верно для зависимого и независимого расчета вероятности.

    Формула условной вероятности

    Как работает формула условной вероятности?
    Допустим, у нас есть 2 события, A и B, и мы хотели вычислить вероятность A при заданном B, P(A|B).
    Мы могли бы начать с выделения A, потому что мы рассматриваем результаты внутри этого круга.
    Тем не менее, у нас есть дополнительная информация для ответа на вопрос — мы знаем, что B произошло.
    Это означает, что мы можем исключить все, что не входит в B, поскольку мы знаем, что рассматриваем исходы, в которых произошло B.
    Мы можем представить это на диаграмме Венна следующим образом:

    AB

    Из этого мы можем видеть, что вероятность A (оранжевого цвета) при заданном B (более светлом цвете) равна P(A∩B)/P(B)

    Теорема Байеса

    Формула условной вероятности:
    P(A|B) = P(A∩B) / P(B)
    = P(A) * P(B|A) / P(B) из правила умножения , подгруппа в P(A∩B) = P(B) * P(B|A)

    Независимые события

    Рассмотрение независимых событий похоже на рассмотрение зависимых событий, за исключением того, что мы также знаем, что P(A|B) = P(A),
    Поскольку вероятность события A не зависит от события B.
    P( A∩B) = P(B) * P(A|B) (из правила умножения)
    P(A∩B) = P(B) * P(A), так как мы знаем, что P(A) = P (A|B)

    Калькулятор нормального распределения

    Калькулятор нормального распределения упрощает вычисление кумулятивного вероятность при стандартной оценке из стандартного нормального распределения или необработанная оценка из любого другого нормального распределения; и наоборот. За помощью в использовании калькулятор, прочтите часто задаваемые вопросы или просмотрите примеры задач.

    Чтобы узнать больше о нормальном распределении, посетите сайт Stat Trek. учебник по нормальному распределению.

    • Введите значение в три из четырех текстовых полей.
    • Оставьте четвертое текстовое поле пустым.
    • Нажмите кнопку Вычислить , чтобы вычислить значение для четвертого текстового поля.
    Стандартная оценка: z
    Вероятность: P(Z≤z)
    Среднее
    Стандартное отклонение

    Примечание : Таблица нормального распределения, приведенная в приложении большинство статистических текстов основано на стандартное нормальное распределение, имеющее среднее значение 0 и стандартное отклонение 1. Для получения выходных данных из стандартного нормального распределения с помощью этого калькулятора, установите среднее значение равным 0, а стандартное отклонение равным 1. Для получения выходных данных из любого другого нормального распределения, установите среднее значение равным чему-то, кроме 0, и/или установите стандартное отклонение равным чему-то кроме 1,

    Часто задаваемые вопросы

    Калькулятор | Пример задачи

    Инструкции: Чтобы найти ответ на часто задаваемый вопрос, просто нажмите на вопрос.

    Почему нормальное распределение так важно?

    Нормальное распределение важно, поскольку оно описывает статистическое поведение многих реальных событий. Форма в норме распределение полностью описывается средним значением и стандартным отклонением.

    Таким образом, зная среднее значение и стандартное отклонение, вы можете использовать свойства нормального распределения для быстрого вычисления кумулятивного вероятность любого значения. Этот процесс проиллюстрирован на Примеры задач ниже.

    Что такое стандартное нормальное распределение?

    Существует бесконечное количество нормальных распределений. Хотя каждое нормальное распределение имеет колоколообразную кривую, некоторые нормальные распределения иметь высокую и узкую кривую; в то время как другие имеют короткую кривую и широкий.

    Точная форма нормального распределения определяется его среднее значение и его стандартное отклонение. Стандартное нормальное распределение — это нормальное распределение, имеющее среднее значение, равное нулю, и стандартное отклонение, равное единице.

    Обычный случайная величина стандартного нормального распределения называется стандартом оценка или z-оценка . Нормальная случайная величина X из любого нормального распределения можно преобразовать в z-оценку из стандартное нормальное распределение по следующему уравнению:

    z = ( X — μ) / σ

    , где X — нормальная случайная величина, μ — среднее значение, σ — стандартное отклонение.

    Поскольку любую нормальную случайную величину можно «преобразовать» в z-значение, стандартное нормальное распределение обеспечивает полезную систему отсчета. На самом деле, это нормальное распределение, которое обычно приводится в приложении. учебников по статистике.

    Что такое нормальная случайная величина?

    Нормальное распределение определяется следующим уравнением:

    Нормальное уравнение . Значение случайной величины Y :

    Y = { 1/[ σ * sqrt(2π) ] } * e -(x — μ) 2 /2σ 2

    , где X — нормальная случайная величина, μ — среднее значение, σ — стандартное отклонение, π — приблизительно 3,14159, а e — приблизительно 2,71828.

    В этом уравнении случайная величина X называется нормальной случайной величиной. Уникальный кумулятивная вероятность может быть связана с каждой нормальной случайной величиной. Учитывая нормальную случайную величину, стандартное отклонение нормального распределение и среднее значение нормального распределения, мы можем вычислить кумулятивная вероятность (т. е. вероятность того, что случайный выбор из нормальное распределение будет меньше или равно нормальной случайной величине.)

    Что такое z-показатель?

    Z-оценка (также известная как стандартная оценка) нормальная случайная величина из стандартное нормальное распределение.

    Чтобы преобразовать обычную случайную величину (x) в эквивалентную z-оценка (z), используйте следующую формулу:

    z = ( x — μ) / σ

    , где μ — среднее значение, а σ — стандартное отклонение.

    Что такое вероятность?

    Вероятность – это число, выражающее вероятность того, что конкретное произойдет событие. Это число может принимать любое значение от 0 до 1. Вероятность 0 означает, что вероятность того, что событие произойдет, равна нулю; вероятность 1 означает, что событие обязательно произойдет.

    Количественное определение чисел от 0 до 1 неопределенность, связанная с событием. Например, вероятность подбрасывание монеты, в результате которого выпадет орел (а не решка), будет равно 0,50. Пятьдесят процентов в то время при подбрасывании монеты выпадал орел; и пятьдесят процентов время, это приведет к хвостам.

    Что такое кумулятивная вероятность?

    Кумулятивная вероятность представляет собой сумму вероятностей. В связи при нормальном распределении кумулятивная вероятность относится к вероятность того, что случайно выбранный результат будет меньше или равен заданное значение, называемое нормальной случайной величиной.

    Предположим, например, что у нас есть школа со 100 первоклассники. Если мы спросим о вероятности того, что случайно выбранный первый грейдер весит ровно 70 фунтов, мы спрашиваем о простой вероятности — не кумулятивная вероятность.

    Но если мы спросим о вероятности того, что случайно выбранный первоклассник на меньше или равен до 70 фунтов, мы действительно спрашиваем о сумме вероятностей (т. е. вероятности того, что студент точно 70 фунтов плюс вероятность того, что он/она весит 69 фунтов плюс вероятность что он/она весит 68 фунтов и т. д.). Таким образом, мы спрашиваем о совокупном вероятность.

    Что такое средний балл?

    Средний балл — это средний балл. это сумма индивидуальных баллы разделить на количество человек.

    Что такое стандартное отклонение?

    Стандартное отклонение — это числовое значение, используемое для указания того, как широко оценки в наборе данных варьируются. Это мера среднего расстояния индивидуальные наблюдения от среднего значения группы.

    Пример задачи

    Калькулятор | Часто задаваемые вопросы


    1. Компания Acme Light Bulb обнаружила, что средняя лампочка работает 1000 часов. часов со стандартным отклонением 100 часов. Предположим, что срок службы лампы в норме распределенный. Какова вероятность того, что случайно выбранная лампочка выгорают за 1200 часов или меньше?

      Решение:

      Мы знаем следующее:

      • Средний балл 1000.
      • Стандартное отклонение равно 100.
      • Необработанная оценка, для которой мы хотим найти кумулятивную вероятность, составляет 1200.

      Поэтому мы подставляем эти числа в Обычный Калькулятор распределения и нажмите кнопку «Рассчитать».

      Калькулятор сообщает, что кумулятивный вероятность равна 0,97725. Таким образом, вероятность того, что Acme Light Лампа перегорит в течение 1200 часов.

    1. Билл утверждает, что может сделать больше 9 отжиманий.0% мальчиков в его школе. В прошлом году средний мальчик сделал 50 отжиманий со стандартным отклонением 10. отжимания. Предположим, что производительность отжиманий нормально распределена. Сколько отжиманий что должен сделать Билл, чтобы победить 90% других мальчиков?

      Решение:

      Мы знаем следующее:

      • Средний балл 50.
      • Стандартное отклонение равно 10.
      • Кумулятивная вероятность равна 0,9.0, так как Билл должен превзойти 90% мальчики. (Если бы он утверждал, что превосходит только 80% мальчиков, совокупный вероятность будет 0,80.)

      Поэтому мы подставляем эти числа в Обычный Калькулятор распределения и нажмите кнопку «Рассчитать».

      Калькулятор сообщает, что исходный балл составляет 62,8.

    8 и 6 наименьшее общее кратное: НОД и НОК для 6 и 8 (с решением)

    2

    Число 24

    Свойства и характеристики одного числа
    Все делители числа, сумма и произведение цифр, двоичный вид, разложение на простые множители…

    Свойства пары чисел
    Наименьшее общее кратное, наибольший общий делитель, сумма, разность и произведение чисел…

    Сейчас изучают числа:

    8 и 6 2011 2022 24742000 277353 31 8743 6929 5713 339 2247 110010100101 1043 1027 1019 1003 320 1543 36 и 45 444888 20449 594 и 60 500 391

    Двадцать четыре

    Описание числа 24

    Неотрицательное рациональное двузначное число 24 является составным числом. Произведение цифр числа: 8. Число имеет следующие делители: 1, 2, 3, 4, 6, 8, 12, 24. Сумма делителей: 60. Обратным числом является 0.041666666666666664.
    Данное число представляется произведением: 2 * 2 * 2 * 3.

    Другие представления числа 24: двоичная система: 11000, троичная система: 220, восьмеричная система: 30, шестнадцатеричная система: 18. 24 байта представляет из себя число байт 24.

    В виде кода азбуки Морзе: ..— ….-

    Косинус 24: 0.4242, синус 24: -0.9056, тангенс 24: -2.1349. Натуральный логарифм числа равен 3.1781. У числа 24 есть десятичный логарифм: 1.3802. Квадратный корень числа: 4.8990, а кубический: 2.8845. Возведение в квадрат: 576.00.

    24 в секундах это 24 секунды . В нумерологии число 24 означает цифру 6.

    • ← 23
    • 25 →

    Вычисление наименьшего общего кратного

    Введите цифры


    • Три автобуса
      Три автобуса общественного транспорта отправляются вместе с автовокзала утром. Первый автобус возвращается на станцию ​​через 18 минут, второй – через 12 минут, а третий – через 24 минуты. Как долго снова будем вместе на вокзале? Пожалуйста, экспресс
    • Портниха
      Портниха оставила кусок холста короче 5 метров. Она решает, сшить ли ей юбку или платье. Холста было ровно столько, сколько они израсходовали, разрезав юбку до 120 см, или 180 сантиметров. Какой кусок холста оставил ей?
    • LCM двух чисел
      Найдите наименьшее кратное 63 и 147
    • Различные 6975
      Три разных автобусных маршрута, 80, 81 и 82, отправляются с конечной станции в 5 ч 20 мин. Маршрут 80 отправляется каждые 30 минут, маршрут 81 — каждые 20 минут, а маршрут 82 — каждые 40 минут. Во сколько они снова уйдут?
    • Напоминание и частное
      Даны числа A = 135, B = 315. Найдите наименьшее натуральное число R, большее единицы, чтобы отношения R:A, R:B были с остатком 1.
    • Бакалейная лавка
      Сьюзен решила сделать продуктовые наборы для своего магазина. Оптовый торговец, у которого она покупает, продает сахар в упаковках по 20 штук в коробке, муку в упаковках по 12 штук в коробке и 15 мешков риса в коробке. Сколько штук каждого предмета она должна купить, чтобы их было одинаковое количество
    • Вокруг клумбы
      Вокруг прямоугольной клумбы размерами 5,25 м и 3,5 м нужно посадить розы через равные промежутки так, чтобы розы находились в каждом углу клумбы и потреблять как можно меньше. а) На каком расстоянии посажены розы? б) Сколько роз
    • Автобусы
      На остановке в 10 часов встретились автобусы №2 и №9. Автобус №2 ходит с интервалом 4 минуты, а автобус №9 с интервалом 9 минут. Сколько раз автобус встречается в 18:00 по местному времени?
    • Зубчатая передача
      Зубчатая передача состоит из двух колес. У одного 88, а у второго 56 зубов. Сколько раз поверните меньшее колесо, чтобы попасть в те же зубья, что и в начале? Сколько раз мы повернём самое большое колесо?
    • Автобусы 4
      Интервалы: 1-й автобус 40 мин. 2-й автобус 2 часа 3-й бутон 20 минут Через какое время они встретятся — как можно скорее?
    • Четыре класса
      Учащиеся всех 7, 8 и 9 классов одной школы могут занимать 4, 5, 6 и 7 ряд подряд, и никого не останется. Сколько в среднем учеников в одном классе, если в каждом классе всегда четыре класса?
    • Gcd и lcm
      Вычислить наибольший общий делитель и наименьшее общее кратное чисел. a) 16 и 18 b) 24 и 22 c) 45 и 60 d) 36 и 30
    • Вычислить 2976
      Вычислить наименьшее общее кратное чисел 120, 660 и 210.
    • Уточните: 4001
      Укажите: a = D (240,320) b = n (40,64)
    • Pardubická 4651
      Йирка решил разделить выигрыш от пари в Velká Pardubická между собой и тремя своими младшими братьями по возрасту в соотношении 2:3:5:7. Каждую сумму они платили целыми кронами. Одна из сумм составила 679 чешских крон. Насколько велик был выигрыш?
    • Веревка
      Пол может разрезать веревку на равные части, не оставив ни одной веревки. Длина может быть 15 см, 18 см или 25 см. Какова наименьшая возможная длина веревки?

    другие математические задачи »

    Что такое наибольший общий делитель и наименьшее общее кратное?

    Привет! Добро пожаловать в это видео, посвященное наименьшему общему кратному и наибольшему общему делителю!

    Как вы знаете, бывают случаи, когда нам приходится алгебраически «настраивать» то, как выглядит число или уравнение, чтобы продолжить нашу математическую работу. Для этого мы можем использовать наибольший общий делитель и наименьшее общее кратное. Наибольший общий делитель (GCF) — это наибольшее число, которое является множителем двух или более чисел, а наименьшее общее кратное (НОК) — это наименьшее число, кратное двум или более числам.

    Чтобы понять, насколько полезны эти понятия, давайте рассмотрим сложение дробей. Прежде чем мы сможем складывать дроби, мы должны убедиться, что знаменатели совпадают, создав эквивалентную дробь:

    \(\frac{2}{3}+\frac{1}{6} \rightarrow \frac{2 }{3} \times \frac{2}{2}\)\(+\frac{1}{6} \rightarrow \frac{4}{6} +\frac{1}{6}=\frac{ 5}{6}\)

     

    В этом примере необходимо определить наименьшее общее кратное 3 и 6. Другими словами, «Каково наименьшее число, на которое можно разделить без остатка и 3, и 6?» Немного подумав, мы понимаем, что 6 является наименьшим общим кратным, потому что 6, деленное на 3, равно 2, а 6, деленное на 6, равно 1. Затем дробь \(\frac{2}{3}\) приводится к эквиваленту дробь \(\frac{4}{6}\) путем умножения числителя и знаменателя на 2. Теперь можно сложить две дроби с общими знаменателями, чтобы получить окончательное значение \(\frac{5}{6}\) .

    Найдите наименьшее общее кратное

    В контексте сложения или вычитания дробей наименьшее общее кратное называется наименьшим общим знаменателем .

    Как правило, вам нужно определить число, большее или равное двум или более числам, чтобы найти их наименьшее общее кратное.

    Важно отметить, что существует несколько способов определения наименьшего общего кратного. Один из способов — просто перечислить все кратные рассматриваемых значений и выбрать наименьшее общее значение, как показано здесь:

    Наименьшее общее кратное 8, 4, 6

    \(8\rightarrow 8,16,24,32,40,48\)
    \(4\rightarrow 4,8,12,16,20,24, 28,32\)
    \(6\стрелка вправо 6,12,18,24,30,36\)

     

    Это показывает, что наименьшее общее кратное чисел 8, 4 и 6 равно 24, поскольку оно является наименьшим число, на которое можно поровну разделить 8, 4 и 6.

    Другой распространенный метод включает первичную факторизацию каждого значения. Помните, что простое число делится только на 1 и само на себя.

    После определения простых множителей перечислите общие множители один раз, а затем умножьте их на остальные оставшиеся простые множители. Результатом является наименьшее общее кратное:

    \(30=2\умножить на 2\умножить на 3\умножить на 3\)
    \(90=2\умножить на 3\умножить на 3\умножить на 5\)

    \(\text {НОК}=2\умножить на 3\умножить на 3\умножить на 2\умножить на 5\)

     

    Наименьшее общее кратное также можно найти путем общего (или повторного) деления. Этот метод иногда считают более быстрым и эффективным, чем листинг 9.0062 умножить на и найти простые множители. Вот пример нахождения наименьшего общего кратного чисел 3, 6 и 9 с помощью этого метода:

    Разделите числа на множители любого из трех чисел. 6 имеет коэффициент 2, поэтому давайте использовать 2. Девять и 3 не могут делиться на 2, поэтому мы просто перепишем здесь 9 и 3. Повторяйте этот процесс, пока все числа не будут уменьшены до 1. Затем перемножьте все множители вместе, чтобы получить наименьшее общее кратное.

    2 3 6 9
    3 3 3 9
    3 1 1 3
    1 1 1

    НОК \(=2\умножить на 3\умножить на 3=18\)

    Теперь, когда были введены методы нахождения наименьших общих кратных, нам нужно изменить свое мышление, чтобы найти наибольший общий множитель двух или более числа. Мы будем определять значение, меньшее или равное рассматриваемым числам. Другими словами, спросите себя: «Какое наибольшее число делит оба этих числа?» Понимание этой концепции необходимо для деления и факторизации многочленов.

    Какой наибольший общий делитель?

    Разложение на простые множители также можно использовать для определения наибольшего общего делителя. Однако вместо того, чтобы умножать все простые множители, как мы это делали для наименьшего общего кратного, мы будем умножать только те простые множители, которые являются общими для чисел. Полученное произведение является наибольшим общим множителем.


    Обзор

    Давайте завершим парой правильных или неверных вопросов:

    1. Наименьшее общее кратное 45 и 60 равно 15.

     

    2. Наименьшим общим кратным является число, большее или равное рассматриваемым числам.

     

    Спасибо за просмотр и приятного изучения!

    Часто задаваемые вопросы

    Q

    Как найти LCM и GCF?

    A

    Существует множество методов определения LCM и GCF. Две наиболее распространенные стратегии включают составление списка или использование простой факторизации.

    Например, НОК 5 и 6 можно найти, просто перечислив кратные \(5\) и \(6\), а затем указав наименьшее кратное, общее для обоих чисел.
    \(5, 10, 15, 20, 25, \mathbf{30}, 35…\)
    \(6, 12, 18, 24, \mathbf{30}, 36…\)
    \(\mathbf{ 30}\) — LCM.

    Аналогичным образом, GCF можно найти, перечислив факторы каждого числа, а затем указав наибольший общий фактор. Например, GCF для \(40\) и \(32\) можно найти, перечислив множители каждого числа.

    \(40\): \(1, 2, 4, 5, \mathbf8, 10, 20, 40\)
    \(32\): \(1, 2, 4, \mathbf8, 16, 32\ )
    \(\mathbf8\) — это GCF.

    Для больших чисел будет нереально составить список факторов или множителей для определения GCF или LCM. Для больших чисел наиболее эффективно использовать метод простой факторизации.

    Например, при нахождении НОК начните с нахождения разложения каждого числа на простые множители (это можно сделать, создав дерево факторов). Простая факторизация \(20\) равна \(2\times2\times5\), а простая факторизация \(32\) — это \(2\times2\times2\times2\times2\). Обведите общие факторы и посчитайте эти только один раз .

    Теперь умножьте все множители (помните, что не нужно дважды считать обведенные \(2\)). Это становится \(2\times2\times5\times2\times2\times2\), что равняется \(160\). LCM \(20\) и \(32\) равен \(160\).

    Находя GCF, начните с перечисления простой факторизации каждого числа (это можно сделать, создав дерево факторов). Например, простая факторизация \(45\) равна \(5\times3\times3\), а простая факторизация \(120\) равна \(5\times3\times2\times2\times2\). Теперь просто умножьте все факторы, которые являются общими для обоих чисел. В этом случае мы умножим \(5\times3\), что равно \(15\). GCF \(45\) и \(120\) равен \(15\).

    Подход с простой факторизацией может показаться довольно длительным процессом, но при работе с большими числами он гарантированно сэкономит время.

    Q

    Как найти GCF?

    A

    Существуют две основные стратегии нахождения GCF: перечисление факторов или использование простой факторизации.

    Первая стратегия состоит в том, чтобы просто перечислить множители каждого числа, а затем найти наибольший множитель, общий для обоих чисел. Например, если мы ищем GCF для \(36\) и \(45\), мы можем перечислить множители обоих чисел и определить наибольшее общее число.
    \(36\): \(1,2,3,4,6,\mathbf9,12,18,36\)
    \(45\): \(1,3,5,\mathbf9,15,45 \)
    GCF для \(36\) и \(45\) равен \(\mathbf9\).

    Составление списка множителей каждого числа с последующим определением наибольшего общего множителя хорошо работает для небольших чисел. Однако при нахождении GCF очень больших чисел более эффективно использовать метод простой факторизации.

    Например, при нахождении GCF чисел \(180\) и \(162\) мы начинаем с перечисления простой факторизации каждого числа (это можно сделать, создав дерево факторов). Простая факторизация \(180\) равна \(2\times2\times3\times3\times5\), а простая факторизация \(162\) — это \(2\times3\times3\times3\times3\). Теперь найдите факторы, которые являются общими для обоих чисел. В этом случае оба числа имеют одну общую \(2\) и две \(3\)s, или \(2\times3\times3\). Результатом \(2\times3\times3\) является \(18\), что является GCF! Эта стратегия часто более эффективна при нахождении GCF действительно больших чисел.

    Q

    Что означает GCF?

    A

    GCF означает «наибольший общий делитель». GCF определяется как наибольшее число, являющееся множителем двух или более чисел. Например, GCF для \(24\) и \(36\) равен \(12\), потому что наибольший делитель, общий для \(24\) и \(36\), равен \(12\). \(24\) и \(36\) имеют другие общие факторы, но \(12\) является самым большим.

    Q

    Как найти наименьшее общее кратное?

    A

    Существует множество методов нахождения наименьшего общего кратного. Двумя распространенными подходами являются перечисление кратных и использование простой факторизации. Список кратных так же, как это звучит, просто перечислите кратные каждого числа, а затем найдите наименьшее кратное, общее для обоих чисел. Например, при нахождении наименьшего общего кратного чисел \(3\) и \(4\) перечислите кратные:
    \(3\): \(3,6,9,\mathbf{12},15,18 …\)
    \(4\): \(4,8,\mathbf{12},16,20…\)
    \(\mathbf{12}\) — это наименьшее кратное, общее для \(3\) и \(4\).

    Список кратных — отличная стратегия, когда числа довольно малы. Когда числа большие, такие как \(38\) и \(42\), мы должны использовать метод простой факторизации. Начните с перечисления простой факторизации каждого числа (это можно сделать с помощью факторного дерева).
    \(38\): \(2\times19\)
    \(42\): \(2\times3\times7\)
    Теперь обведите общие множители (считайте эти только один раз ).

    Теперь умножьте все множители (не забудьте подсчитать \(2\) только один раз). Это становится \(2\times19\times3\times7\), что равняется \(798\). LCM \(38\) и \(42\) равен \(798\).

    Q

    Как вынуть LCM?

    A

    Извлечение НОК полезно при сложении или вычитании дробей. При определении наименьшего общего кратного получается знаменатель, одинаковый для обеих дробей. Например, общий знаменатель для \(\frac{2}{7}+\frac{3}{5}\) будет равен \(35\), потому что \(35\) является НОК для \(7\ ) и \(5\). Новые дроби становятся \(\frac{10}{35}+\frac{21}{35}\), что равняется \(\frac{31}{35}\).

    Практические вопросы

    Вопрос №1:

     
    Каков наибольший общий делитель чисел 16 и 42? Используйте его, чтобы уменьшить дробь \(\frac{16}{42}\).

    GCF равно 8, и мы уменьшаем его до \(\frac{2}{5}\).

    GCF равен 1, и мы не можем больше уменьшать.

    GCF равно 4, и мы уменьшаем его до \(\frac{4}{11}\).

    GCF равен 2, и мы уменьшаем его до \(\frac{8}{21}\).

    Показать ответ

    Ответ:

    Правильный ответ D: GCF равен 2, и мы уменьшаем до \(\frac{8}{21}\).

    Давайте подойдем к этой проблеме, перечислив простые множители как числителя, так и знаменателя.
    \(16=2×2×2×2\)
    \(42=2×3×7\)

    Здесь мы видим, что 2 — единственный общий делитель чисел 16 и 42 и, следовательно, их наибольший общий делитель. Затем мы можем разделить оба числа на 2, чтобы уменьшить дробь:
    \(\frac{16\div2}{42\div2}=\frac{8}{21}\)

    Скрыть ответ

    Вопрос № 2:

     
    Найдите наименьшее общее кратное 2, 6 и 8.

    16

    18

    24

    48

    Показать ответ

    Ответ:

    Правильный ответ C: 24,

    2 простых чисел.


    \(2=2\) (обратите внимание, что мы могли написать \(2\times1\), но 1 понимается или подразумевается, и обычно писать не нужно)
    \(6=2\times3\)
    \( 8=2\times2\times2\)

    Помните, что при вычислении НОК двух или более чисел мы перечисляем каждый простой множитель один раз, который является общим для всех чисел. Поскольку каждое из наших чисел имеет 2 в качестве простого множителя, наш LCM также будет иметь 2 в качестве одного из своих простых множителей.
    LCM \(=2\times\) _______

    Теперь из 6 у нас осталась 3, а из 8 осталось две двойки. Мы умножаем их, чтобы получить
    LCM \(=2\times3\times2\times2=24\)

    Обратите внимание, что хотя 2, 6 и 8 являются делителями 48, решение не D, потому что 48 не является наименьшее общее кратное.

    Скрыть ответ

    Вопрос № 3:

     
    Перечислите первые несколько чисел, кратных 3, 5 и 6, чтобы найти наименьшее общее кратное.

    LCM IS 15

    LCM — 30

    LCM — 18

    LCM — 75

    Показать ответ

    Ответ:

    Правильный ответ B: LCM — 30.
    . : 3, 6, 9, 12, 15, 18, 21, 24, 27, 30, 33, …
    Первые несколько кратных 5: 5, 10, 15, 20, 25, 30, 35, 40 , 45, 50, …
    Первые несколько кратных 6: 6, 12, 18, 24, 30, 36, 42, 48, 54, 60, …

    Как мы видели выше, 30 — это первое (наименьшее) число, которое является общим среди кратных чисел 3, 5 и 6, поэтому наименьшее общее кратное равно 30.

    Скрыть ответ


    У Кортни 54 конфеты, а у Триш 36. Они хотят приготовить мешочки с конфетами на день рождения своей подруги Ким, но в каждом мешочке должно быть одинаковое количество конфет. Чтобы в каждом пакете было как можно больше конфет, когда Кортни и Триш работают отдельно, сколько пакетов они могут сделать и сколько конфет будет в каждом пакете?

    10 пакетов по 9 конфет

    9 пакетов по 10 конфет

    15 пакетов по 6 конфет

    5 пакетов по 18 конфет

    Показать Ответ

    Ответ:

    Правильный ответ D: 5 пакетов по 18 конфет в каждом.

    Для начала перечислите простые множители чисел 54 и 36:
    \(54=2\times3\times3\times3\)
    \(36=2\times2\times3\times3\)

    Обратите внимание, что они оба разделяют двойка и две тройки. Произведение этих общих простых множителей равно \(2\times3\times3=18\).

    Перевод джипег в пдф онлайн в один файл: Конвертировать JPG в PDF — быстрый, онлайн, бесплатный

    конвертер jpg в pdf — конвертер пдф из джипег в пдф

    Зачем конвертер пдф из джипег в пдф?

    Предположим, вам нужно отправить группу изображений на утверждение, и вы предпочитаете отправлять их в виде одного файла PDF в «формате переносимого документа». Возможно, при преобразовании изображения в PDF необходимо учитывать общий размер файла. Что ж, какой бы ни была причина, наш бесплатный онлайн- фотографий конвертер пдф- лучший и быстрый способ конвертировать JPG в PDF.


    конвертер пдф из джипег в пдф (с сохранением исходного качества):

    Наш конвертер изображений в PDF имеет удобный интерфейс, который лучше всего подходит для преобразования файлов изображений JPG в формат PDF. Он точно сохраняет макет исходных изображений. Этот онлайн конвертировать джипег в пдф то же качество изображения после процесса преобразования без потери качества. Благодаря этим алгоритмам прецизионного онлайн-преобразования, которые обеспечивают высокое качество изображения!

    Как конвертировать JPG в PDF онлайн (шаг за шагом)?

    Процесс преобразования изображения в PDF становится удобным с помощью этого бесплатного конвертер JPG в PDF. Вот шаги, которые помогут вам безопасно сохранить JPG как PDF:

    Шаг 1:

    • Перетащите или загрузите файл .JPG в этот онлайн-конвертер конвертер пдф. Вы также можете добавить несколько файлов JPG, чтобы преобразовать их в файлы PDF.

    Шаг 2:

    • Теперь нажмите кнопку «Конвертировать» и подождите секунду, конвертер(конвертер пдф) превратит из джипег в пдф.

    Шаг 3:

    • Просто нажмите кнопку «Загрузить».

    Часто задаваемые вопросы:

    Как сохранить картинку в формате PDF на вашем компьютере?
    • Сначала откройте фото на компьютере
    • Затем перейдите в меню «Файл»> «Печать» или воспользуйтесь сочетанием клавиш Command + P.
    • Теперь появится диалоговое окно «Печать», просто выберите раскрывающееся меню PDF и выберите «Сохранить как PDF».
    • Наконец, выберите имя нового PDF-файла и нажмите Сохранить.

    Как конвертировать ​JPG в PDF на macOS?

    Чтобы начать преобразование, откройте предварительный просмотр

    • Зайдите в меню File, там вы увидите опцию Open, выберите ее.
    • Пришло время выбрать изображения JPG, которые нужно преобразовать в PDF.
    • Вы можете увидеть кнопку Открыть внизу окна, нажмите на нее
    • Еще раз нажмите на меню «Файл» и прямо сейчас выберите «Экспорт в PDF».
    • Теперь добавьте желаемое имя для вашего PDF-файла.
    • После этого выберите место для сохранения в раскрывающемся списке «Где».
    • Наконец, нажмите кнопку «Сохранить».

    Воспользуйтесь бесплатным онлайн-конвертером JPEG в PDF из нашего лучшего источника, который поможет вам сохранить формат JPG / JPEG (изображение) в PDF на macOS без искажения макета.

    Как преобразовать несколько файлов JPG в несколько файлов PDF?

    Чтобы преобразовать несколько файлов JPG в файлы PDF:

    • Вам просто нужно вставить изображения JPG в документ текстового процессора, а затем отправить документ в формат PDF в меню печати.

    Теперь стало удобно сохранять несколько изображений JPG в документы PDF с помощью конвертер jpg в pdf.

    Как преобразовать JPEG в PDF на моем телефоне?

    Используйте наш JPEG конвертер пдф, который является наиболее удобным для мобильных устройств, с помощью которого вы можете легко преобразовать неограниченное количество файлов Jконвертировать из джипег в пдф.


    Как конвертировать ​JPG в PDF на Android?

    Выполните следующие действия, чтобы выполнить такие преобразования на Android:

    • Сначала откройте Google Фото.
    • Нажмите на изображение JPG, которое вы хотите преобразовать.
    • Теперь нажмите на трехточечное меню «⋮»
    • Затем нажмите кнопку «Печать» в меню.
    • Затем выберите «Сохранить как PDF» в меню «Выбрать принтер».
    • Теперь нажмите на значок PDF(конвертер пдф)
    • Назовите преобразованный файл и нажмите «Сохранить».

    Совместное использование файла PDF, а не JPG, является идеальным, поскольку получатель с большей вероятностью сможет его открыть. Просто воспользуйтесь этим бесплатным онлайн конвертер JPG в PDF, который обеспечивает точное качество преобразования, он эффективно преобразует один или несколько файлов конвертировать джипег в пдф.

    Other Languages: JPG To PDF Converter, Ubah JPG ke PDF, JPG PDF Çevirme, Konwerter JPG na PDF, Převod JPG do PDF, JPG in PDF Umwandeln, Converter JPG em PDF, Convertir JPG en PDF, Convertir JPG a PDF, JPG PDF Konvertáló, Converti da JPG a PDF, JPG PDF 변환, JPG PDF 変換, Μετατροπη JPG σε PDF, แปลงไฟล์ JPG เป็น PDF, Chuyển JPG sang PDF

    Комбинатор PDF – Объединение PDF-файлов

    ЗАГРУЗИТЬ

    Перетащите файлы сюда.

    Файл PDF — это тип цифрового документа. В большинстве случаев PDF содержит текст, но он также может содержать активные ссылки и даже изображения. Иногда человек создает PDF на компьютере, но PDF также может быть сканом физических предметов, таких как бумажные документы, удостоверения личности и многое другое.

    Вы можете подумать, что PDF — это то же самое, что и DOC. Однако они очень разные. Файлы DOC обычно создаются в текстовом процессоре либо в Интернете, например, в Google Docs, либо в отдельной программе, такой как Microsoft Word. Если вы сохраните этот файл в формате DOC и передадите его кому-то другому, DOC, скорее всего, будет выглядеть совсем по-другому для второго человека. Это потому, что DOC связаны с установленными шрифтами и другими специфическими атрибутами операционной системы, поэтому невозможно сказать, как они будут выглядеть на других системах. У PDF нет такой проблемы: они выглядят одинаково на всех системах!

    Можно ли объединять PDF-файлы?

    Объединение PDF-файлов вполне возможно. Однако для этого обычно требуется программное обеспечение для редактирования PDF, например, Adobe Acrobat. Эти программы могут быть довольно дорогими.

    К счастью, вам не нужно покупать модное программное обеспечение для объединения PDF-файлов. Наш инструмент сделает все это за вас бесплатно, и вам даже не нужно ничего устанавливать.

    Наш бесплатный PDF-объединитель: как им пользоваться

    Наш бесплатный инструмент может объединить до 20 PDF-файлов за один раз. Он очень прост в использовании.

    Сначала перетащите файлы PDF в область с надписью “Перетащите файлы сюда”. Если вы используете мобильное устройство, вы можете просто нажать кнопку “ЗАГРУЗИТЬ”, а затем перейти к своим PDF-файлам.

    Теперь, когда файлы загружены, вам нужно убедиться, что они расположены в правильном порядке. В очереди вы увидите все ваши файлы, расположенные в виде сетки. Перемещайте файлы, пока они не будут расположены правильно. Первый PDF в списке будет первым PDF в объединенном файле. Второй PDF будет расположен под первым, и так далее.

    Если все выглядит нормально, нажмите кнопку “ОБЪЕДИНИТЬ ФАЙЛЫ”. Наша система объединит все PDF в один новый PDF. Загрузка нового PDF начнется автоматически. Если у вас более 20 PDF, не волнуйтесь. Вы можете объединить их все с помощью нескольких дополнительных шагов. После загрузки объединенного PDF вы можете загрузить его в очередь. Затем добавьте дополнительные PDF и снова объедините их. Вы можете делать это столько раз, сколько захотите.

    Обратите внимание, что если вы загрузите несколько файлов, вам нужно будет объединить их вместе и загрузить новый файл в течение часа. Если вы этого не сделаете, наша система отбросит загруженные файлы, и вам придется начинать все сначала.

    Безопасен ли PDFJoiner.com?

    Наш инструмент абсолютно безопасен в использовании. Вам не нужно беспокоиться о потере оригинальных PDF-файлов при их объединении. Наш инструмент копирует файлы из вашей системы, а затем объединяет эти копии. Ваши оригиналы в безопасности на вашем компьютере.

    Кроме того, наша система безопасна. Через 60 минут наш инструмент очищает все конвертации и загрузки. Это гарантирует, что ваша конфиденциальная информация не будет скомпрометирована.

    Преобразование JPEG в PDF онлайн

    • Дом
    • org/ListItem»> Индекс функциональности
    • Конвертер PDF: конвертируйте PDF в редактируемый
    • Конвертируйте любой формат в редактируемый PDF онлайн
    • Конвертировать Jpeg в PDF

    Формы заполнены

    Формы подписаны

    Формы отправлены

    Начать бесплатно

    Загрузите свой документ в редактор PDF

    Введите в любом месте или подпишите форму

    Печать, электронная почта, факс, или экспорт

    900 14 Попробуйте прямо сейчас! Редактировать pdf

    Пользователи доверяют управлению документами на платформе pdfFiller

    65,5 тыс. +

    документов, добавляемых ежедневно

    53%

    документов, созданных

    из шаблонов

    4M

    90 014 PDF-файлов, отредактированных в месяц

    238K

    общих документов

    в месяц месяц

    Универсальное программное обеспечение для работы с PDF

    Единая таблетка от всех головных болей, связанных с PDF. Редактируйте, заполняйте, подписывайте и делитесь — на любом устройстве.

    Начать бесплатную пробную версию

    Инструкции и помощь по преобразованию Jpeg в PDF для

    Преобразование JPEG в PDF: редактируйте PDF-файлы из любого места

    Формат PDF является одним из наиболее широко используемых форматов документов по разным причинам. С любого устройства можно обмениваться ими между настольными компьютерами и телефонами с разными экранами и настройками. PDF-файлы будут выглядеть одинаково, независимо от того, открываете ли вы их на Mac, Microsoft или на смартфонах.

    Безопасность данных — еще одна причина, по которой мы предпочитаем использовать PDF-файлы для хранения и обмена личной информацией и документами. Вот почему важно выбрать безопасный редактор, особенно при работе в Интернете. Используя онлайн-решение, можно отслеживать историю просмотров, чтобы узнать, кто имел к ней доступ раньше.

    pdfFiller — это онлайн-инструмент для создания и редактирования документов, который позволяет создавать, изменять, подписывать и отправлять PDF-файлы в одном окне браузера. Преобразуйте файл MS Word или электронную таблицу Google и начните редактировать их внешний вид и добавьте несколько заполняемых полей, чтобы сделать документ доступным для пения. Используйте готовый документ для личных нужд или поделитесь им с другими любым удобным способом — вы получите уведомление, когда человек откроет и заполнит форму.

    Используйте мощные функции редактирования для ввода текста, комментирования и выделения. Добавляйте и редактируйте визуальный контент. Изменить порядок страниц формы. Добавьте заполняемые поля и отправьте на подпись. Сотрудничайте с другими, чтобы заполнить поля. Когда документ будет готов, загрузите его на свое устройство или сохраните в облаке.

    Чтобы отредактировать документ, выполните следующие действия:

    01

    Начните с загрузки документа.

    02

    Чтобы изменить содержимое документа, щелкните вкладку «Инструменты» и следуйте инструкциям.

    03

    Чтобы вставить заполняемые поля, щелкните вкладку «Добавить заполняемые поля» справа и добавьте в них текст, подписи, изображения и многое другое.

    04

    По завершении нажмите Готово и перейдите к загрузке, отправке или печати документа.

    Как конвертировать JPEG в PDF

    Связанные функции

    В нынешнем виде на каждый вопрос типа «Нужен ли вам Adobe® Flash® для преобразования PDF?» Каждый веб-сайт должен задать этот вопрос и сделать его доступным для своих пользователей. Это все равно, что сказать, что если вы используете инструмент CDR только для преобразования одного видеоизображения в одно видео MP4, это все равно отличный инструмент для преобразования видео MP4 в видео H.264. В ближайшем будущем у нас есть действительно замечательные вещи. Получите доступ к огромному архиву изображений в формате JPEG. Скачивайте файлы JPEG быстро и легко Экономьте время с удобным интерфейсом приложения Скачивайте изображения в формате JPEG на рабочий стол, компьютер или куда угодно Переходите непосредственно к папкам с изображениями или копируйте папки с изображениями в другие папки. Скачивайте файлы JPEG онлайн, включая фото и видео Копируйте изображения на другие компьютеры или мобильные устройства Экономьте время с удобным интерфейсом приложения Форматы файлов PDF могут быть сложными в управлении по разным причинам, таким как нехватка места для хранения или трудности с поддержанием правильного межстрочного интервала между документами (как отображается на вашем мониторе). JPEG разработан, чтобы помочь вам выбрать и извлечь файлы изображений JPEG, а затем быстро преобразовать их в формат PDF. Кроме того, вы можете управлять и копировать изображения JPEG с любого компьютера, а также на мобильные устройства, не выходя из этого приложения. Получите доступ к обширному архиву изображений JPEG Самая большая коллекция файлов изображений JPEG доступна в формате JPEG бесплатно. Это идеальное хранилище для всех ваших потребностей в изображениях JPEG, включая фотографии, слайд-шоу, логотипы, изображения, видео и изображения, используемые в качестве фона в журналах. Скачивайте изображения в формате JPEG на рабочий стол, компьютер или куда угодно JPEG — интуитивно понятное приложение с удобным интерфейсом. Используйте его для удаления пустых страниц, объединения нескольких PDF-файлов в один или удаления всего документа. Обратите внимание, что на Mac формы PDF создаются и сохраняются в файлы PDF с помощью встроенного приложения для чтения PDF. Для наших последних примеров я объединил примеры 2 и 3 отдельной бумажной тетради — той же тетради, которую мы использовали раньше, только с большим количеством заметок и пустых страниц. Рисунок 1-4a: Использование сенсорной панели для удаления или объединения страниц Страницы этой записной книжки пусты, но с каждой стороны есть по одной странице. Страницы сгруппированы по годам, и мы видим, что они были подписаны «Джоном Доу». Страницы с номерами от 1 до 4 находятся на разных сторонах дневника, поэтому, когда мы объединяем страницы 1 и 2 для создания новой страницы, мы делаем ее первой страницей, подписанной Джоном Доу..

    Что говорят наши клиенты о pdfFiller

    Убедитесь сами, прочитав отзывы на самых популярных ресурсах:

    Елена С

    05.01.2017

    Администратор юридической практики

    28.01.2019 9001 5

    Получите мощный редактор PDF для своего Mac или ПК с Windows

    Установите настольное приложение, чтобы быстро редактировать PDF-файлы, создавать заполняемые формы и безопасно хранить документы в облаке.

    Редактируйте PDF-файлы и управляйте ими из любого места с помощью устройства iOS или Android

    Установите наше мобильное приложение и редактируйте PDF-файлы с помощью удостоенного наград набора инструментов, где бы вы ни находились.

    Получите редактор PDF в браузере Google Chrome

    Установите расширение pdfFiller для Google Chrome, чтобы заполнять и редактировать PDF-файлы прямо из результатов поиска.

    Загрузка из Интернет-магазина Chrome

    pdfFiller получает высшие оценки в нескольких категориях на G2

    Список дополнительных функций

    Часто задаваемые вопросы о pdfFiller

    Ниже приведен список наиболее частых вопросов клиентов.

    Если вы не можете найти ответ на свой вопрос, не стесняйтесь обращаться к нам.

    Как преобразовать файл из JPG в PDF?

    Перетащите изображения JPG, которые вы хотите объединить в один PDF-файл (или нажмите кнопку «Добавить файл»). При необходимости измените порядок файла. Нажмите кнопку «Преобразовать файл(ы)», чтобы преобразовать изображения JPG в PDF. Сохраните преобразованный файл, нажав кнопку «Загрузить PDF-файл».

    Как преобразовать JPG в PDF?

    Перетащите изображения JPG, которые вы хотите объединить в один PDF-файл (или нажмите кнопку «Добавить файл»). При необходимости измените порядок файла. Нажмите кнопку «Преобразовать файл(ы)», чтобы преобразовать изображения JPG в PDF. Сохраните преобразованный файл, нажав кнопку «Загрузить PDF-файл».

    Как добавить JPEG в PDF?

    Нажмите «Редактировать PDF» > «Добавить изображение», а затем выберите изображение JPG, которое вы хотите вставить в документ. Поместите изображение в файл PDF, а затем выберите параметр сохранения в меню «Файл»> «Сохранить», чтобы получить новый файл PDF на свой компьютер.

    Как объединить несколько файлов JPEG в один PDF?

    Выберите все изображения, которые вы хотите в своем PDF-файле, щелкните правой кнопкой мыши и выберите «Открыть с предварительным просмотром». На боковой панели предварительного просмотра перетащите изображения в том порядке, в котором они должны отображаться в вашем PDF-файле. Выберите/выделите все изображения, которые необходимо включить в документ PDF. В противном случае только одно изображение может оказаться в документе PDF.

    Как объединить файлы JPEG в один?

    Шаг 1: Запустите программное обеспечение и выберите режим слияния. Шаг 2: Выберите количество изображений и другие настройки. Шаг 3: Добавьте файлы JPG и перетащите их в интерфейс. Шаг 4: Сохраните режим и объедините JPEG. Шаг 5. Добавьте текст в объединенный файл (необязательно)

    Как поместить несколько изображений в PDF?

    Чтобы объединить изображения в файл PDF, сначала выберите все изображения в проводнике или на рабочем столе. Затем щелкните правой кнопкой мыши одно из выбранных изображений и нажмите «Печать». Появится окно Печать изображений. В раскрывающемся меню «Принтер» в левом верхнем углу выберите Microsoft Print to PDF.

    Рабочие процессы электронной подписи стали проще

    Подписывайте, отправляйте на подпись и отслеживайте документы в режиме реального времени с помощью signNow.

    Начать бесплатную пробную версию

    Бесплатный конвертер JPG в PDF онлайн

    Главная

    >

    JPG в PDF

    Преобразование изображений в файлы PDF за считанные секунды. Поддерживаются форматы JPG, JPEG, PNG, BMP и другие.

    Загрузить для просмотра всех функций

    Перетащите сюда изображение

    Выбрать файл изображенийВыбрать изображение

    Оценить JPG в PDF

    4.4 /5

    0 голосов

    Почему стоит выбрать WPS Office для преобразования изображений в PDF?

    • Бесплатное и высококачественное преобразование

      WPS PDF Converter обеспечивает преобразование высококачественных изображений JPG в PDF и несколько настроек вывода, помогающих преобразовывать изображения в высококачественные файлы PDF.

    • Совместимость с несколькими форматами изображений

      Онлайн-конвертер JPG в PDF — это универсальный инструмент, позволяющий добавлять изображения JPG, GIF, BMP и PNG и преобразовывать их в файлы PDF.

    • Высокая гибкость

      WPS PDF Converter позволяет выводить изображения по одному или в объединенном виде. Вы можете настроить размер страницы, ориентацию страницы и поля по своему усмотрению.

    Часто задаваемые вопросы о преобразовании изображений в PDF

    Как преобразовать изображения в файлы PDF онлайн?

    Как сохранить изображение JPEG в формате PDF без потери качества?

    Какой самый быстрый способ конвертировать изображения в файлы PDF?

    Узнайте больше о JPG в PDF

    Другие статьи >

    • Учебные пособия

      Как конвертировать PDF в JPG на Mac: 3 простых руководства

      PDF — это универсальный стандарт для отправки файлов, и это нередактируемые документы. Мы часто используем PDF для загрузки документа, но иногда веб-сайт или программное обеспечение не имеют формата файла для поддержки формата PDF. Вы можете преобразовать PDF в JPG, если ваш файл PDF не поддерживает его. Mac имеет несколько различных функций для преобразования pdf в jpg. Пользователи внимательно читают статью, потому что я показываю, как мы конвертируем PDF-файл в JPG на Mac. Используйте WPS Office для преобразования PDF в JPG WPS Office широко распространен…

    • Учебные пособия

      Как объединить JPG в PDF

      Иногда, когда у нас есть набор изображений JPG или JPEG и мы можем поделиться с другими, мы выбираем сжатие изображений в файл. Но что, если мы не хотим, чтобы эти фотографии были скопированы или даже украдены для коммерческого использования? Можем ли мы объединить изображения JPG в PDF для кросс-платформенного обмена и представления с защитой интеллектуальной собственности? Здесь мы представляем вам решение для слияния и объединения изображений JPG в файл PDF.

    Разложение тангенса в ряд тейлора: Parabola: Разложение тангенса.

    Котангенс в нуле в ряд тейлора : Анализ-I

    Правила форума

    В этом разделе нельзя создавать новые темы.

    Если Вы хотите задать новый вопрос, то не дописывайте его в существующую тему, а создайте новую в корневом разделе «Помогите решить/разобраться (М)».

    Если Вы зададите новый вопрос в существующей теме, то в случае нарушения оформления или других правил форума Ваше сообщение и все ответы на него могут быть удалены без предупреждения.

    Не ищите на этом форуме халяву, правила запрещают участникам публиковать готовые решения стандартных учебных задач. Автор вопроса обязан привести свои попытки решения и указать конкретные затруднения.

    Обязательно просмотрите тему Правила данного раздела, иначе Ваша тема может быть удалена или перемещена в Карантин, а Вы так и не узнаете, почему.

     
    never-sleep 

     Котангенс в нуле в ряд тейлора

    10. 01.2012, 14:52 

    27/11/11
    153

    Нужно разложить в ряд Тейлора в окрестности с точностью до

    В лоб по формуле Тейлора — не получается, ибо не определено

    Тут дело в том, что приравнивая коэффициенты при одинаковых степенях, мы получим при
    равенство

    Мне кажется, что это все связано с неопред. котангенса в нуле…

    Появилась идея разложить тангенс в нуле, а потом воспользоваться тем, что
    и раскладывать в Тейлора так, но мне не нравится эта затея…

    Может подскажете — как быть?)


       

                      

    Nemiroff 

     Re: Котангенс в нуле в ряд тейлора

    10. 01.2012, 14:58 

    Заслуженный участник

    20/07/09
    4026
    МФТИ ФУПМ

    Ну вообще, котангенс в нуле издалека напоминает бесконечность, так что странно ждать нормального значения у него при нулевой степени.
    Добавьте, что ли, еще минус первую степень икса в неопределенное разложение котангенса.


       

                      

    never-sleep 

     Re: Котангенс в нуле в ряд тейлора

    10. 01.2012, 15:00 

    27/11/11
    153

    Nemiroff в сообщении #525264 писал(а):

    Ну вообще, котангенс в нуле издалека напоминает бесконечность, так что странно ждать нормального значения у него при нулевой степени.
    Добавьте, что ли, еще минус первую степень икса в неопределенное разложение котангенса.

    Ок, спасибо! А как вы до этого догадались и как это можно обосновать, что мы должны добавлять отрицательные степени? А почему только минус первую? А может еще нужно минус вторую?
    Как в аналогичных примерах действовать)

    Есть ли еще подобные функции, где с такими подвохами нужно бороться?!


       

                      

    PAV 

     Re: Котангенс в нуле в ряд тейлора

    10. 01.2012, 15:02 

    Супермодератор

    29/07/05
    8248
    Москва

    Рассмотрите произведение — особенность в нуле исчезнет.


       

                      

    Nemiroff 

     Re: Котангенс в нуле в ряд тейлора

    10. 01.2012, 15:08 

    Заслуженный участник

    20/07/09
    4026
    МФТИ ФУПМ

    Разделил косинус на синус, вынес из синус одну степень икса и разложил знаменатель.


       

                      

    svv 

     Re: Котангенс в нуле в ряд тейлора

    10. 01.2012, 15:16 

    Заслуженный участник

    23/07/08
    9987
    Crna Gora

    never-sleep писал(а):

    Есть ли еще подобные функции, где с такими подвохами нужно бороться?!

    Нужно всё-таки иметь в виду, что исходное задание — разложить котангенс в ряд Тейлора в нуле — невыполнимо. Ни сама функция, ни её производные в этой точке не определены. Если Вам дано такое задание, Вы вправе просто объяснить его некорректность и больше ничего.

    Вообще, полезно помнить, что не любую функцию и не в любой точке можно разложить в ряд Тейлора.

    Все дальнейшие советы проистекают из стремления представить функцию в нуле хоть как-то, раз нельзя с помощью ряда Тейлора. Или — Тейлором, но несколько другую функцию (). Но это уже другие задачи.


       

                      

    never-sleep 

     Re: Котангенс в нуле в ряд тейлора

    10.01.2012, 18:21 

    27/11/11
    153

    PAV в сообщении #525269 писал(а):

    Рассмотрите произведение — особенность в нуле исчезнет.

    Ок, спасибо, понятно)

    — 10.01.2012, 18:25 —

    Nemiroff в сообщении #525273 писал(а):

    Разделил косинус на синус, вынес из синус одну степень икса и разложил знаменатель.

    Спасибо) Я понял так)

    Правильно?

    — 10.01.2012, 18:30 —

    svv в сообщении #525275 писал(а):

    Нужно всё-таки иметь в виду, что исходное задание — разложить котангенс в ряд Тейлора в нуле — невыполнимо. Ни сама функция, ни её производные в этой точке не определены. Если Вам дано такое задание, Вы вправе просто объяснить его некорректность и больше ничего.

    Вообще, полезно помнить, что не любую функцию и не в любой точке можно разложить в ряд Тейлора.

    Все дальнейшие советы проистекают из стремления представить функцию в нуле хоть как-то, раз нельзя с помощью ряда Тейлора. Или — Тейлором, но несколько другую функцию (). Но это уже другие задачи.

    Ок, спасибо, теперь понятнее. В ряде тейлора вроде как нет отрицательных степеней…видимо поэтому это разложение и нельзя назвать рядом тейлора.


       

                      

    SpBTimes 

     Re: Котангенс в нуле в ряд тейлора

    10.01.2012, 19:10 

    Заслуженный участник

    18/12/10
    1600
    spb

    Лоран, если хотите


       

                      

    Показать сообщения за: Все сообщения1 день7 дней2 недели1 месяц3 месяца6 месяцев1 год Поле сортировки АвторВремя размещенияЗаголовокпо возрастаниюпо убыванию 
      Страница 1 из 1
     [ Сообщений: 8 ] 

    Модераторы: Модераторы Математики, Супермодераторы


    Помогите решить / разобраться (М)

     
    denmanorwat 

     Разложить по Тейлору 1/cos(x)

    01. 10.2021, 17:56 

    31/07/20
    16

    Здравствуйте. Разбираюсь в разложении функции в ряд Тейлора в окрестности нуля. Автор предлагает следующий трюк: По Тейлору раскладываем косинус: . Представляем кусок разложения заменой переменной: . Далее, раскладываем по Тейлору не , а уже функцию с заменённой переменной: , после чего подставляем . Возникает вопрос: а почему мы можем пользоваться трюком с заменой переменной чтобы добиться разложения тангенса по Тейлору?


       

                      

    nnosipov 

     Re: Разложить по Тейлору 1/cos(x)

    01. 10.2021, 18:56 

    Заслуженный участник

    20/12/10
    8862

    denmanorwat в сообщении #1533521 писал(а):

    а почему мы можем пользоваться трюком с заменой переменной

    А почему нет? Что нам мешает заменить сложное выражение от одной буквой ? Мы же не собираемся (надеюсь) игнорировать о-малые. Да и при , так что все оценки валидны.

    — Пт окт 01, 2021 22:56:45 —

    denmanorwat в сообщении #1533521 писал(а):

    разложения тангенса

    Наверное, секанса.


       

                      

    novichok2018 

     Re: Разложить по Тейлору 1/cos(x)

    01. 10.2021, 21:48 

    Заблокирован

    16/04/18

    1129

    Это же какие то Бернулли?


       

                      

    denmanorwat 

     Re: Разложить по Тейлору 1/cos(x)

    01.10.2021, 22:45 

    31/07/20
    16

    nnosipov в сообщении #1533535 писал(а):

    denmanorwat в сообщении #1533521 писал(а):

    а почему мы можем пользоваться трюком с заменой переменной

    А почему нет? Что нам мешает заменить сложное выражение от одной буквой ? Мы же не собираемся (надеюсь) игнорировать о-малые. Да и при , так что все оценки валидны.

    — Пт окт 01, 2021 22:56:45 —

    denmanorwat в сообщении #1533521 писал(а):

    разложения тангенса

    Наверное, секанса.

    Спасибо. Прочитал ваше сообщение, подумал, и да, действительно, ничего не мешает. И поэтому о-малые игнорировать таки не стоит. Спасибо за ответ.
    P.S. Да, секанса, опечатка.


       

                      

    Nemiroff 

     Re: Разложить по Тейлору 1/cos(x)

    01.10.2021, 22:52 

    Заслуженный участник

    20/07/09
    4026
    МФТИ ФУПМ

    denmanorwat в сообщении #1533604 писал(а):

    И поэтому о-малые игнорировать таки не стоит.


    .
    Заменим всюду сперва на , затем на .
    Получим и
    Первое ещё куда ни шло: при подстановке получаем .
    Второе не столь позитивно — .


       

                      

    nnosipov 

     Re: Разложить по Тейлору 1/cos(x)

    02.10.2021, 04:24 

    Заслуженный участник

    20/12/10
    8862

    novichok2018 в сообщении #1533585 писал(а):

    Это же какие то Бернулли?

    Числа Эйлера, см.

    https://en.wikipedia.org/wiki/Trigonome … _expansion


       

                      

    thething 

     Re: Разложить по Тейлору 1/cos(x)

    03.10.2021, 05:16 

    Заслуженный участник

    27/12/17
    1381
    Антарктика

    denmanorwat в сообщении #1533521 писал(а):

    Возникает вопрос: а почему мы можем пользоваться трюком с заменой переменной чтобы добиться разложения тангенса по Тейлору?

    У нас есть разложение для . Далее, замечаем, что выражение в какой-то окрестности нуля меньше единицы (ибо вообще стремится к нулю), так что подстановка оправдана. Ну а возможность перегруппировки слагаемых вытекает из абсолютной сходимости и теорем о повторных рядах. Не понятно только, причём тут о-малые, когда речь о рядах

    Тейлора? Строгое доказательство можете посмотреть во втором томе Фихтенгольца. Пункт 446 так и называется: подстановка ряда в ряд.


       

                      

    Показать сообщения за: Все сообщения1 день7 дней2 недели1 месяц3 месяца6 месяцев1 год Поле сортировки АвторВремя размещенияЗаголовокпо возрастаниюпо убыванию 
      Страница 1 из 1
     [ Сообщений: 7 ] 

    Модераторы: Модераторы Математики, Супермодераторы


    {n-1} a_k\ a_{n-1-k}&n>0\\ \end{случаи} $$ то есть последовательность $$(a_n)_{n\in\mathbb{N}}=\left(\frac 11,\frac 13, \frac 2{15}, \frac {17}{315}, \frac { 62}{2835}, \frac{1382}{155925},\cdots\right)$$

    $\endgroup$

    1

    $\begingroup$

    Этот ответ основан на ответе coffeemath. 7) \end{выравнивание} Отсюда можно вывести линейную систему \начать{выравнивать} \begin{pматрица} 1 и 0 и 0 и 0 и 0 и 0&0\\ 0 и 1 и 0 и 0 и 0 и 0 &0\\ -1/2 &0 &1 & 0& 0& 0&0\\ 0 и -1/2 и 0 и 1 и 0 и 0&0 \\ 1/24 и 0 и -1/2 и 0 и 1 и 0&0\\ 0 и 1/24 и 0 и -1/2 и 0 и 1 и 0\\ 1/720 и 0 и 1/24 и 0 и -1/2 и 0 и 1 \end{pматрица} \begin{pматрица} а\\б\\в\\г\\е\\е\\г \end{pматрица} «=» \begin{pmatrix} 0\1\0\-1/6\0\1/120\0 \end{pматрица} \end{выравнивание} Что имеет решение \начать{выравнивать} \begin{pматрица} а\\б\\в\\г\\е\\е\\г \end{pматрица} = \begin{pmatrix} 0\1\0\1/3\0\2/15 \end{pматрица} \end{выравнивание} [Обратите внимание, что вы можете извлечь две линейные системы из приведенной выше. Затем вам нужно найти только нечетные степени $x$, поскольку $\tan(x)$ нечетно.] 97) \end{выравнивание} Что (на удивление) правильно, поскольку мы можем сравнить наше решение с WolframAlpha.

    $\endgroup$

    3

    $\begingroup$

    Если вы можете использовать известные ряды для $\sin(x)$ и $\cos(x)$, составьте уравнение $(\cos x)*(\tan x) = \sin x$, где вы подставите неизвестные константы как коэффициенты ряда $\tan x$.

    18 в 7 степени: § Возведение в степень онлайн. Калькулятор «Возведение в степень»

    Human Papillomavirus высокого канцерогенного риска (16, 18, 31, 33, 35, 39, 45, 51, 52, 56, 58, 59, 66, 68 типы), ДНК количественно, скрининг с определением возможности интеграции вируса в геном [реал-тайм ПЦР]

    Исследование используется для диагностики папиллома-вирусной инфекции, с одновременным определением количества вирусной ДНК в исследуемом материале. Анализ позволяет сделать вывод о степени поражения слизистой, оценить эффективность терапии в процессе лечения. Определяется суммарное количество ДНК 14 типов ВПЧ высокого канцерогенного риска (ВКР) с отдельным определением ДНК ВПЧ 16-го, 18-го и 45-го генотипов. Тест также позволяет оценить вероятность интеграции ВПЧ 16-го, 18-го и 45-го типа в геном человека (что является опасным и прогностически неблагоприятным фактором).

    Синонимы русские

    Папиллома-вирусы высокого канцерогенного риска, вирусы папилломы высокого канцерогенного риска, ВПЧ высокого канцерогенного риска.

    Синонимы английские

    Human Papillomavirus, DNA; HPV, Viral Load.

    Метод исследования

    Полимеразная цепная реакция в режиме реального времени.

    Какой биоматериал можно использовать для исследования?

    Соскоб урогенитальный.

    Как правильно подготовиться к исследованию?

    • Женщинам исследование рекомендуется производить до менструации или через 2-3 дня после её окончания.
    • Мужчинам не мочиться в течение 3 часов до взятия урогенитального мазка.

    Общая информация об исследовании

    Вирусы папилломы человека (ВПЧ) представляют собой широко распространённую и вариабельную группу вирусов, поражающих эпителий кожи и слизистых оболочек и обладающих онкогенным потенциалом. ВПЧ передаётся при тесном контакте с инфицированным эпителием, поэтому основные пути заражения – половой и контактно-бытовой. Возможна передача ВПЧ от инфицированной матери к плоду.

    К факторам, провоцирующим развитие ВПЧ-инфекции, относятся ранее начало половой жизни, большое количество половых партнеров, сниженный иммунитет, применение оральных контрацептивов, авитаминозы, инфекции, передаваемые половым путём, курение и проживание в крупных городах.

    Инкубационный период может длиться от 2 месяцев до 2-10 лет. Характерно скрытое течение заболевания, при котором отсутствуют клинические проявления, а при кольпоскопическом, цитологическом и гистологическом обследовании выявляется норма. В 30 % случаев в течение 6-12 месяцев может произойти избавление от вируса. Диагностика скрытой ВПЧ-инфекции осуществляется только методом ПЦР.

    ВПЧ может по-разному воздействовать на эпителий: способствовать возникновению доброкачественных образований (папиллом, кондилом) или объединению ДНК папиллома-вируса с геном клетки, что приводит к дисплазии (неоплазии) и в дальнейшем к раку (чаще всего в переходной зоне шейки матки).

    К папиллома-вирусам высокого онкогенного риска относятся ВПЧ типов 16, 18, 31, 33, 35, 39, 45, 51, 52, 56, 58, 59, 66, 68. 

    Для выявления возбудителя используется метод полимеразной цепной реакции, которая позволяет обнаружить ДНК вируса. Принцип метода основан на многократном увеличении числа копий специфичного для данного возбудителя участка ДНК.

    Полимеразная цепная реакция (ПЦР) – это метод молекулярной диагностики, позволяющий выявлять генетический материал возбудителя на ранних этапах заболевания. Он характеризуется высокими показателями диагностической чувствительности и специфичности, быстротой получения конечного результата и доступностью. Особенностью метода является возможность выявлять ДНК вируса даже при малом содержании её в исследуемом биологическом материале. Метод основан на многократном увеличении числа копий специфичного для данного возбудителя участка ДНК. Для диагностики может использоваться разнообразный биологический материал, взятый у детей, взрослых лиц, людей с патологией иммунной системы, беременных женщин. Метод позволяет определять количество молекул ДНК исследуемого вируса, что является важным в диагностике острой формы инфекции, реактивации персистентной инфекции, носительстве вируса, а также при контроле эффективности проводимой противовирусной терапии.

    По данным исследований, количественное содержание вируса в материале коррелирует со степенью неоплазии: чем выше вирусная нагрузка, тем выраженнее цитологические изменения в эпителии. При обследовании необходимо учитывать генотип вируса, наличие и степень цитологических и гистологических изменений в тканях, увеличение или снижение вирусной нагрузки через несколько месяцев (6 месяцев и более) после предыдущего обследования.

    В исследовании проводится определение суммарного количества ДНК 14 типов ВПЧ высокого канцерогенного риска (ВКР) с отдельным определением ДНК ВПЧ 16, 18 и 45 генотипов. ВПЧ ВКР являются главной этиологической причиной развития РШМ и предшествующей ему тяжелой дисплазии. ВПЧ обнаруживается в 95 % случаев РШМ, из них 16, 18, 45 генотипы в совокупности являются причиной 75 % плоскоклеточных раков и 94 % аденокарцином шейки матки.

    Известно, что развитие РШМ часто ассоциировано с интеграцией ДНК вируса в геном клетки-хозяина. Наиболее часто интегрируют ВПЧ 16 и 18 генотипов, при этом происходит разрыв участка Е1/Е2 при сохранении онкогена Е6/Е7. В тест-системе, на которой выполняется данное исследование, по разным каналам прибора детектируются разные участки генома ВПЧ 16, 18 и 45 типов. Выявление области Е6 при отсутствии области Е1/Е2 позволяет косвенно судить о возможности интеграции вируса в геном человека. В случае обнаружения возможности интеграции вируса в геном в бланк ответа выводится соответствующая информация.

    Для чего используется исследование?

    • Чтобы оценить риск развития новообразований, ассоциированных с ВПЧ (рак шейки матки, рак прямой кишки, рак аногенитальной области и других локализаций).
    • Для контроля за эффективностью лечения предраковых заболеваний, ассоциированных с ВПЧ.
    • Для мониторинга папиллома-вирусной инфекции и прогнозирования ее течения.

    Когда назначается исследование?

    • При выявлении цитологических изменений в мазке на атипию, в мазке по Папаниколау, в гистологическом препарате.
    • При кондиломах и других морфологических изменениях половых путей.
    • При скрининге рака шейки матки у женщин старше 25-30 лет (в качестве дополнительного исследования).
    • При наблюдении за инфицированными ВПЧ.
    • При лечении рака и предраковых состояний, ассоциированных с ВПЧ.

    Что означают результаты?

    Уровень вирусной нагрузки интерпретируется с учетом результатов цитологического исследования мазков, гистологических изменений в биоптате и генотипа вируса, изменении его количества с течением времени.

    • Количество ДНК ВПЧ не определяется при отсутствии вируса в исследуемом образце или его минимальном количестве (ниже детектируемого уровня) – риск развития патологического процесса, связанного с ВПЧ, минимальный.
    • Клинически малозначимая концентрация вируса (менее 103 копий ДНК ВПЧ на 105 клеток) – минимальный риск развития дисплазии, транзиторное течение вирусного процесса.
    • Клинически значимая концентрация вируса (более 103 копий ДНК ВПЧ на 105 клеток) – хроническая инфекция с высоким риском развития дисплазии и РШМ.
    • Более 105 копий ДНК ВПЧ на 105 клеток при установленном факте персистентного течения инфекции (ВПЧ выявляется более 1 года) – усиленная вирусная нагрузка, ассоциированная с повышенным риском тяжелой дисплазии, часто встречается при РШМ.
    • Снижение вирусной нагрузки в 10 раз за 6 месяцев – транзиторная инфекция.
    • Рост вирусной нагрузки через 6 и более месяцев после лечения указывает на возможность рецидива.

    Важные замечания

    • Инфицирование ВПЧ не всегда приводит к раку шейки матки.
    • Возможно одновременное заражение несколькими генотипами ВПЧ.
    • Результат анализа должен интерпретироваться с учетом заключений цитологического и гистологического исследований.

    Также рекомендуется

    • Human Papillomavirus 16/18 (HPV 16/18), ДНК [реал-тайм ПЦР]
    • Human Papillomavirus 6/11 (HPV 6/11), ДНК [реал-тайм ПЦР]
    • Human Papillomavirus высокого канцерогенного риска (16, 18, 31, 33, 35, 39, 45, 51, 52, 56, 58, 59 типы), ДНК генотипирование [реал-тайм ПЦР]
    • Цитологическое исследование мазков (соскобов) с поверхности шейки матки (наружного маточного зева) и цервикального канала – окрашивание по Папаниколау (Рар-тест)
    • Цитологическое исследование мазков (соскобов) с поверхности шейки матки (наружного маточного зева) и цервикального канала на атипию
    • Антиген плоскоклеточной карциномы (SCCA)

    Кто назначает исследование?

    Акушер-гинеколог, дерматовенеролог, уролог, инфекционист, проктолог, педиатр, ЛОР.

    Литература

    • Arbyn M. et al. (2010). «European Guidelines for Quality Assurance in Cervical Cancer Screening. Second Edition – Summary Document». Annals of Oncology 21 (3): 448 – 458.
    • Hsiu-Ting Tsai, Ching-Hu Wu, Hsiao-Ling Lai, et al. Association between Quantitative High-Risk Human Papillomavirus DNA Load and Cervical Intraepithelial Neoplasm Risk Cancer Epidemiol Biomarkers Prev 2005;14:2544-2549.
    • Saslow D, Solomon D, Lawson HW, et al. American Cancer Society, American Society for Colposcopy and Cervical Pathology, and American Society for Clinical Pathology Screening Guidelines for the Prevention and Early Detection of Cervical Cancer. Am J Clin Pathol. 2012;137:516-542.
    • «Genital HPV Infection – CDC Fact Sheet». Centers for Disease Control and Prevention (CDC). April 10, 2008. Retrieved 13 November 2009.
    • Материалы и рекомендации Противоракового Общества России. Режим доступа: http://www.pror.ru/

    Вирус папилломы человека (ВПЧ)

    Вирус папилломы человека (ВПЧ) — одна из самых распространённых инфекций, передающихся половым путём и при контакте «кожа-к-коже». Нет точной статистики, сколько людей в мире заражены ВПЧ, потому что у большинства заражённых вирус никак себя не проявляет.

    Анализы

    НК вируса папилломы (HPV) 16/18 типов

    3-4 дня

    от 215 ₽

    В корзину

    НК вируса папилломы человека — скрининг (16, 18, 31, 33, 35, 39, 45, 51, 52, 56, 58, 59 типы)

    3-4 дня

    от 450 ₽

    В корзину

    НК вируса папилломы человека — генотипирование (16, 18, 31, 33, 35, 39, 45, 51, 52, 56, 58, 59 типы)

    3-4 дня

    от 1 000 ₽

    В корзину

    Биологи считают, что около 80 % населения земли — носители ВПЧ.

    Опасность вируса в том, что некоторые его типы могут привести к онкологии. 98 % рака шейки матки (по данным учёных США — 100 %) связаны с ВПЧ. Вирус провоцирует рак влагалища, вульвы, анального канала, мужских половых органов, горла, рта. Причём смертельно опасное заболевание может развиться спустя годы, десятилетия после заражения, если возникнут благоприятные для этого условия.

    Всего в группу ВПЧ входит более 170 вирусов и штаммов, из них примерно 40 передаются половым путём, а 13 способны вызвать рак.

    Заражение отдельными типами ВПЧ — при наличии высокого иммунитета — проходит незаметно, бесследно для организма. Другие штаммы заставляют клетки кожи усиленно делиться, в итоге появляются бородавки, папилломы, кондиломы («венерические бородавки»). Третьи типы, интегрируя в ДНК человека, действуют как онкогены и способствуют злокачественной трансформации клеток, росту опухолей.

    Это важно! ВПЧ не является достаточным фактором для развития онкологического заболевания. Но он — один из важнейших онкофаторов, «провокатор» озлокачествления клеток.

    Чтобы началось перерождение здоровой ткани в раковую, необходимо сочетание нескольких условий, в их числе — сбои в работе иммунной системы. Именно иммунитет — главный защитник в том числе и от увеличения вирусной нагрузки ВПЧ. Очень важно периодически сдавать анализа на ВПЧ, чтобы предотвратить опасность, вызванную вирусом. Так, можно вылечить предраковое состояние шейки матки. Другие онкологии, связанные с ВПЧ, лучше поддаются лечению, если оно начинается на ранней стадии заболевания и контролируется периодическими исследованиями на снижение/повышение вирусной нагрузки.

    ВПЧ: способы заражения, симптомы, онкогенные типы вируса


    Основные способы заражения:
    • вагинальный, оральный, анальный секс с человеком, заражённым ВПЧ;
    • при контактах «кожа-к-коже» с человеком, заражённым папилломавирусом, или контакте с поверхностями, которых касался заражённый человек — бытовое заражение возможно, если на коже имеются порезы, ссадины, другие повреждения;
    • при родах — от матери к ребёнку.

    Последние исследования американских и европейских учёных показали, что велика вероятность заражения ВПЧ в медицинских учреждениях — при переливании крови, использовании медицинского оборудования, вдыхании вирусных частиц, например — при лазерной абляции или электрокоагуляции кондилом.

    Сложность борьбы с неполовыми формами заражениями в том, что вирус чрезвычайно устойчив к большинству дезинфицирующих средств. ВПЧ — первый вирус, оказавшийся нечувствительным к инактивации (обработке) глутаровым альдегидом (средством для стерилизации хирургических инструментов, требующих абсолютной чистоты). Перед врачами и технологами встала проблема обеззараживания приборов, которые нельзя автоклавировать и подвергать воздействию агрессивных химических соединений.


    Симптомы и типы вируса папилломы человека 

    Симптомы заражения различаются в зависимости от типа ВПЧ. Некоторые типы, например — ВПЧ5, сохраняются в организме человека без клинических симптомов и могут быть обнаружены только специальными исследованиями. Штаммы ВПЧ1, 2, 4, 7, 22, 63 вызывают образование бородавок на руках, ногах, подошвах.

    Типы 6, 11, 42, 44 способны вызвать развитие генитальных бородавок, папилломатоза гортани; 6, 16, 18, 31 и другие — анальной дисплазии; 60 — вирусной кисты.

    К раку половых органов способны привести штаммы 26, 53, 66. Штаммы с высоким онкориском — 33, 35, 39, 51, 52, 56, 58, 59. Самым высоким риском трансформации клеток в злокачественные обладают типы 16, 18, 31, 45.

    Анализы на ВПЧ у женщин и мужчин. Когда назначаются?

    Анализы на ВПЧ нужно пройти каждому взрослому человеку самостоятельно, без назначения, т. к. вероятность, что вы заражены — 8 из 10. Направление на анализ обычно выдаётся дерматологом, урологом, гинекологом при наличии характерных внешних признаков или заболеваний, причиной которых может стать вирус.

    Анализы на ВПЧ сдают при планировании беременности, при выявлении причин и лечении бесплодия, патологий беременности и вынашивания. В этом случае анализы сдают оба партнёра.

    Факторами риска и поводом для сдачи анализа у женщин также являются:
    • ранняя половая жизнь;
    • отношения с разными, иногда сразу несколькими половыми партнёрами;
    • общие хронические, гинекологические заболевания, патологии;
    • слабый иммунитет.
    Факторами риска и поводом для сдачи анализа у мужчин также являются:
    • множественные половые контакты;
    • половые контакты с женщинами, заражёнными ВПЧ;
    • плохая гигиена;
    • сужение крайней плоти;
    • слабый иммунитет.

    Методы диагностики ВПЧ

    Кольпоскопическое исследование

    Кольпоскопия — осмотр с помощью кольпоскопа влагалищной части шейки матки, входа и стенок влагалища. Это простой, недорогой, но высокоинформативный метод диагностирования заболеваний шейки матки. Клиническое значение имеет расширенная кольпоскопия с применением нескольких тестов — с 3-процентной уксусной кислотой, йодным раствором Люголя. Тесты выявляют различные типы эпителия, позволяют оценить размеры и качество патологических образований (при их наличии), сосудистый рисунок, качество шеечных желёз.

    Во время кольпоскопии проводят прицельную биопсию с наиболее атипично изменённых участков.

    Цитологическое исследование

    Задача цитологического исследования шеечных мазков (тест Папаниколау, пап-тест) — выявление специфических для ВПЧ-инфекции клеток — койлоцитов и дискератоцитов.

    Подтверждением папилломовирусной инфекции считается обнаружение койлоцитов, трансэпителиальной лимфоцитарной инфильтрации, базально-клеточной гиперплазии в биоптате (биоматериале, взятом на исследование).

    Пап-тест обязателен для:

    • женщин после 30 лет;
    • женщин, у кого был ранее диагностирован ВПЧ;
    • женщин, у кого во время кольпоскопии обнаружили зоны с изменённым эпителием.

    По результатам пап-теста определяют класс опасности для здоровья женщины: 1—2 класс — без подозрения на рак, 3 класс — подозрение на онкологию, 4—5 класс — наличие раковых клеток в малом или большом количестве.

    К недостаткам цитологического исследования относят сложность исполнения, высокие квалификационные требования к врачу-цитологу. Потому проходить исследование нужно в диагностических центрах и лабораториях, персонал которых постоянно подтверждает свой профессионализм.

    Гистологическое исследование

    Гистологический метод обнаружения ВПЧ можно было бы считать золотым стандартом диагностики вируса, однако мешает его высокая стоимость, невозможность частого проведения и не всегда точный прицельный забор биоптата из шейки матки. Для проведения гистологической диагностики также требуются специалисты очень высокой квалификации.

    Поэтому гистологическое исследование биоптата часто служит дополнением к пап-анализу. Оно позволяет оценить состояние клеток, степень поражения, определить, чем является новообразование — опухолью или кондиломой.

    ПЦР диагностика папилломавируса 

    Полимеразная цепная реакция (ПЦР) относится к высокоразрешающим технологиям детекции нуклеиновых кислот. Современные ПЦР тест-системы обладают высокой чувствительностью, используются не только для выявления ВПЧ, но и вирусной нагрузки на организм (количественный показатель заражённости) главных клинически значимых генотипов (16, 18, 31, 33, 35, 39, 45, 51, 52, 56, 58, 59), которые ответственны за почти 94 % случаев тяжёлых цервикальных дисплазий и рака шейки матки. ПЦР тест-системы используют также для обнаружения штаммов ВПЧ 6 и 11.

    Такие возможности тест-системы позволяют прогнозировать течение ВПЧ-инфекции, оценивать эффективность терапии. Установлено, что папилломавирусная инфекция имеет дозозависимый эффект: чем выше концентрация ДНК вируса в исследуемом материале, тем выше риск развития неоплазии и раковой опухоли.

    В результатах теста указывают концентрацию ВПЧ:

    • Lg < 3 — папилломавирус обнаружен в клинически незначимом количестве;
    • Lg 3–5 — папилломавирус обнаружен в клинически значимом количестве;
    • Lg > 5 — папилломавирус обнаружен в высокой концентрации, вирусная нагрузка на организм высокая.

    Виды исследуемого материала, правила забора:

    • для пап-теста — шеечный мазок;
    • для цитологического исследования — биоптат, взятый прицельно с атипично изменённых участков слизистой.
    • для ПЦР-теста — соскоб клеток слизистых оболочек генитального тракта.

    Мазок у женщин берут из цервикального канала, у мужчин — из уретры. Для мазка используют мягкую щётку или ватный тампон. Их бережно вводят в канал, затем осторожно вынимают, вращая. На поверхности щётки/тампона остаются эпителиальные клетки, нужные для исследования.

    Как подготовиться к анализу на ВПЧ?


    Взятие биоматериала не проводится во время менструации, за 5 дней до её начала и в течение 5 дней по окончании. Нельзя проходить исследование, если есть воспалительные процессы.

    За 2 суток до забора биоматериала женщинам и мужчинам нужно воздержаться от сексуальных контактов. Женщинам нельзя в течение 48 часов использовать вагинальные кремы, лекарства, суппозитории, спринцевания, тампоны, вместо ванны нужно принимать душ. Мазок берут до проведения любых гинекологических манипуляций или через 2 суток после них.

    Если биоматериал берут из уретры, то от последнего мочеиспускания до забора биоматериала должно пройти не менее 90 минут.

    При заборе биоптата для гистологического исследования соблюдаются те же правила, что и при подготовке к забору мазка. 

    После биопсии в течение 2—3 недель нельзя:

    вступать в половые контакты,

    • испытывать значительные физические нагрузки,
    • перегреваться (баня, сауна, жаркая погода),
    • купаться в открытом водоёме или бассейне,
    • принимать препараты, разжижающие кровь;
    • использовать вагинальные средства.

    Если возникнет кровотечение, следует пользоваться только прокладками (не тампонами!)

    На точность результата могут влиять антибиотики, пробиотики, местные антисептики, которые вы принимали/применяли даже 2 месяца назад. Перед сбором материала нужно рассказать врачу обо всех лекарственных средствах, которыми вы пользовались или пользуетесь.

    Стоимость исследования на наличие папилломавирусной инфекции в АО «СЗДЦМ»

    Стоимость исследования на заражение ВПЧ зависит от вида исследования, охвата штаммов вируса, определения типа/без определения, расчёта вирусной нагрузки.

    Цены на виды исследований в медицинских подразделениях АО «СЗДЦМ» представляют собой разнообразные комбинации по охватности, подробности, прогностической ценности.

    Какой способ исследования выбрать, вам подскажет врач — гинеколог, дерматолог, венеролог. Если вы сдаёте анализы по собственной инициативе, выберите исследование на наличие онкогенных штаммов ВПЧ.

    Где сдать анализы на ВПЧ?

    Анализы на ВПЧ вы можете сдать в медицинских подразделениях АО «СЗДЦМ», расположенных в Санкт-Петербурге, Ленинградской области, в Великом Новгороде, Старой Руссе и других городах.

    Чтобы найти ближайший к вам пункт, воспользуйтесь интерактивной картой или перечнем медицинских учреждений АО «СЗДЦМ».

    Во всех наших отделениях — терминалах и центрах — вас встретят внимательные, опытные специалисты с высокой квалификацией. Медицинские учреждения АО «СЗДЦМ» оснащены современным оборудованием, лабораторными материалами, одноразовыми инструментами и расходными материалами.

    Мы гарантируем вам точность исследований, заботливое отношение, полную конфиденциальность ваших личных данных и результатов обследования.

    Будьте здоровы! А для этого вовремя и регулярно проходите важные обследования в АО «СЗДЦМ».
    Мы поможем вам сохранить и вернуть здоровье! 

    Сколько будет 5 в 7-й степени?

    Итак, вы хотите знать, сколько будет 5 в 7-й степени, не так ли? В этой статье мы объясним, как именно выполнить математическую операцию под названием «возведение числа 5 в степень 7». Это может показаться фантастическим, но мы объясним это без жаргона! Давай сделаем это.

    Что такое возведение в степень?

    Давайте сначала зафиксируем наши термины, а затем посмотрим, как вычислить число 5 в 7-й степени.

    Когда мы говорим об возведении в степень, все, что мы на самом деле имеем в виду, это то, что мы умножаем число, которое мы называем 9) для обозначения показателя степени. Знак вставки полезен в ситуациях, когда вы не хотите или не нуждаетесь в использовании надстрочного индекса.

    Итак, мы упомянули, что возведение в степень означает умножение базового числа само на себя для получения показателя степени число раз. Давайте посмотрим на это более наглядно:

    5 в 7-й степени = 5 x … x 5 (7 раз)

    Итак, каков ответ?

    Теперь, когда мы объяснили теорию, лежащую в основе этого, давайте поработаем над числами и выясним, чему равно 5 в 7-й степени:

    5 в степени 7 = 5 7 = 78 125

    Почему мы вообще используем возведение в степень 5 7 ? Что ж, нам намного проще писать умножения и выполнять математические операции как с большими, так и с маленькими числами, когда вы работаете с числами с большим количеством конечных нулей или большим количеством десятичных знаков.

    Надеюсь, эта статья помогла вам понять, как и почему мы используем возведение в степень, и дала вам ответ, который вы изначально искали. Теперь, когда вы знаете, что такое 5 в 7-й степени, вы можете продолжить свой веселый путь.

    Не стесняйтесь поделиться этой статьей с другом, если вы считаете, что она поможет ему, или перейдите вниз, чтобы найти еще несколько примеров.

    Процитируйте, дайте ссылку или ссылку на эту страницу

    Если вы нашли этот контент полезным в своем исследовании, пожалуйста, сделайте нам большую услугу и используйте приведенный ниже инструмент, чтобы убедиться, что вы правильно ссылаетесь на нас, где бы вы его ни использовали. Мы очень ценим вашу поддержку!

    • Сколько будет 5 в 7-й степени?

    • «Сколько будет 5 в 7-й степени?». VisualFractions.com . По состоянию на 27 апреля 2023 г. http://visualfractions.com/calculator/exponent/what-is-5-to-the-7th-power/.

    • «Сколько будет 5 в 7-й степени?». VisualFractions.com , http://visualfractions. com/calculator/exponent/what-is-5-to-the-7th-power/. По состоянию на 27 апреля 2023 г.

    • Сколько будет 5 в 7-й степени?. VisualFractions.com. Получено с http://visualfractions.com/calculator/exponent/what-is-5-to-the-7th-power/.

    Калькулятор возведения в степень

    Хотите найти решение еще одной задачи? Введите число и мощность ниже и нажмите «Рассчитать».

    Вычисление возведения в степень

    Случайный список примеров возведения в степень

    Если вы добрались до этого места, вам должно быть ДЕЙСТВИТЕЛЬНО нравится возведение в степень! Вот несколько случайных вычислений:

    Сколько будет 48 в 52-й степени?

    Сколько будет 24 в 63-й степени?

    Сколько будет 14 в 45-й степени?

    Сколько будет 38 в 33-й степени?

    Сколько будет 60 в 33-й степени?

    Сколько будет 12 в 13-й степени?

    Сколько будет 74 в 37-й степени?

    Сколько будет 26 в 8-й степени?

    Сколько будет 63 в 51-й степени?

    Сколько будет 88 в 30-й степени?

    Сколько будет 42 в 97-й степени?

    Сколько будет 57 в 9-й степени?

    Сколько будет 34 в 49-й степени?

    Сколько будет 89 в 59-й степени?

    Сколько будет 31 в 96-я сила?

    Сколько будет 88 в 89-й степени?

    Сколько будет 26 в 94-й степени?

    Сколько будет 54 в 17-й степени?

    Сколько будет 60 в 32-й степени?

    Сколько будет 95 в 60-й степени?

    Сколько будет 48 в 73-й степени?

    Сколько будет 26 в 60-й степени?

    Сколько будет 22 в 35-й степени?

    Сколько будет 66 в 84-й степени?

    Сколько будет 45 в 37-й степени?

    Сколько будет 48 в 18-й степени?

    Сколько будет 43 в 57-й степени?

    Сколько будет 42 в 63-й степени?

    Сколько будет 57 в 80-й степени?

    Сколько будет 45 в 14-й степени?

    Сколько будет 19 в 36-й степени?

    Сколько будет 34 в 77-й степени?

    Сколько будет 4 в 54-й степени?

    Сколько будет 44 в 31-й степени?

    Сколько будет 41 в 38-й степени?

    Сколько будет 5 в 74-й степени?

    Сколько будет 50 в 66-й степени?

    Сколько будет 87 в 49-й степени?

    Сколько 50 к 79сила?

    Сколько будет 25 в 84-й степени?

    Сколько будет 51 в 64-й степени?

    Сколько будет 57 в 63-й степени?

    Сколько будет 99 в 29-й степени?

    Сколько будет 395 в 3-й степени?

    Сколько будет 100 в 77-й степени?

    Сколько будет 70 в 47-й степени?

    Сколько будет 37 в 48-й степени?

    Сколько будет 60 в 47-й степени?

    Сколько будет 69 в 87-й степени?

    Сколько будет 17 в 41 степени?

    Сколько будет 47 в 21-й степени?

    Сколько будет 59 в 81 степени?

    Сколько будет 68 в 30-й степени?

    Сколько будет 21 в 62-й степени?

    Сколько будет 71 в 73-й степени?

    Сколько будет 34 в 79-й степени?

    Сколько будет 57 в 30-й степени?

    Сколько будет 26 в 36-й степени?

    Сколько будет 85 в 24-й степени?

    Сколько будет 3 в 39-й степени?

    Сколько будет 17 в 14-й степени?

    Сколько будет 99 в 98-й степени?

    Сколько 32 к 29сила?

    Сколько будет 99 в 45-й степени?

    Сколько будет 20 в 25-й степени?

    Что такое 570392 во 2-й степени?

    Сколько будет 11 в 26-й степени?

    Сколько будет 81 в 39-й степени?

    Сколько будет 41 в 68-й степени?

    Сколько будет 78 в 68-й степени?

    Сколько будет 2 в 4-й степени?

    Сколько будет 26 в 8-й степени?

    Сколько будет 13 в 80-й степени?

    Сколько будет 84 в 81-й степени?

    Что такое 9к 69-й державе?

    Сколько будет 95 в 24-й степени?

    Сколько будет 67 в 44-й степени?

    Сколько будет 93 в 34-й степени?

    Сколько будет 81 в 14-й степени?

    Сколько будет 14 в 95-й степени?

    Сколько будет 31 в 10-й степени?

    Сколько будет 48 в 83-й степени?

    Сколько будет 94 в 63-й степени?

    Сколько будет 100 в 91-й степени?

    Сколько будет 58 в 75-й степени?

    Сколько будет 49 в 47-й степени?

    Сколько будет 35 в 46-й степени?

    Сколько будет 56 в 26-й степени?

    Сколько будет 62 в 34-й степени?

    Сколько будет 85 в 45-й степени?

    Сколько будет 44 в 48-й степени?

    Сколько будет 90 в 7-й степени?

    Сколько будет 33 в 20-й степени?

    Сколько будет 68 в 44-й степени?

    Сколько будет 26 в 85-й степени?

    Сколько будет 36 в 17-й степени?

    Сколько будет 17 в 23-й степени?

    Сколько будет 97 в 45-й степени?

    Сколько будет 44 в 42 степени?

    Сколько будет 100 в 78-й степени?

    Сколько будет 64 в 12-й степени?

    Калькулятор и конвертер научных представлений

    9029 1
    Научное представление Десятичное число
    Миллион в экспоненциальном представлении 1 × 10 6 1 000 000
    10 Миллион в научном выражении обозначение 1 × 10 7 10 000 000
    100 миллионов в экспоненциальном представлении 1 × 10 8 100 000 ,000
    Миллиард в экспоненциальном представлении 1 × 10 9 1 000 000 000
    10 Миллиарды в экспоненциальном представлении 1 × 10 10 10 000 000 000
    100 Миллиарды в экспоненциальном представлении 1 × 10 900 20 11 100 000 000 000
    Триллион в экспоненциальном представлении 1 × 10 12 1 000 000 000 000
    10 Триллион в экспоненциальном представлении 1 × 10 13 10 000 000 000 000
    1 00 триллионов в научных нотация 1 × 10 14 100 000 000 000 000

    Экспоненциальная запись: сложение и вычитание

    Научное обозначение: умножение и деление

    В каждой области науки приходится иметь дело с числами, числа могут быть в любом формате, например, вес земли в килограммах — это очень большое число или масса электрон — очень маленькое число. Представление этих типов чисел непросто.

    Предположим, что имеется число с 22 нулями, записанное таким образом:

    602200000000000000000000000

    Это число нелегко прочитать или понять. Итак, существует способ, известный как научная запись, которую мы можем использовать для записи очень больших или малых чисел более простым и понятным способом.

    • Десятичная в научной записи
    • Десятичная в научной записи

     

    Важность научной записи

    В 1998 году НАСА запустило орбитальный аппарат для поиска данных об изменении климата на Марсе, но через три года после выхода на орбиту э заблудился и расследование на этот вопрос показывает, что оценка данных была неправильной из-за того, что две команды передают свои данные в разных единицах измерения. Представление числа в стандартной форме очень важно, потому что точность чисел имеет значение, читая слишком большое или слишком маленькое число, мы можем ошибиться при подсчете нулей.

     

    Как представить числа в экспоненциальном представлении?

    Числа записываются в произведение так, как если бы первое число было мантисса, а второе число было бы степенью 10:

    Любое число = мантисса x 10 большое количество их научное обозначение будет таким:

    7 000 000 000 000 000 000 000 = 7 × 10 21

    Здесь 7 — мантисса, а 21 — показатель степени. Теперь мы видим, что при чтении и записи этих типов чисел вероятность ошибки уменьшается, и мы можем легко использовать эти числа в любой письменной работе.

    Прежде чем использовать научные обозначения, давайте найдем лежащую в их основе теорию. Как мы можем найти различные степени для 10?

    • 10 0 = 1
    • 10¹ = 10
    • 10³ = 1000

    Здесь 10 степень помогает найти нули, например, когда степень равна 0, нуля нет следуйте за 1, когда мощность равна 3, есть 3 нули следуют за 1. Аналогично 10 100 = 1000….0 представляют 100 нулей. Это очень большое число, и его нелегко прочитать, поэтому с помощью научных обозначений представление очень большого числа становится легким.

    Чтобы выразить числа в экспоненциальном представлении, слева от десятичной точки должна оставаться только одна цифра, для этого десятичная точка перемещается влево или вправо в зависимости от числа, больше или меньше нуля . Умножение на другой показатель степени позволяет нам перемещать десятичные разряды.

     

    Например, в

    9,2867 × 10 4 = 92876,00

     

    показатель степени 10 положителен, он позволяет десятичной дроби двигаться вправо. В то время как, с другой стороны, отрицательная экспонента сдвигает десятичную дробь влево что мы должны оставить всего одна цифра слева от запятой.

    Еще одно преимущество научного представления заключается в том, что мы можем легко складывать, вычитать, умножать и делить большие числа. Давайте посмотрим, как мы можем использовать этот метод для сложения чисел.

     

    Для сложения или вычитания чисел в экспоненциальном представлении используются следующие шаги:

    • показатели степени 10 должны оставаться одинаковыми в обоих терминах.
    • Уравняв степени 10, мы можем складывать или вычитать числа.
    • Наконец, мы должны скорректировать ответ в соответствии с экспоненциальной записью.

     

    Добавить экспоненциальное представление: (5,7 × 10 4 ) + (2,25 × 10 5 )
    1. ponent в зависимости от малой мощности.

    5,7 × 10 4 ) + (2,25 × 10 1 × 10 4 )

    1. Теперь сгруппируйте числа
    9 0002 (5,7 + 2,25 × 10 1 ) × 10 4
    (5,7 + 22,5) × 10 4
    28,2 × 10 4

    1. Теперь скорректируйте число в соответствии со стандартной формой научного представления 003

       

      Аналогичным образом мы можем вычесть две величины .

      Вычесть экспоненциальное представление: (6,67 × 10 8 ) – (8,4 × 10 6 )
      1. Отрегулируйте степень большой экспоненты в соответствии с малой степенью.

      (6,67 × 10 2 × 10 6 ) – (8,4 × 10 6 )

      1. Теперь сгруппируйте числа

      (667 – 8,4) × 10 6 905 44 658,6 × 10 6

      1. Теперь отрегулируйте число в соответствии с к стандартной форме экспоненциального представления

      (6,58 × 10 2 ) × 10 6 = 6,58 × 10 8

        900 03

      Все числа в экспоненциальном представлении имеют основание 10, поэтому мы можем легко умножать и разделить их:

      1. Умножаем и делим два числа, умножаем или делим их мантиссы, складываем и вычитаем степени показателей соответственно.
      2. В обоих случаях мы должны преобразовать число в стандартную форму научного представления.

    Sin 2 x 2 cos x 0: Mathway | Популярные задачи

    Mathway | Популярные задачи

    1Найти точное значениеsin(30)
    2Найти точное значениеsin(45)
    3Найти точное значениеsin(30 град. )
    4Найти точное значениеsin(60 град. )
    5Найти точное значениеtan(30 град. )
    6Найти точное значениеarcsin(-1)
    7Найти точное значениеsin(pi/6)
    8Найти точное значениеcos(pi/4)
    9Найти точное значениеsin(45 град. )
    10Найти точное значениеsin(pi/3)
    11Найти точное значениеarctan(-1)
    12Найти точное значениеcos(45 град. )
    13Найти точное значениеcos(30 град. )
    14Найти точное значениеtan(60)
    15Найти точное значениеcsc(45 град. )
    16Найти точное значениеtan(60 град. )
    17Найти точное значениеsec(30 град. )
    18Найти точное значениеcos(60 град. )
    19Найти точное значениеcos(150)
    20Найти точное значениеsin(60)
    21Найти точное значениеcos(pi/2)
    22Найти точное значениеtan(45 град. )
    23Найти точное значениеarctan(- квадратный корень из 3)
    24Найти точное значениеcsc(60 град. )
    25Найти точное значениеsec(45 град. )
    26Найти точное значениеcsc(30 град. )
    27Найти точное значениеsin(0)
    28Найти точное значениеsin(120)
    29Найти точное значениеcos(90)
    30Преобразовать из радианов в градусыpi/3
    31Найти точное значениеtan(30)
    32Преобразовать из градусов в радианы45
    33Найти точное значениеcos(45)
    34Упроститьsin(theta)^2+cos(theta)^2
    35Преобразовать из радианов в градусыpi/6
    36Найти точное значениеcot(30 град. )
    37Найти точное значениеarccos(-1)
    38Найти точное значениеarctan(0)
    39Найти точное значениеcot(60 град. )
    40Преобразовать из градусов в радианы30
    41Преобразовать из радианов в градусы(2pi)/3
    42Найти точное значениеsin((5pi)/3)
    43Найти точное значениеsin((3pi)/4)
    44Найти точное значениеtan(pi/2)
    45Найти точное значениеsin(300)
    46Найти точное значениеcos(30)
    47Найти точное значениеcos(60)
    48Найти точное значениеcos(0)
    49Найти точное значениеcos(135)
    50Найти точное значениеcos((5pi)/3)
    51Найти точное значениеcos(210)
    52Найти точное значениеsec(60 град. )
    53Найти точное значениеsin(300 град. )
    54Преобразовать из градусов в радианы135
    55Преобразовать из градусов в радианы150
    56Преобразовать из радианов в градусы(5pi)/6
    57Преобразовать из радианов в градусы(5pi)/3
    58Преобразовать из градусов в радианы89 град.
    59Преобразовать из градусов в радианы60
    60Найти точное значениеsin(135 град. )
    61Найти точное значениеsin(150)
    62Найти точное значениеsin(240 град. )
    63Найти точное значениеcot(45 град. )
    64Преобразовать из радианов в градусы(5pi)/4
    65Найти точное значениеsin(225)
    66Найти точное значениеsin(240)
    67Найти точное значениеcos(150 град. )
    68Найти точное значениеtan(45)
    69Вычислитьsin(30 град. )
    70Найти точное значениеsec(0)
    71Найти точное значениеcos((5pi)/6)
    72Найти точное значениеcsc(30)
    73Найти точное значениеarcsin(( квадратный корень из 2)/2)
    74Найти точное значениеtan((5pi)/3)
    75Найти точное значениеtan(0)
    76Вычислитьsin(60 град. )
    77Найти точное значениеarctan(-( квадратный корень из 3)/3)
    78Преобразовать из радианов в градусы(3pi)/4
    79Найти точное значениеsin((7pi)/4)
    80Найти точное значениеarcsin(-1/2)
    81Найти точное значениеsin((4pi)/3)
    82Найти точное значениеcsc(45)
    83Упроститьarctan( квадратный корень из 3)
    84Найти точное значениеsin(135)
    85Найти точное значениеsin(105)
    86Найти точное значениеsin(150 град. )
    87Найти точное значениеsin((2pi)/3)
    88Найти точное значениеtan((2pi)/3)
    89Преобразовать из радианов в градусыpi/4
    90Найти точное значениеsin(pi/2)
    91Найти точное значениеsec(45)
    92Найти точное значениеcos((5pi)/4)
    93Найти точное значениеcos((7pi)/6)
    94Найти точное значениеarcsin(0)
    95Найти точное значениеsin(120 град. )
    96Найти точное значениеtan((7pi)/6)
    97Найти точное значениеcos(270)
    98Найти точное значениеsin((7pi)/6)
    99Найти точное значениеarcsin(-( квадратный корень из 2)/2)
    100Преобразовать из градусов в радианы88 град.

    Mathway | Популярные задачи

    1Найти точное значениеsin(30)
    2Найти точное значениеsin(45)
    3Найти точное значениеsin(30 град. )
    4Найти точное значениеsin(60 град. )
    5Найти точное значениеtan(30 град. )
    6Найти точное значениеarcsin(-1)
    7Найти точное значениеsin(pi/6)
    8Найти точное значениеcos(pi/4)
    9Найти точное значениеsin(45 град. )
    10Найти точное значениеsin(pi/3)
    11Найти точное значениеarctan(-1)
    12Найти точное значениеcos(45 град. )
    13Найти точное значениеcos(30 град. )
    14Найти точное значениеtan(60)
    15Найти точное значениеcsc(45 град. )
    16Найти точное значениеtan(60 град. )
    17Найти точное значениеsec(30 град. )
    18Найти точное значениеcos(60 град. )
    19Найти точное значениеcos(150)
    20Найти точное значениеsin(60)
    21Найти точное значениеcos(pi/2)
    22Найти точное значениеtan(45 град. )
    23Найти точное значениеarctan(- квадратный корень из 3)
    24Найти точное значениеcsc(60 град. )
    25Найти точное значениеsec(45 град. )
    26Найти точное значениеcsc(30 град. )
    27Найти точное значениеsin(0)
    28Найти точное значениеsin(120)
    29Найти точное значениеcos(90)
    30Преобразовать из радианов в градусыpi/3
    31Найти точное значениеtan(30)
    32Преобразовать из градусов в радианы45
    33Найти точное значениеcos(45)
    34Упроститьsin(theta)^2+cos(theta)^2
    35Преобразовать из радианов в градусыpi/6
    36Найти точное значениеcot(30 град. )
    37Найти точное значениеarccos(-1)
    38Найти точное значениеarctan(0)
    39Найти точное значениеcot(60 град. )
    40Преобразовать из градусов в радианы30
    41Преобразовать из радианов в градусы(2pi)/3
    42Найти точное значениеsin((5pi)/3)
    43Найти точное значениеsin((3pi)/4)
    44Найти точное значениеtan(pi/2)
    45Найти точное значениеsin(300)
    46Найти точное значениеcos(30)
    47Найти точное значениеcos(60)
    48Найти точное значениеcos(0)
    49Найти точное значениеcos(135)
    50Найти точное значениеcos((5pi)/3)
    51Найти точное значениеcos(210)
    52Найти точное значениеsec(60 град. )
    53Найти точное значениеsin(300 град. )
    54Преобразовать из градусов в радианы135
    55Преобразовать из градусов в радианы150
    56Преобразовать из радианов в градусы(5pi)/6
    57Преобразовать из радианов в градусы(5pi)/3
    58Преобразовать из градусов в радианы89 град.
    59Преобразовать из градусов в радианы60
    60Найти точное значениеsin(135 град. )
    61Найти точное значениеsin(150)
    62Найти точное значениеsin(240 град. )
    63Найти точное значениеcot(45 град. )
    64Преобразовать из радианов в градусы(5pi)/4
    65Найти точное значениеsin(225)
    66Найти точное значениеsin(240)
    67Найти точное значениеcos(150 град. )
    68Найти точное значениеtan(45)
    69Вычислитьsin(30 град. )
    70Найти точное значениеsec(0)
    71Найти точное значениеcos((5pi)/6)
    72Найти точное значениеcsc(30)
    73Найти точное значениеarcsin(( квадратный корень из 2)/2)
    74Найти точное значениеtan((5pi)/3)
    75Найти точное значениеtan(0)
    76Вычислитьsin(60 град. )
    77Найти точное значениеarctan(-( квадратный корень из 3)/3)
    78Преобразовать из радианов в градусы(3pi)/4
    79Найти точное значениеsin((7pi)/4)
    80Найти точное значениеarcsin(-1/2)
    81Найти точное значениеsin((4pi)/3)
    82Найти точное значениеcsc(45)
    83Упроститьarctan( квадратный корень из 3)
    84Найти точное значениеsin(135)
    85Найти точное значениеsin(105)
    86Найти точное значениеsin(150 град. )
    87Найти точное значениеsin((2pi)/3)
    88Найти точное значениеtan((2pi)/3)
    89Преобразовать из радианов в градусыpi/4
    90Найти точное значениеsin(pi/2)
    91Найти точное значениеsec(45)
    92Найти точное значениеcos((5pi)/4)
    93Найти точное значениеcos((7pi)/6)
    94Найти точное значениеarcsin(0)
    95Найти точное значениеsin(120 град. )
    96Найти точное значениеtan((7pi)/6)
    97Найти точное значениеcos(270)
    98Найти точное значениеsin((7pi)/6)
    99Найти точное значениеarcsin(-( квадратный корень из 2)/2)
    100Преобразовать из градусов в радианы88 град.
    3 6 Решить для ? cos(x)=1/2 7 Найти x sin(x)=-1/2 8 Преобразование градусов в радианы 225 9 Решить для ? cos(x)=(квадратный корень из 2)/2 10 Найти x cos(x)=(квадратный корень из 3)/2 11 Найти x sin(x)=(квадратный корень из 3)/2 92=9 14 Преобразование градусов в радианы 120 градусов 15 Преобразование градусов в радианы 180 16 Найти точное значение желтовато-коричневый(195) 92-4 38 Найти точное значение грех(255) 39 Оценить лог база 27 из 36 40 Преобразовать из радианов в градусы 2 шт. 92-3sin(x)+1=0 43 Найти x tan(x)+ квадратный корень из 3=0 44 Найти x sin(2x)+cos(x)=0 45 Упростить (1-cos(x))(1+cos(x)) 92=25 59 График f(x)=- натуральный логарифм x-1+3 60 Найдите значение с помощью единичного круга угловой синус(-1/2) 61 Найти домен квадратный корень из 36-4x^2 92=0 66 Найти x cos(2x)=(квадратный корень из 2)/2 67 График у=3 68 График f(x)=- логарифмическая база 3 x-1+3 92 71 Найти x квадратный корень из x+4+ квадратный корень из x-1=5 72 Решить для ? cos(2x)=-1/2 73 Найти x логарифмическая база x из 16=4 9х 75 Упростить (cos(x))/(1-sin(x))+(1-sin(x))/(cos(x)) 76 Упростить сек(х)sin(х) 77 Упростить кубический корень из 24 кубический корень из 18 92=0 96 Найти x 3x+2=(5x-11)/(8г) 97 Решить для ? sin(2x)=-1/2 98 Найти x (2x-1)/(x+2)=4/5 92=0

    .

     

    Подписаться І 3

    Подробнее

    Отчет

    2 ответа от опытных наставников

    Лучший Новейшие Самый старый

    Автор: ЛучшиеНовыеСамыеСтарые

    Парвиз Ф. ответил 02.09.13

    Репетитор

    4,8 (4) 92 +2CosX -3 =0

       

          (CosX +3)( CosX -1 ) =0

      

           CosX =1 — единственный ответ.

                 X = 0  , X= 2n(360)

       

     

    Голосовать за 0 Понизить

    Подробнее

    Отчет

    Дебби Д. ответил 22.04.13

    92 -2y +1 = (y — 1)(y — 1), поэтому замените y на cos (x), и вы получите

    (cos (x) — 1 )(cox (x) — 1 ) = 0  на ноль свойство продукта, cos (x) — 1 = 0,

    , поэтому cos (x) = 1, а угол, косинус которого равен 1, равен 0,

    , поэтому x = 0 (плюс любое число, кратное 360 градусам или 2 пи радианы)

    Голосовать за 0 Понизить

    Подробнее

    Отчет

    Все еще ищете помощи? Получите правильный ответ, быстро.

    Юзгу тестирование: Электронная информационно-образовательная среда ЮЗГУ (версия 2.0) Информационный портал ЮЗГУ.

    Электронная информационно-образовательная среда ЮЗГУ. Учебные курсы ЮЗГУ

    Перейти к основному содержанию

    Пропустить новости сайта


    Пропустить Навигация Пропустить Вход Пропустить Основное меню Пропустить Ссылки

    Расписание занятий студентов

    Научная библиотека

    Электронный каталог Научной библиотеки ЮЗГУ

    Пропустить Календарь

    мая 2023

    1 2 3 4 5 6 7
    8 9 10 11 12 13 14
    15 16 17 18 19 20 21
    22 23 24 25 26 27 28
    29 30 31    
    Пропустить Последние действия

    Действия с Четверг, 18 мая 2023, 14:01

    Полный отчет о последних действиях

    Со времени Вашего последнего входа ничего не произошло

    Пропустить Счетчики Версия сайта для слабовидящих

    Заказать онлайн-тест ЮЗГУ (Юго-Западный государственный университет)

    Как мы выполняем онлайн-тест ЮЗГУ

    Оформить заявку

    Гарантии заказа онлайн-теста ЮЗГУ

    • На протяжении выбранного вами гарантийного срока вы можете вернуть потраченные деньги, если качество написанной под заказ студенческой работы вас не устроит или не будет соответствовать требованиям учебного заведения.

    • Перед сдачей клиенту, каждая работа проходит тщательную проверку на плагиат по выбранному вами сервису.

    • Написанием работ занимаются только опытные авторы, которые хорошо знают тонкости учебной дисциплины и имеют практический опыт в отрасли.

    • Мы строго храним тайну сотрудничества между агентством по написанию студенческих работ и клиентом.

    Отзывы наших клиентов

    Узнайте почему выбирают студенты:

    • Хочу выразить благодарность за поддержку сотрудникам компании ФастФайн. Когда у меня возникли проблемы с написанием бакалаврской работы, мне сразу нашли автора, который был со мной на связи все время, даже после сдачи работы. Считаю, что профессионалам FastFine можно доверять написание любых работ, здесь «своих» не бросают.

    • Хотя бы раз в семестр обращался в компанию ФастФайн. Не обошелся без помощи профессионалов и при написании дипломной работы. Тема специфическая, касается литературы 18 века. Не хватало времени и сил поработать с информационными источниками. Автор оперативно и недорого подготовил теоретический раздел. Получил «отлично» на защите, за что искренне благодарен этой конторе. Всем будущим выпускника рекомендую!

    • Мне нужно было срочно заполнить сайт уникальными статьями. Нашла у конкурентов подходящий контент и решила заказать рерайт. Компанию fastfine нашла через поисковик, почитала отзывы. Цены и сроки меня устроили, решила попробовать. Если честно, даже не ожидала, что можно переписать уже готовый текст своими словами и получить такой высокий процент уникальности. Видно, что это авторская работа, даже стиль написания немного отличается от исходника. На мой взгляд, статьи получились намного лучше оригинальных, такие не стыдно выкладывать. Автору ставлю отличную оценку! Тем, кого интересует рерайт высокого качества, рекомендую сайт fastfine.ru.

    Как сдать тест онлайн

    Теперь каждый студент ЮЗГУ и школьник знает, что такое дистанционное обучение. Проверку знаний преподаватели тоже часто проводят удаленно. Самый распространенный вариант такой проверки — это тестирование.  

    Проблема в том, что, если раньше ответы можно было подсмотреть у товарища, то теперь списать фактически невозможно. Даже если вы знаете ответы, вас может подвести слабая связь с интернетом или проблемы с компьютером. В общем, теперь тестирование проходит сложнее. Чтобы не рисковать успеваемостью, лучше купить тест онлайн.

    Выполнение тестов онлайн: главные проблемы

    Многие студенты ЮЗГУ наивно полагают, что смогут списать, вбив вопрос в поисковике. Но не все так просто. Их ждут трудности:

    • Они не находят нужный ответ. В интернете легко наткнуться на ложную информацию. Многие данные быстро устаревают.
    • Они не успевают найти ответ. Времени не много, а ответ на вопрос чаще всего спрятан где-то в длинных статьях.
    • Они сталкиваются с мошенниками, которые просят отправить СМС за правильный ответ.
    • Преподаватели замечают, что они списывают.

    Любой вариант грозит двойкой за тест. А от него может зависеть общая оценка по дисциплине, зачет, а может это и вовсе часть экзамена. Поэтому решение тестов онлайн на заказ — это отличный способ перестраховаться. Обеспечьте себя хорошей оценкой и не рискуйте успеваемостью.

    Как это работает

    Помощь с тестами онлайн в ЮЗГУ проходит в несколько этапов:

    • Вы пишите нам и предоставляете доступ к личному кабинету, где проходит тестирование.
    • В согласованное время специалист нашей компании подключается к вам и решает задания.
    • Вы получаете положительную оценку.

    И не нужно просиживать долгие часы за подготовкой, зубрить ответы и переживать. С нашей поддержкой сессия пройдет весело и без проблем.

    Заказ тестов онлайн

    Стоимость онлайн-тестов зависит от исполнителя. Многие компании завышают цену, делают ее неподъемной для студентов. Мы же устанавливаем адекватные тарифы, потому ценим своих клиентов. Кроме этого, мы гарантируем безопасность:

    • Мы никуда не пропадем в день теста. Если мы обещали, мы сделаем.
    • Мы предоставим в помощники специалистов, которые ориентируются в теме и без труда решат тест.
    • Мы справимся с любой дисциплиной и темой.

    Сотрудничество с FastFine поможет вам без проблем решить проблемы с учебой. С нами экзамены и тесты покажутся вам несложными.

    По вузам

    • КРСУ
    • ЛГУ
    • ЛГПУ
    • ЛГТУ
    • ЛЭГИ
    • Литературный институт имени А. М. Горького

    Бесконтактный георадар: результаты, области применения и ограничения испытаний в США

    NASA/ADS

    Бесконтактный георадар: результаты, области применения и ограничения испытаний в США

    • Доусон, К.
    • ;
    • Лейн, Дж. В., младший
    Аннотация

    Хотя возможности и грузоподъемность систем малых незанятых летательных аппаратов (БАС) и их интеграция в науку о Земле развиваются, адаптация традиционных геофизических инструментов для развертывания БАС идет медленно. Геологическая служба США (USGS) собирала данные с помощью sUAS для выполнения нашей научной миссии в широком спектре геолого-геофизических исследований, в том числе уделяя приоритетное внимание разработке набора инструментов, обеспечивающих полностью бесконтактное измерение потока с помощью sUAS. Здесь мы представляем результаты продолжающихся USGS испытаний бесконтактного георадара (GPR) для гидрологических и гидрогеофизических исследований.

    В 2017 году Геологическая служба США начала сотрудничество по разработке и тестированию бесконтактных систем георадара с воздушной связью, которые будут развернуты над целевой областью исследования. Полевые испытания на нескольких участках в Соединенных Штатах были сосредоточены на использовании бесконтактного георадара для оценки площади поперечного сечения пресноводных водотоков; затем эти измерения можно использовать в сочетании с данными о скорости потока для оценки расхода потока. Прототипы георадара, испытанные для батиметрического и приповерхностного картографирования, развертывались по канатной дороге, мосту и (или) БПЛА, как правило, на расстоянии от 1 до 20 метров над поверхностью земли или воды.

    Тестирование показало, что бесконтактный георадар, развернутый с помощью sUAS и других методов, может успешно измерять батиметрию потока и отображать неглубокие слои при определенных геофизических условиях. Тестирование также показало, что необходимо решить определенные проблемы сбора и обработки данных, а также системные ограничения, прежде чем Геологическая служба США сможет внедрить широкое оперативное использование бесконтактного георадара. Геологическая служба США также разрабатывает инструменты поддержки принятия решений, чтобы информировать негеофизиков о понимании гидрологических и геологических условий, подходящих для бесконтактного развертывания георадара, и связанных с этим ограничений метода, основанных на опыте батиметрического картирования, приповерхностного изображения и глубины снежного покрова. опросы.


    Публикация:

    Тезисы осенней встречи AGU

    Дата публикации:
    Декабрь 2020
    Биб-код:
    2020AGUFMNH0280006D «/>
    Ключевые слова:
    • 3394 Приборы и техника;
    • АТМОСФЕРНЫЕ ПРОЦЕССЫ;
    • 1920 Новые информационные технологии;
    • ИНФОРМАТИКА;
    • 4314 Математическое и компьютерное моделирование;
    • ПРИРОДНЫЕ ОПАСНОСТИ;
    • 4262 Системы наблюдения за океаном;
    • ОКЕАНОГРАФИЯ: ОБЩИЕ СВЕДЕНИЯ

    Исследование коренной породы мышьяка и урана: часто задаваемые вопросы по мышьяку и урану

    Часто задаваемые вопросы о мышьяке и уране в воде.

    В 2009 году Геологическая служба США (USGS) исследовала присутствие мышьяка и урана в колодцах с питьевой водой в восточном Массачусетсе. Район исследования был выбран потому, что записи свидетельствуют о высоких концентрациях мышьяка в этом районе. Эти вопросы и ответы касаются некоторых распространенных проблем, связанных с естественным присутствием этих элементов в колодцах коренных пород.

    Пропустить оглавление

    Содержание

    Вы пропустили раздел оглавления.

    Для чего проводились испытания частных скважин и кто проводил это исследование?

    Благодаря лучшему пониманию воздействия мышьяка и урана на здоровье человека Агентство по охране окружающей среды США (EPA) установило стандарты питьевой воды на содержание мышьяка и урана. Однако без тестирования может быть трудно определить, в каких частных колодцах может присутствовать мышьяк или уран в количествах, превышающих стандарты. Исследование Геологической службы США (USGS) было проведено для определения геологических характеристик, которые могут помочь определить области с более высокой вероятностью загрязнения мышьяком или ураном. Обратите внимание, что в исследовании изучались только уровни мышьяка и урана природного происхождения, а не уровни мышьяка и урана, возникающие в результате выбросов этих химических веществ в результате промышленных или других искусственных процессов.

    В 2003 году Агентство по охране окружающей среды США установило стандарт питьевой воды 30 частей на миллиард (млрд) для урана в системе питьевого водоснабжения. В 2001 году EPA изменило стандарт содержания мышьяка в питьевой воде с 50 частей на миллиард до 10 частей на миллиард; этот стандарт вступил в силу в 2006 году.

    Цель исследования Геологической службы США, начатого в 2008 году, состояла в том, чтобы определить, существует ли корреляция между присутствием мышьяка и урана в скважинах, пробуренных в коренных породах, и типом коренных пород. Эта информация будет использоваться для руководства будущей деятельностью по разведке общественного водоснабжения. Дополнительным преимуществом исследования является то, что эта информация может быть использована для определения того, находится ли частная скважина для водоснабжения в районе с более высокой или низкой вероятностью превышения любого из этих стандартов.

    Исследование включало лабораторный анализ образцов частных колодцев, собранных 478 владельцами частных колодцев, расположенных в 116 населенных пунктах в центральном и северо-восточном штате Массачусетс. Район исследования был выбран на основе обзора существующих общедоступных записей о скважинах коренных пород, которые указывают на более высокие концентрации мышьяка в этом районе. Меньше известно о распределении урана в колодезной воде. Исследование включало анализ урана, чтобы лучше понять его возможную корреляцию с типами коренных пород в Массачусетсе. Геологическая служба США и Департамент охраны окружающей среды штата Массачусетс (MassDEP) совместно финансировали исследование частной скважины.

    Чтобы устранить любые потенциальные опасения по поводу воздействия на здоровье человека, связанного с мышьяком и/или ураном, обнаруженным в пробах воды из частных колодцев, взятых в ходе этого проекта, Департамент общественного здравоохранения штата Массачусетс (MDPH) предложил некоторым участникам анализ мочи в качестве государственной услуги. исследования Геологической службы США. За дополнительной информацией обращайтесь в MDPH по телефону 617-624-5757 или 800-240-4266.

    Должен ли я беспокоиться о мышьяке и уране в моем личном колодце с питьевой водой?

    Для того, чтобы определить, следует ли вам беспокоиться о мышьяке и уране в вашей воде из частного колодца, вам сначала нужно знать, какие уровни присутствуют в воде из вашего колодца. MassDEP и MDPH рекомендуют всем домовладельцам проверять свои колодцы на наличие мышьяка и радионуклидов, таких как уран. Если результаты первоначального и последующего тестирования превышают общедоступные стандарты питьевой воды, вам следует рассмотреть возможность установки системы очистки воды для достижения этих стандартов.

    Пока я жду результатов анализов, можно ли пить воду? Или купаться в нем?

    Да. Основываясь на информации, доступной по результатам тестирования воды и сопутствующих исследований MDPH на мышьяк и уран в моче, вероятность того, что чей-либо колодец представляет опасность для здоровья, очень мала. Тем не менее, мы рекомендуем вам проверить вашу воду. Проживание в районе, где испытания USGS показывают, что грунтовые воды имеют больше шансов превысить стандарты питьевой воды по мышьяку и урану, не обязательно означает, что у вашего колодца будут проблемы. На самом деле мы ожидаем, что большинство частных скважин на этих участках не превышают нормы. Таким образом, вполне вероятно, но не обязательно, что ваша вода будет ниже стандартов. Единственный способ узнать наверняка, это хорошо проверить. Тем временем, если вы обеспокоены, вы можете использовать воду в бутылках, пока не получите результаты теста.

    Государственные нормы содержания мышьяка и урана в питьевой воде составляют 10 микрограммов и 30 микрограммов на литр воды соответственно (или 10 частей на миллиард и 30 частей на миллиард соответственно). Стандарты питьевой воды устанавливаются Агентством по охране окружающей среды США для защиты здоровья населения. Они основаны на потреблении человеком 2 литров (приблизительно 2 кварты или 8 чашек воды в день) воды каждый день в течение всей жизни. Фактические риски для здоровья зависят от трех ключевых факторов: уровня мышьяка и урана в питьевой воде; количество воды, фактически потребленной из вашей частной скважины; и сколько лет расходуется вода. Риски снижаются, если вы выпиваете менее 2 литров в день или потребляете воду меньше, чем всю жизнь.

    Почему мышьяк и уран могут присутствовать в колодезной воде?

    Некоторые районы Новой Англии могут содержать более высокие уровни мышьяка и урана в коренных породах, чем другие районы региона. Мышьяк можно найти в двух различных формах: органический мышьяк и неорганический мышьяк. Неорганические формы мышьяка обычно обнаруживаются в подземных водах и представляют большую опасность для здоровья, чем органические формы мышьяка (которые присутствуют в некоторых пищевых продуктах, например в рыбе), которые считаются менее токсичными. В результате естественных процессов мышьяк и уран могут покинуть горную породу и попасть в грунтовые воды в коренной породе. Если скальный колодец установлен в месте, которое перехватывает подземные воды, содержащие относительно высокие концентрации мышьяка и/или урана, то содержание мышьяка и/или урана, извлекаемого из колодца, может превышать общедоступные стандарты питьевой воды.

    Каковы нормы содержания мышьяка и урана в питьевой воде?

    Государственный и федеральный стандарт питьевой воды (PWS) для мышьяка составляет 0,010 миллиграмма на литр (мг/л) (или 10 частей на миллиард частей на миллиард). Стандарт питьевой воды для урана составляет 0,030 мг/л (или 30 частей на миллиард). Стандарты частных колодцев определяются местным советом здравоохранения (BOH) и обычно совпадают со стандартами штата и федеральными стандартами PWS.

    Показало ли исследование корреляцию между геологией и концентрацией мышьяка в колодцах коренных пород?

    Да. 478 проб воды из колодцев, отобранных для исследования, были распределены среди многих различных типов геологических единиц коренных пород, нанесенных на карту в районе исследования. Результаты показали корреляцию между вероятностью превышения общественного стандарта питьевой воды по мышьяку и типом коренных пород. Исследование также показало сильную корреляцию между концентрациями мышьяка и близостью скважины к зоне разлома Клинтон-Ньюбери. Зона разлома Клинтон-Ньюбери простирается от восточной оконечности границы Массачусетса и Коннектикута на юге до долины реки Мерримак на северо-востоке.

    Геологическая служба США спрогнозировала расчетное количество скважин в районе исследования, которое, как ожидается, превысит стандарт мышьяка, путем умножения вероятностей, полученных на основе результатов выборки, на расчетное количество скважин в этом районе.

    В результате этого метода был сделан прогноз о том, что примерно в 5700 жилых скальных колодцах из 90 000 в исследуемой области концентрация мышьяка может превышать общедоступный стандарт питьевой воды. По оценкам Геологической службы США, примерно 3800 из этих 5700 скважин не очищаются от мышьяка.

    Показало ли исследование корреляцию между геологией и концентрацией урана в скважинах коренных пород?

    Да. Результаты 478 проб, отобранных для исследования, показали корреляцию между вероятностью превышения общедоступного стандарта питьевой воды по урану и типом коренных пород. В целом скважины, пройденные в гранитных (гранитного типа) коренных породах, имели наибольшую вероятность превышения норматива.

    Геологическая служба США спрогнозировала оценочное количество скважин в районе исследования, которое, как ожидается, превысит урановый стандарт, умножив вероятности, полученные на основе результатов выборки для каждой отдельной единицы коренных пород, на оценочное количество скважин в каждой единице. Этот метод привел к прогнозу, что примерно 3300 жилых скальных колодцев из

    человек в районе исследования имеют концентрацию урана, превышающую общедоступный стандарт питьевой воды. По оценкам Геологической службы США, примерно 3000 из этих 3300 скважин не очищаются от урана.

    Какой процент образцов превысил общедоступные стандарты питьевой воды по содержанию мышьяка и урана?

    Результаты 478 проб из частных колодцев из 116 городов, которые были собраны и проанализированы для исследования Геологической службы США, следующие:

    • примерно 13% проб, взятых из случайно выбранных колодцев, превышали стандарт содержания мышьяка в питьевой воде в 10 частей на миллиард
    • менее чем в 3% всех собранных проб содержание урана в питьевой воде превышало 30 частей на миллиард.

    Каковы потенциальные последствия для здоровья, связанные с высоким содержанием мышьяка в питьевой воде?

    Кратковременное воздействие питьевой воды с более высоким содержанием неорганического мышьяка может привести к тошноте, рвоте, диарее, сердечно-сосудистым заболеваниям и последствиям для головного мозга (например, энцефалопатии), а также к снижению выработки красных и белых кровяных телец и нарушению функции нервной системы. Длительное употребление питьевой воды, содержащей неорганический мышьяк, может привести к ряду изменений кожи (например, пятнам потемневшей кожи, небольшим «мозолям» или «бородавкам» на ладонях, подошвах ног и туловище) и повреждению нервной системы ( то есть периферическая невропатия). Несколько исследований также показали, что длительное воздействие неорганического мышьяка может увеличить риск развития некоторых видов рака, включая рак легких, кожи, мочевого пузыря, печени, почек и простаты.

    Дополнительные ресурсы

    Каковы потенциальные последствия для здоровья, связанные с высоким содержанием урана в питьевой воде?

    Основным последствием воздействия повышенных уровней урана на здоровье является повреждение почек. Тем не менее, большинство эффектов, наблюдаемых у людей, были вызваны сильными кратковременными воздействиями, а некоторые профессиональные исследования показали обратное воздействие на почки после прекращения воздействия. Хроническое воздействие также может привести к почечным последствиям. Воздействие на здоровье природного урана в питьевой воде связано с химической токсичностью урана. Агентство по охране окружающей среды США установило свой стандарт питьевой воды для урана в первую очередь из-за его химической токсичности, но в качестве меры предосторожности также рассмотрело возможность урана вызывать рак после продолжительного воздействия питьевой воды, исходя из предположения, что любой компонент, обладающий радиоактивными свойствами, может вызвать рак.

    Дополнительные ресурсы

    Существуют ли другие радионуклиды, помимо урана, о которых мне следует беспокоиться?

    В исследовании Геологической службы США изучалось присутствие мышьяка и урана только в колодезной воде. Другие встречающиеся в природе радионуклиды, такие как газ радон и радий, обычно встречаются в тех же типах коренных пород, что и уран. Таким образом, несмотря на отсутствие данных о вероятности для газообразного радона и радия, районы с повышенной вероятностью повышенных концентраций урана, как правило, будут иметь повышенную вероятность повышенных концентраций радона и радия. Дополнительную информацию о стандартах общественной питьевой воды по радону и радию, воздействии на здоровье и вариантах лечения см. в разделе «Вопросы и ответы по радионуклидам».

    Подвергается ли общественное водоснабжение риску содержания мышьяка или урана в воде?

    Нет. Колодцы общественного водоснабжения, обслуживающие жилые дома, проверяются на мышьяк и уран. Любая общественная система водоснабжения, которая не может в обычном порядке соответствовать общественным стандартам питьевой воды, должна обеспечивать очистку. Они также обязаны сообщать о любых результатах качества воды, которые превышают общедоступные стандарты питьевой воды, в своем ежегодном отчете о доверии потребителей, который предоставляется их клиентам.

    Где можно проверить воду из колодца на мышьяк, уран и другие радионуклиды?

    Доступный для поиска список лабораторий, сертифицированных MassDEP, см. в электронной таблице лабораторий, сертифицированных MassDEP, которые тестируют соединения мышьяка, ниже.

    Дополнительные ресурсы

    Где я могу проверить пробу воды из колодца на концентрацию мышьяка 3 и мышьяка 5?

    Неорганический мышьяк встречается в природе в виде мышьяка 3 и мышьяка 5. Мышьяк 3 значительно труднее удалить из воды, чем мышьяк 5. Поэтому, если общая концентрация мышьяка в вашей колодезной воде превышает государственный стандарт питьевой воды, вам следует подумать о том, чтобы взять образец проверены на содержание мышьяка, чтобы выбрать вариант очистки, который удалит достаточное количество мышьяка для получения готовой воды, соответствующей стандарту. См. список лабораторий, сертифицированных MassDEP, которые проверяют мышьяк и уран (ссылка на предыдущий раздел).

    Включен ли мой город в изучаемую территорию?

    См. список карт города, ссылка на который приведена ниже, чтобы определить, доступна ли информация о вероятности мышьяка или урана для вашего города. Если ваш город указан в списке, вы также найдете ссылки на PDF-карты вашего города для зон вероятности мышьяка и урана.

    Дополнительные ресурсы

    Где я могу найти конкретную информацию о вероятности наличия мышьяка или урана в колодезной воде?

    Вы можете использовать Инструмент поиска скважин для получения оценок вероятности содержания мышьяка и урана, чтобы ввести адрес улицы и название города, чтобы получить информацию о вероятности того, что вода из вашего колодца может не соответствовать общедоступным стандартам питьевой воды по содержанию мышьяка и урана. Кроме того, доступны карты городов, на которых показаны зоны вероятности мышьяка и урана.

    Как инструмент для определения местоположения скважин, так и соответствующие карты городов для зон вероятности залегания урана определяют участки коренных пород, обозначенные как гранит или пегматит, которые расположены за пределами области, охваченной исследованием Геологической службы США. Хотя статистические данные о вероятности появления урана для этих конкретных гранитных или пегматитовых единиц отсутствуют, колодцы коренных пород в этих типах горных пород обычно имеют повышенную вероятность содержания природного радона, радия и урана в концентрациях, превышающих пределы питьевой воды.

    Дополнительные ресурсы

    Какую обработку нужно установить для удаления мышьяка или урана и сколько это будет стоить?

    Мышьяк:
    Обычно в воде присутствуют два варианта или разновидности неорганического мышьяка: «мышьяк 3» и «мышьяк 5». Это важно, потому что «мышьяк 3» очень трудно удалить из воды, и его необходимо заменить или окислить до «мышьяка 5», прежде чем его можно будет удалить. Для удаления мышьяка чаще всего используются анионообменные фильтры, фильтры обратного осмоса, активированный оксид алюминия и другие типы фильтров с адсорбционными средами. В зависимости от того, обрабатываете ли вы только воду, поступающую в кухонную раковину, или всю воду, поступающую в ваш дом, средняя стоимость установки систем удаления мышьяка составляет от 1200 до 3000 долларов (не включая стоимость предварительной обработки для преобразования). мышьяк 3» до «мышьяк 5», что может потребоваться, а может и не потребоваться).

    Уран:
    Обратный осмос и анионообмен чаще всего используются для удаления урана. В зависимости от того, обрабатываете ли вы только воду, поступающую в кухонную раковину, или всю воду, поступающую в ваш дом, средняя стоимость установки систем удаления урана составляет от 1600 до 2500 долларов.

    Нужно ли очищать всю воду или только воду, поступающую в кран кухонной раковины?

    Для тех колодцев, которые превышают стандарты питьевой воды по мышьяку и урану в Массачусетсе, концентрации, как правило, опасны только для приема внутрь. Таким образом, обработка только той воды, которая используется для питья и приготовления пищи, — это, как правило, все, что требуется для защиты здоровья вашей семьи. Если концентрация мышьяка в вашем организме превышает 0,5 миллиграмма на литр (мг/л) (или 500 частей на миллиард), что крайне редко встречается в Массачусетсе, вам следует подумать о лечении всего дома.

    Могу ли я сбрасывать отработанную воду из моей системы очистки обратного осмоса в мою септическую систему согласно Разделу 5?

    Ответ зависит от типа системы очистки обратного осмоса, концентрации мышьяка и урана в сырой воде и использования имущества. Сбросы воды обратного осмоса на месте из точек использования (POU) (т. Е. Очистка только воды, поступающей в кухонную раковину). Устройства для очистки питьевой воды в жилых домах с одной-четырьмя квартирами. 5 септическая система, но только при соблюдении следующих предельных значений концентрации мышьяка и урана в сырой (неочищенной) воде:

    • общая концентрация мышьяка составляет менее 1 миллиграмма на литр (мг/л) или 1 часть на миллион (ppm) в сырой (неочищенной) воде; и
    • свяжитесь с Джо Черрутти или Стивом Халлемом (контактная информация указана в конце этой страницы) для получения информации о допустимых концентрациях урана в неочищенной воде.

    Устройства POU, которые используются на объектах, состоящих из более чем четырех жилых единиц, или используются для целей, отличных от жилых, не могут сбрасываться в септическую систему Раздела 5. Системы обработки питьевой воды в точке входа (POE) (т. Е. Обработка всей воды, поступающей в дом) не могут сбрасываться в септическую систему Раздела 5. Сточные воды из системы очистки питьевой воды, которые нельзя сбрасывать в септическую систему Раздела 5, должны сбрасываться либо в муниципальную систему очистки сточных вод, либо в сухой колодец. Для сброса в сухой колодец требуется подача заявки на регистрацию MassDEP Underground Injection Control (UIC), за исключением объектов, которые используются только для проживания одной семьи.

    Могу ли я сбрасывать сточные воды, содержащие мышьяк или уран, из моей системы очистки питьевой воды в сухой колодец?

    Для всех сбросов из систем очистки воды требуется регистрация UIC, одобренная MassDEP, за исключением свойств, которые используются только для проживания одной семьи. См. обсуждение предыдущего вопроса об исключении для некоторых сбросов обратного осмоса (RO) для систем POU. Чтобы утвердить заявку на регистрацию UIC для сбросов системы очистки воды, MassDEP оценивает, может ли сброс привести к нарушению стандарта питьевой воды в принимающих грунтовых водах. Из-за эффектов разбавления MassDEP может одобрить сбросы в сухой колодец, которые превышают общедоступные стандарты питьевой воды, но не существует установленного предела, и каждое заявление рассматривается в каждом конкретном случае. MassDEP не может одобрить любые сбросы воды, в которых содержание мышьяка превышает федеральный порог содержания опасных отходов (характеристика токсичности) в 5 миллиграммов на литр (мг/л) или 5 частей на миллион (ppm), или в которых содержание урана превышает 0,600 мг/л или 600 частей на миллиард (ppb). Свяжитесь с Джо Черутти или Стивом Халлемом (контактная информация указана в конце этой страницы) для получения информации о допустимых концентрациях урана в сбросе в скважину UIC.

    Есть ли средства для домовладельцев с низким доходом для установки системы очистки?

    Финансирование Программы ссуд и грантов на благоустройство и ремонт домов Министерства сельского хозяйства США (USDA) (раздел 504) может быть доступно домовладельцам с очень низким доходом в соответствующих сообществах. Эти ссуды или гранты могут быть использованы домовладельцами для проведения общего ремонта с целью улучшения или модернизации, защиты от погодных условий и/или устранения нарушений кодекса. безопасное и санитарное состояние жилого помещения. Эта программа доступна в сельских населенных пунктах и ​​малых городах с населением до 10 000 человек. Некоторые сообщества с населением от 10 000 до 20 000 человек могут иметь право на участие. Домовладельцам следует посетить веб-сайт USDA Rural Development, чтобы узнать о требованиях и о том, как подать заявку, на веб-сайте USDA Rural Development: https://www.rd.usda.gov/ma.

    Кто регулирует частные колодцы?

    В соответствии с Общим законом штата Массачусетс (MGL Ch.111 s.122) местные советы по здравоохранению (BOH) имеют основную юрисдикцию в отношении регулирования частных колодцев. Местный BOH уполномочен принять Положение о частных колодцах, которое устанавливает критерии для размещения частных колодцев, строительства, качества и количества воды.

    Что делает государство с этой проблемой?

    MassDEP предоставил большую часть финансирования исследования Геологической службы США, в ходе которого были определены зоны высокой и низкой вероятности, а MDPH предоставил финансирование для картографирования частных скважин, которое использовалось в исследовании Геологической службы США, а MDPH финансировал и проводил анализ мочи, который проводился одновременно. с исследованием Геологической службы США.

    MassDEP разработал Инструмент определения местоположения скважин для получения оценок вероятности содержания мышьяка и урана, чтобы предоставить владельцам частных скважин простой способ получить дополнительную информацию о вероятности того, что вода из их коренных скважин превышает общедоступные стандарты питьевой воды для мышьяка и урана.

    MassDEP, MDPH и Геологическая служба США предоставили и будут продолжать поддерживать связь с местными советами здравоохранения, информируя их о результатах исследования и о том, куда они могут направить владельцев частных скважин для получения дополнительной информации.

    MassDEP разработал карты в формате PDF зон вероятности содержания мышьяка и урана в масштабе города и сделал их доступными для широкой общественности и местных советов здравоохранения для городов, для которых существует информация Геологической службы США о вероятности того, что мышьяк или уран превышают общедоступные стандарты питьевой воды. , или для которых существуют типы коренных пород, которые, как известно, имеют повышенную вероятность содержания урана и других радионуклидов в концентрациях, превышающих общедоступные стандарты питьевой воды.

    MDPH предоставляет информацию о влиянии мышьяка и урана на здоровье любому владельцу частной скважины, который ищет информацию.

    MassDEP предоставляет информацию о доступных технологиях очистки любому частному владельцу скважины, который ищет информацию.

    Обратите внимание, что MassDEP не регулирует частные колодцы с питьевой водой.

    Может ли мой питомец пить воду, содержание мышьяка/урана в которой превышает стандарт питьевой воды?

    Различные виды домашних животных могут иметь различную чувствительность к мышьяку и урану в питьевой воде. Тем не менее, рекомендуется давать домашним животным питьевую воду, соответствующую тем же стандартам/рекомендациям, которые установлены для людей.

    К кому я могу обратиться за дополнительной информацией или вопросами?

    Если у вас есть дополнительные вопросы, на которые нет ответов в этом документе и связанных ссылках, вы можете получить дополнительную информацию по следующему адресу:

    По вопросам, касающимся отчета Геологической службы США, обращайтесь в Центр водных исследований Геологической службы США: https://www.usgs. gov/centers/new-england-water/connect

    По вопросам, касающимся воздействия мышьяка и урана на здоровье, обращайтесь в MDPH по телефону 617-624-5757 или 800-240-4266.

    По вопросам, касающимся лечения мышьяком, обращайтесь к Joe Cerutti, MassDEP, по телефону 617-292-5859, электронная почта: [email protected].

    По вопросам лучевой терапии обращайтесь к Steve Hallem, MassDEP, по телефону 617-292-5681, электронная почта: stephen. [email protected].

    Помогите нам улучшить Mass.gov своими отзывами

    Вы нашли то, что искали на этой веб-странице? Если у вас есть предложения по сайту, сообщите нам. Как мы можем улучшить страницу? *

    Пожалуйста, не указывайте личную или контактную информацию.

    Отзывы будут использованы только для улучшения сайта. Если вам нужна помощь, посетите страницу «Контакты и сервисный центр MassDEP». Пожалуйста, ограничьте ввод до 500 символов.

    Пожалуйста, удалите любую контактную информацию или личные данные из вашего отзыва.

    Если вам нужна помощь, посетите страницу контактов и сервисного центра MassDEP.

    Пожалуйста, сообщите нам, как мы можем улучшить эту страницу.

    Пожалуйста, удалите любую контактную информацию или личные данные из вашего отзыва.

    Если вам нужна помощь, посетите страницу контактов и сервисного центра MassDEP.